Tópicos de Matemática Elementar - Volume 3 Introdução à Análise [3, 2 ed.]
 9788583370079

  • 0 0 0
  • Like this paper and download? You can publish your own PDF file online for free in a few minutes! Sign Up
File loading please wait...
Citation preview

Tópicos de Matemática Elementar: introdução à análise Copyright© 2013 Antonio Caminha Muniz Neto. Direitos reservados pela Sociedade Brasileira de Matemática Coleção Professor de Matemática

Tópicos de Maten1ática Elen1entar

Volume 3 Introdução à Análise

Comitê Editorial Abdênago Alves de Barros Abramo Refez (Editor-Chefe) Djairo Guedes de Figueiredo José Alberto Cuminato Roberto lmbuzeiro Oliveira Sílvia Regina Costa Lopes

Antonio Caminha Muniz Neto

Assessor Editorial Tiago Costa Rocha

y

Capa Pablo Diego Regino Sociedade Brasileira de Matemática Presidente: Marcelo Viana Vice-Presidente: Vanderlei Horita Primeiro Secretário: Ali Tahzibi Segundo Secretário: Luiz Manoel de Figueiredo Terceiro Secretário: Marcela Souza Tesoureiro: Carmen Mathias

Gr

Distribuição e vendas Sociedade Brasileira de Matemática Estrada Dona Castorina, 110 Sala 109 - Jardim Botânico 22460-320 Rio de Janeiro RJ Telefones: (21) 2529-5073 / 2529-5095 http://www.sbm.org.br / email:[email protected]

a

Xo

X

b

X

ISBN 978-85-8337-007-9

MUNIZ NETO, Antonio Caminha. Tópicos de Matemática Elementar: introdução à análise funcional / Caminha Muniz Neto. -2.ed. -- Rio de Janeiro: SBM, 2012. v.3 ; 331 p. (Coleção Professor de Matemática; 26)

2ª edição 2013 Rio de Janeiro

F(x) =

ISBN 978-85-8337-007-9 1. Conceitos de Função. 3. Funções Contínuas. I. Título.

2. Gráficos de Função. 4. Limites e Derivadas.

.!SBM

COLEÇÃO DO PROFESSOR DE MATEMÁTICA

(x 0 ) +

1:

J(t)dt,

•,. SBM i11II

COLEÇÃO DO PROFESSOR DE MATEMÁTICA Logaritmos - E. L. Lima Análise Combinatória e Probabilidade com as soluções dos exercícios A. C. Morgado, J. B. Pitombeira, P. C. P. Carvalho e P. Fernandez Medida e Forma em Geometria (Comprimento, Área, Volume e Semelhança) E. L. Lima Meu Professor de Matemática e outras Histórias • E. L. Lima Coordenadas no Plano com as soluções dos exercícios - E. L. Lima com a colaboração de P. C. P. Carvalho Trigonometria, Números Complexos - M. P. do Carmo, A. C. Morgado e E. Wagner, Notas Históricas de J. B. Pitombeira Coordenadas no Espaço - E. L. Lima Progressões e Matemática Financeira - A. C. Morgado, E. Wagner e S. C. Zani Construções Geométricas - E. Wagner com a colaboração de J. P..Q. Carneiro Introdução à Geometria Espacial - P. C. P. Carvalho Geometria Euclidiana Plana - J. L. M. Barbosa Isometrias - E. L. Lima A Matemática do Ensino Médio Vol. 1 - E. L. Lima, P. C. P. Carvalho, E. Wagner e A. C. Morgado A Matemática do Ensino Médio Vol. 2 - E. L. Lima, P. C. P. Carvalho, E. Wagner e A. C. Morgado A Matemática do Ensino Médio Vol. 3 - E. L. Lima, P. C. P. Carvalho, E. Wagner e A. C. Morgado Matemática e Ensino - E. L. Lima Temas e Problemas - E. L. Lima, P. C. P. Carvalho, E. Wagner e A. C. Morgado Episódios da História Antiga da Matemática - A. Aaboe Exame de Textos: Análise de livros de Matemática - E. L. Lima A Matemática do Ensino Medio Vol. 4 - Exercicios e Soluções - E. L. Lima, P. C. P. Carvalho, E. Wagner e A. C. Morgado Construções Geométricas: Exercícios e Soluções - S. Lima Netto Um Convite à Matemática - D.C de Morais Filho Tópicos de Matemática Elementar - Volume 1 - Números Reais - A. Caminha Tópicos de Matemática Elementar - Volume 2 - Geometria Euclidiana Plana - A. Caminha Tópicos de Matemática Elementar - Volume 3 - Introdução à Análise - A. Caminha Tópicos de Matemática Elementar - Volume 4 - Combinatória - A. Caminha Tópicos de Matemática Elementar - Volume 5 - Teoria dos Números - A. Caminha

A meus filhos Gabriel e Isabela, na esperança de que um dia leiam este livro.

Sumário

Prefácio Prefácio à segunda edição 1 O Conceito de Função 1.1 Definições e exemplos . 1.2 Monotonicidade, extremos e imagem 1.3 Composição de funções . . . . . . 1.4 Inversão de funções . . . . . . . . 1.5 Funções definidas implicitamente 2

Gráficos de Funções 2.1 Generalidades e exemplos 2.2 Funções trigonométricas .

3 Mais sobre Números Reais 3.1 Limites de sequências . 3.2 Séries de números reais 3.3 O lema de Kronecker .

IX XVII 1

2 14

28 39 44

55 56 70 77 78 93 107

VIII 4

Tópicos de Matemática Elementar 3

Funções Contínuas 4.1 Definição e exemplos ....... 4.2 O teorema do valor intermediário 4.3 Continuidade sequencial ...

117 117 128 141

5 A Concavidade de uma Função 5.1 A integral de urna função contínua 5.2 A função logaritmo natural . 5.3 Funções côncavas e convexas 5.4 A desigualdade de Jensen

151 152 163 175 184

6 Limites e Derivadas 6.1 Limites de funções ......... 6.2 Propriedades básicas de derivadas . 6.3 A primeira variação de urna função 6.4 O teorema fundamental do Cálculo 6.5 A segunda variação de urna função

195 196 213 230 241 254

7 Soluções e Sugestões

265

Referências

309

A Glossário

317

Prefácio

Esta coleção evoluiu a partir de sessões de treinamento para olimpíadas de Matemática, por rnirn ministradas para alunos e professores · do Ensino Médio, várias vezes ao longo dos anos de 1992 a 2003 e, rnais recentemente, corno orientador do Programa de Iniciação Científica para os premiados na Olimpíada Brasileira de Matemática das Escolas Públicas (OBMEP) e do Projeto Amílcar Cabral de cooperação educacional entre Brasil e Cabo Verde. Idealmente, planejei o texto corno urna mistura entre urna iniciação suave e essencialmente autocontida ao fascinante mundo das competições de Matemática, alérn de urna bibliografia auxiliar aos estudantes e professores do secundário interessados ern aprofundar seus conhecimentos rnaternáticos. Resurnidarnente, seu propósito primordial é apresentar ao leitor urna abordagem de quase todos os conteúdos geralmente constantes dos currículos do secundário, e que seja ao rnesrno tempo concisa, não excessivamente tersa, logicamente estruturada e rnais aprofundada que a usual. Na estruturação dos livros, rne ative à rnáxirna do eminente rnaternático húngaro-americano George Pólya, que dizia não se poder fazer

X

Tópicos de Matemática Elementar 3

Matemática sem sujar as mãos. Assim sendo, em vários pontos deixei a cargo do leitor a tarefa de verificar aspectos não centrais aos desenvolvimentos principais, quer na forma de detalhes omitidos em demonstrações, quer na de extensões secundárias da teoria. Nestes casos, frequentemente referi o leitor a problemas específicos, os quais se encontram marcados com * e cuja análise e solução considero parte integrante e essencial do texto. Colecionei ainda, em cada seção, outros tantos problemas, cuidadosamente escolhidos na direção de exercitar os resultados principais elencados ao longo da discussão, bem como estendê-los. Uns poucos destes problemas são quase imediatos, ao passo que a maioria, para os quais via de regra oferto sugestões precisas, é razoavelmente difícil; no entanto, insto veementemente o leitor a debruçar-se sobre o maior número possível deles por tempo suficiente para, ainda que não os resolva todos, passar a apreciá-los como corpo de conhecimento adquirido. O primeiro volume discorre sobre vários aspectos relevantes do conjunto dos números reais e de álgebra elementar, no intuito de munir o leitor dos requisitos necessários ao estudo dos tópicos constantes dos volumes subsequentes. Após começar com uma discussão não axiomática das propriedades mais elementares dos números reais, são abordados, em seguida, produtos notáveis, equações e sistemas de equações, sequências elementares, indução matemática e números binomiais; o texto finda com a discussão de várias desigualdades algébricas importantes, notadamente aquela entre as médias aritmética e geométrica, bem como as desigualdades de Cauchy, de Chebychev e de Abel. Dedicamos o segundo volume a uma iniciação do leitor à geometria Euclidiana plana, inicialmente de forma não axiomática e enfatizando construções geométricas elementares. Entretanto, à medida em que o texto evolui, o método sintético de Euclides - e, consequentemente, demonstrações - ganha importância, principalmente com a discussão dos conceitos de congruência e semelhança de triângulos; a partir desse ponto, vários belos teoremas clássicos da geometria, usualmente ausen-

Antonio Caminha M. Neto

XI

tes dos livros-texto do secundário, fazem sua aparição. Numa terceira etapa, o texto apresenta outros métodos elementares usuais no estudo da geometria, quais sejam, o método analítico de R. Descartes, a trigonometria e o uso de vetores; por sua vez, tais métodos são utilizados tanto para reobter resultados anteriores de outra(s) maneira(s) quanto para deduzir novos resultados. De posse do traquejo algébrico construído no volume inicial e do aparato geométrico do volume dois, discorremos no volume três sobre aspectos elementares de funções e certos excertos de cálculo diferencial e integral e análise matemática, os quais se fazem necessários em certos pontos dos três volumes restantes. Prescindindo, inicialmente, das noções básicas do Cálculo, elaboramos, dentre outros, as noções de gráfico, monotonicidade e extremos de funções, bem como examinamos o problema da determinação de funções definidas implicitamente por relações algébricas. Na continuação, o conceito de função contínua é apresentado, primeiramente de forma intuitiva e, em seguida, axio- · mática, sendo demonstrados os principais resultados pertinentes. Em especial, utilizamos este conceito para estudar a convexidade de gráficos - culminando com a demonstração da desigualdade de J. Jensen e o problema da definição rigorosa da área sob o gráfico de uma função contínua e positiva - que, por sua vez, possibilita a apresentação de uma construção adequada das funções logaritmo natural e exponencial. O volume três termina com uma discussão das propriedades mais elementares de derivadas e do teorema fundamental do cálculo, os quais são mais uma vez aplicados ao estudo de desigualdades, em especial da desigualdade entre as médias de potências. O volume quatro é devotado à análise combinatória. Começamos revisando as técnicas mais elementares de contagem, enfatizando as construções de bijeções e argumentos recursivos como estratégias básicas. Na continuação, apresentamos um apanhado de métodos de contagem um tanto mais sofisticados, como o princípio da inclusão exclusão e os métodos de contagem dupla, do número de classes de

XII

Tópicos de Matemática Elementar 3

equivalência e mediante o emprego de métricas em conjuntos finitos. A cena é então ocupada por funções geradoras, onde a teoria elementar de séries de potências nos permite discutir de outra maneira problemas antigos e introduzir problemas novos, antes inacessíveis. Terminada nossa excursão pelo mundo da contagem, enveredamos pelo estudo do problema da existência de uma configuração especial no universo das configurações possíveis, utilizando para tanto o princípio das gavetas de G. L. Dirichlet - vulgo "princípio das casas dos pombos" -, um célebre teorema de R. Dilworth e a procura e análise de invariantes associados a problemas algorítmicos. A última estrutura combinatória que discutimos é a de um grafo, quando apresentamos os conceitos básicos usuais da teoria com vistas à discussão de três teoremas clássicos importantes: a caracterização da existência de caminhos Eulerianos, o teorema de A. Cayley sobre o número de árvores rotvladas e o teorema extremal de P. Turán sobre a existência de subgrafos completos em um grafo. Passamos em seguida, no quinto volume, à discussão dos conceitos e resultados mais elementares de teoria dos números, ressaltando-se inicialmente a teoria básica do máximo divisor comum e o teorema fundamental da aritmética. Discutimos também o método da descida de P. de Fermat como ferramenta para provar a inexistência de soluções inteiras para certas equações diofantinas, e resolvemos também a famosa equação de J. Pell. Em seguida, preparamos o terreno para a discussão do famoso teorema de Euler sobre congruências, construindo a igualmente famosa função de Euler com o auxílio da teoria mais geral de funções aritméticas multiplicativas. A partir daí, o livro apresenta formalmente o conceito de congruência de números em relação a um certo módulo, discutindo extensivamente os resultados usualmente constantes dos cursos introdutórios sobre o assunto, incluindo raízes primitivas, resíduos quadráticos e o teorema de Fermat de caracterização dos inteiros que podem ser escritos como soma de dois quadrados. O grande diferencial aqui, do nosso ponto de vista, é o calibre dos

Antonio Caminha M. Neto

XIII

exemplos discutidos e dos problemas propostos ao longo do texto, boa parte dos quais oriundos de variadas competições ao redor do mundo. Finalmente, números complexos e polinômios são os objetos de estudo do sexto e último volume da coleção. Para além da teoria correspondente usualmente estudada no secundário - como a noção de grau, o algoritmo da divisão e o conceito de raízes de polinômios -, vários são os tópicos não padrão abordados aqui. Dentre outros, destacamos inicialmente a utilização de números complexos e polinômios como ferramentas de contagem e a apresentação quase completa de uma das mais simples demonstrações do teorema fundamental da álgebra. A seguir, estudamos o famoso teorema de I. Newton sobre polinômios simétricos e as igualmente famosas desigualdades de Newton, as quais estendem a desigualdade entre as médias aritmética e geométrica. O próximo tema concerne os aspectos básicos da teoria de interpolação de polinômios, quando dispensamos especial atenção aos polinômios interpoladores de J. L. Lagrange. Estes, por sua vez, são utilizados para resolver sistemas lineares de Vandermonde sem o recurso à álgebra linear, os quais, a seu turno, possibilitam o estudo de uma classe particular de sequências recorrentes lineares. O livro termina com o estudo das propriedades de fatoração de polinômios com coeficientes inteiros, racionais ou pertencentes ao conjunto das classes de congruência relativas a algum módulo primo, seguido do estudo do conceito de número algébrico. Há, aqui, dois pontos culminantes: por um lado, uma prova mais simples do fechamento do conjunto dos números algébricos em relação às operações aritméticas básicas; por outro, o emprego de polinômios ciclotômicos para provar um caso particular do teorema de Dirichlet sobre primos em progressões aritméticas. Várias pessoas contribuíram ao longo dos anos, direta ou indiretamente, para que um punhado de anotações em cadernos pudesse transformar-se nesta coleção de livros. Os ex-professores do Departamento de Matemática da Universidade Federal do Ceará, Marcondes

XIV

Tópicos de Matemática Elementar 3

Cavalcante França, João Marques Pereira, Guilherme Lincoln Aguiar Ellery e Raimundo Thompson Gonçalves, ao criarem a Olimpíada Cearense de Matemática na década de 1980, motivaram centenas de jovens cearenses, dentre os quais eu me encontrava, a estudarem mais Matemática. Meu ex-professor do Colégio Militar de Fortaleza, Antônio Valdenísio Bezerra, ao convidar-me, inicialmente para assistir a suas aulas de treinamento para a Olimpíada Cearense de Matemática e posteriormente para dar aulas consigo, iniciou-me no maravilhoso mundo das competições de Matemática e influenciou definitivamente minha escolha profissional. Os comentários de muitos de vários de ex-alunos contribuíram muito para o formato final de boa parte do material aqui colecionado; nesse sentido, agradeço especialmente a João Luiz de Alencar Araripe Falcão, Roney Rodger Sales de Castro, Marcelo Mendes de Oliveira, Marcondes Cavalcante França Jr., Marcelo Cruz de Souza, Eduardo Cabral Balreira, Breno de Alencar Araripe Falcão, Fabrício Siqueira Benevides, Rui Facundo Vigelis, Daniel Pinheiro Sobreira, Antônia Taline de Souza Mendonça, Carlos Augusto David Ribeiro, Samuel Barbosa Feitosa, Davi Máximo Alexandrino Nogueira e Yuri Gomes Lima. Vários de meus colegas professores teceram comentários pertinentes, os quais foram incorporados ao texto de uma ou outra maneira; agradeço, em especial, a Fláudio José Gonçalves, Francisco José da Silva Jr., Onofre Campos da Silva Farias, Emanuel Augusto de Souza Carneiro, Marcelo Mendes de Oliveira, Samuel Barbosa Feitosa e Francisco Bruno de Lima Holanda. Os professores João Lucas Barbosa e Hélio Barros deram-me a conclusão de parte destas notas como alvo a perseguir ao me convidarem a participar do Projeto Amílcar Cabral de treinamento dos professores de Matemática da República do Cabo Verde. Meus colegas do Departamento de Matemática da Universidade Federal do Ceará, Abdênago Alves de Barros, José Othon Dantas Lopes, José Robério Rogério e Fernanda Esther Camillo Camargo, bem como meu orientando de iniciação científica Itamar Sales de Oliveira Filho, leram partes do texto final e oferece-

Antonio Caminha M. Neto

XV

ram várias sugestões. Os pareceristas indicados pela SBM opinaram decisivamente para que os livros certamente resultassem melhores que a versão inicial por mim submetida. O presidente da SBM, professor Hilário Alencar da Silva, o antigo editor-chefe da SBM, professor Roberto Imbuzeiro de Oliveira, bem como o novo editor-chefe, professor Abramo Refez, foram sempre extremamente solícitos e atenciosos comigo ao longo de todo o processo de edição. Por fim, quaisquer erros ou incongruências que ainda se façam presentes, ou omissões na lista acima, são de minha inteira responsabilidade. Por fim e principalmente, gostaria de agradecer a meus pais, Antonio Caminha Muniz Filho e Rosemary Carvalho Caminha Muniz, e à minha esposa Mônica Valesca Mota Caminha Muniz. Meus pais me fizeram compreender a importância do conhecimento desde a mais tenra idade, sem nunca terem medido esforços para que eu e meus irmãos desfrutássemos o melhor ensino disponível; minha esposa brindou-me com a harmonia e o incentivo necessários à manutenção de meu ânimo. e humor, em longos meses de trabalho solitário nas madrugadas. Esta coleção de livros também é dedicada a eles.

FORTALEZA, JANEIRO de 2012

Antonio Caminha M. Neto

r XVI

Tópicos de Matemática Elementar 3

Prefácio à 2-ª- edição

Para a segunda edição fiz uma extensa revisão do texto e dos problemas propostos, corrigindo várias imprecisões de língua portuguesa e de Matemática. Adicionei também alguns problemas novos, no intuito de melhor exercitar certos pontos da teoria, os quais não se encontravam adequadamente contemplados pelos problemas propostos à primeira edição. Nesta segunda edição as sugestões e soluções aos problemas propostos foram colecionadas em um capítulo separado (o capítulo 7, para este volume); adicionalmente, apresentei sugestões ou soluções a todos os problemas com algum grau de dificuldade. Por fim, gostaria de aproveitar o ensejo para agradecer à comunidade matemática brasileira, em geral, e a todos os leitores que me enviaram sugestões ou correções, em particular, o excelente acolhimento desfrutado pela primeira edição desta obra.

FORTALEZA, OUTUBRO de 2013 Antonio Caminha M. Neto

1 .

!í,li

XVIII

Tópicos de Matemática Elementar 3

CAPÍTULO 1

O Conceito de Função

De posse do ferramental algébrico desenvolvido no volume 1, começamos, neste capítulo, o desenvolvimento das propriedades elementares de funções. Como naquele volume, assumiremos do leitor uma relativa familiaridade com conjuntos e operações elementares com os mesmos. Após apresentar os conceitos mais básicos sobre funções, como as noções de domínio, contradomínio e imagem, alguns exemplos relevantes são introduzidos. Em seguida, introduzimos as noções de monotonicidade e valores extremos, discutindo vários exemplos que, apesar de elementares, se revelarão bastante úteis. O capítulo continua com o estudo das operações de composição e inversão de funções, e culmina com a orquestração de todo o material discutido no estudo de funções definidas implicitamente por relações algébricas, as quais podem ser vistas como análogos discretos de equações diferenciais ordinárias.

Tópicos de Matemática Elementar 3

2

1.1

Definições e exemplos

Antonio Caminha M. Neto

3

não correspondem a funções.

Sejam dados conjuntos não vazios X e Y. Informalmente, uma função f de X em Y é uma regra que associa a cada x E X um único y E Y. Por vezes podemos visualizar uma função f : X --+ Y de uma maneira mais concreta por meio de diagramas como o da figura 1.1, onde cada seta indica que elemento y E Y está associado a cada

X

y

xEX.

y

X

Figura 1.2: X tem elementos dos quais não parte seta alguma.

Figura 1.1: exemplo de função

f de

X em Y.

Escrevemos f : X --+ Y para denotar que f é uma função de X em Y. Nesse caso, o elemento y E Y associado a x E X por fé denotado por y = f (x), sendo denominado a imagem de x E X pela função f. No exemplo da figura 1.1, temos X = {1, 2, 3}, Y = {a, b, e, d} e f(l) = a, f(2) = a, f(3) = e. Assim, a é a imagem de 1 e de 2 por f, e e é a imagem de 3 por f. Como atestado pelo exemplo acima, a definição de função permite que, no diagrama correspondente, um ou mais elementos de Y não recebam setas ou, ainda, que um ou mais elementos de Y recebam mais de uma seta (observe que ambas essas possibilidades estão presentes na figura 1.1). Note, contudo, que os diagramas das figuras 1.2 e 1.3

A situação da figura 1.2 é proibida porque não há nenhuma seta partindo do elemento 1 E X. A situação da figura 1.3 é proibida porque do elemento 1 E X parte mais de uma seta. As três definições a seguir explicitam alguns tipos extremamente úteis de funções.

Definição 1.1. Dados conjuntos não vazios X e Y e fixado um elemento e E Y, a função constante e de X em Y é a função f : X --+ Y tal que f(x) = e para todo x E X. No caso extremo da função constante e igual a e, definida acima, todo x E X está associado a um mesmo y E Y, qual seja, y = e. Contudo, as condições impostas na "definição" de função são plenamente satisfeitas, i.e., todo x E X está associado a um único y E X.

Definição 1.2. Dado um conjunto não vazio X, a função identidade de X, denotada por ldx : X--+ X, é a função dada por Idx(x) = x, para todo x E X.

' t'.

Tópicos de Matemática Elementar 3

4

y

X

Antonio Caminha M. Neto

5

Definição 1.4. Dados conjuntos não vazios X e Y, uma relação de X em Y (ou entre X e Y, nessa ordem) é um subconjunto R do produto cartesiano X x Y, i.e., R é um conjunto de pares ordenados do tipo (x, y), com x E X e y E Y. Se Ré uma relação de X em X, diremos simplesmente que R é uma relação em X. Exemplo 1.5. Se X= {1, 2, 3} e Y R

Figura 1.3: X tem elementos dos quais parte mais de uma seta.

Assim como no exemplo anterior, é imediato que as condições exigidas pela "definição" de função estão satisfeitas, de sorte que Idx é realmente uma função. Para o que segue, fixado arbitrariamente n EN, denotamos por ln o conjunto dos n primeiros números naturais, i.e., ln = {1, 2, ... , n }. Por exemplo, 11 = {1 }, 12 = {1, 2}, 13 = {1, 2, 3} e assim por diante.

Definição 1.3. Uma sequência (infinita) de números reais é uma função f : N--+ R Uma sequência (finita) de números reais é uma função f : ln --+ IR, para algum n E N. Dada uma sequência f : N --+ IR (resp. f : ln --+ IR) é costume denotar, para k ~ 1 inteiro, ak = f(k). Desta forma, obtemos a notação usual para sequências, qual seja, a1 =

f(l), a2

=

f(2), a3

=

f(3), ....

Como no capítulo 4 do volume 1, denotamos a sequência em questão simplesmente por (ak)k2:1 (resp. (ak)i::;k::;n)· De um ponto de vista matematicamente mais rigoroso, uma função é um caso particular de uma relação entre dois conjuntos, de acordo com a seguinte

= {(x, y)

=

{2, 3, 4, 5}, o conjunto

E X x Y;

x

~

y}

é a relação de X em Y dada por R = {(2, 2), (3, 2), (3, 3)}; de fato, esses são os únicos pares ordenados (x, y), com x E { 1, 2, 3}, y E {2, 3, 4, 5} e tais que x ~ y.

O exemplo acima é um caso particular de um procedimento óbvio a que podemos recorrer para construirmos relações específicas R entre conjuntos não vazios X e Y: basta especificarmos, de alguma maneira, um subconjunto do produto cartesiano X x Y. Aqueles pares ordenados de X x Y que satisfizerem a especificação prescrita serão os elementos de R. Sendo (x, y) um tal par, diremos que x e y são relacionados por R. Se R é uma relação de X em Y, então R e X x Y por definição. Reciprocamente, escolhido um par ordenado (x, y) E X x Y, pode ocorrer que (x, y) E R ou (x, y) 1 R (i.e., que x e y sejam relacionados por R, ou não). No primeiro caso, escrevemos x Ry; no segundo, x lJ, y. Em símbolos,

x Ry {::} (x, y)

E

R.

(1.1)

Assim é que, para a relação do exemplo acima, temos 3 R 2 mas 2 }J, 3, uma vez que 2 ~ 3 é falso. Dentre todos os tipos de relação que podemos considerar entre dois conjuntos não vazios, o principal é aquele dado pela seguinte

6

Tópicos de Matemática Elementar 3

Definição 1.6. Dados conjuntos não vazios X e Y, uma relação X em Y é uma função se a seguinte condição for satisfeita:

f de

\:;/x E X, :3 um único y E Y; xfy. Como antes, escrevemos f : X ---+ Y para denotar que f é uma função de X em Y e f(x) = y para denotar que o par (x, y) E X x Y é relacionado por f, i.e., satisfaz xfy. Observe que tal notação faz sentido, uma vez que a definição de função garante que se (x, Y1) e (x, y2 ) são pares ordenados em X x Y tais que xfy1 e xfy2, então y 1 = y 2 . Por outro lado, é imediato verificar que a definição formal acima coincide com a definição informal dada no início desta seção. O mais das vezes, trabalharemos com funções f : X ---+ Y tais que X, Y e R Em tais casos, geralmente indicaremos quem é o elemento f (x) E Y associado a um elemento genérico x E X por meio de uma fórmula em x que explicite uma regra que a função deva satisfazer. Por exemplo, podemos dizer: considere a função f : R ---+ R dada por f(x) = x 2 . Isto quer dizer que a função associa, a cada x E R, seu quadrado x 2 . Veja que os requisitos definidores de uma função estão satisfeitos, uma vez que, a cada x E R, temos associado um único outro real f(x), qual seja, x 2 . Assim é que, ainda em relação a esse exemplo, temos f( v'2) = (v'2) 2 = 2, f(3) = 32 = 9, etc. Quando X, Y e R e f: X---+ Y é uma função tal que o elemento f (x) E Y associado a x E X é dado por uma fórmula em x, denotamos por vezes tal correspondência escrevendo f:X --+ X

1----t

Y f(x)

Assim, a função do parágrafo anterior, que associa a cada x E R seu quadrado x 2 , poderia ser denotada da seguinte maneira:

f:R--+

R

1----t

x2

X

Vejamos mais um exemplo.

.

Antonio Caminha M. Neto

7

Exemplo 1. 7. Considere a função

f (x)

= {

·

f : (Q ---+ R dada por

J x 2 + 1, se x ::; O x + 1, se x > O

.

Certamente que temos uma função, pois as expressões que definem f(x) têm sentido em R e, apesar de devermos aplicar fórmulas diferentes, conforme o racional x satisfaça x ::; O ou x > O, cada racional x tem uma única imagem f (x) bem definida, posto que os casos x ::; O ex > O cobrem todos os racionais. Assim, por exemplo, f(-1) = y'(-1) 2 + 1 = v'2 (uma vez que -1::; O), mas f(2) = 2 + 1 = 3 (uma vez que 2 > O). Observe que poderíamos ter definido f(x) escrevendo

f (x)

= {

J x 2 + 1, x

se x ::; O .

+ 1, se x 2'.: O

Nesse caso, as condições x ::; O e x 2'.: O cobrem todos os racionais mas não são mais disjuntas: x = O satisfaz ambas; no entanto, as fórmulas que devemos aplicar a um caso ou outro dão um mesmo resultado quando x = O (uma vez que \1'0 2 + 1 = O+ 1), afastando assim qualquer possibilidade de inconsistência. Dizemos por vezes que uma função f como a do exemplo acima está definida por partes, em alusão ao fato de que há uma fórmula para calcular f(x) quando x E (-oo, O] e outra quando x E (O, +oo) (ou x E [O, +oo), conforme se queira). Ao lidarmos com uma função f : X ---+ Y, é frequentemente útil denominarmos os conjuntos X e Y, respectivamente, de domínio e contradomínio da função; nesse contexto, denotaremos X = Dom (!). Assim é que, para a função f do exemplo 1. 7, o domínio e o contradomínio são, respectivamente, (Q e R. O mais das vezes vamos trabalhar com funções f : X ---+ R tais que X e R Em tais casos, diremos que f é uma função real (em alusão

Tópicos de Matemática Elementar 3

8

ao fato de que f assume valores reais - i.e., de que seu contradomínio é R) de uma variável real (em alusão ao fato de que um elemento genérico x do domínio X de f - a variável da função - é um número real). Ainda em tais casos, quando os valores f (x) forem dados por uma fórmula, como por exemplo f (x) = ~ , salvo menção em conx(x-1)

trário, convencionamos tomar X como sendo igual ao maior domínio possível. De outro modo, tomamos X como igual ao maior subconjunto de R no qual as operações matemáticas que definem a expressão f (x) têm sentido. Diremos, então, que X é o domínio maximal de definição, ou simplesmente o domínio maximal de f. Exemplo 1.8. A título de ilustração, encontremos o domínio maximal X e R da função f: X---+ R dada por f(x) = ~ - Temos x(x-1)

X

{ x E

-

{x

E

R; Jx(x -1) R} --;::::=l== E

R; x(x - 1) > O}.

Como, para um produto ser positivo, ambos os fatores devem ter um mesmo sinal, devemos ter x > Oe x-1 > O, ou então x < Oe x-1 < O, i.e., x > 1 ou então x < O. Portanto,

X= (-oo, O) U (1, +oo). No contexto de funções reais de variável real, temos maneiras padrão de construir novas funções a partir de outras já conhecidas, utilizando as operações aritméticas do contradomínio R das mesmas. Mais precisamente, dados um conjunto não vazio X e R, um número real e e funções reais de uma variável real f, g : X ---+ R (de mesmo domínio!), definimos as funções

f

+ g,

f · g e e· f : X ---+ R

Antonio Caminha M. Neto

9

pondo

f(x) + g(x), f(x) · g(x), e· f(x),

(! + g)(x) (!. g)(x) (e· f)(x)

para todo x E X. Algumas observações são pertinentes. Em primeiro lugar, veja que os sinais de adição ria igualdade

(! + g)(x)

=

f(x)

+ g(x)

têm significados distintos: no primeiro membro temos a definição da função f +g, ao passo que, no segundo membro, f(x)+g(x) representa a adição usual dos números reais f(x) e g(x). Uma observação análoga é válida para os sinais de multiplicação utilizados nas definições das funções f ·g e e· f. Em segundo lugar, assim como com números reais, omitiremos em geral o sinal de multiplicação, escrevendo f g e cf em . vez de f · g e e · f, mas isto não deve causar confusão. É evidente que f + g, f g e cf são realmente funções de X em R As funções f + g e f g são denominadas, respectivamente, a soma e o produto das funções f e g. Por outro lado, o produto cf do número real e pela função f pode ser visto como caso particular do produto de duas funções: tomando g como a função constante e igual a e, temos f g = cf; ainda para tal g, denotaremos f + g simplesmente por f + e, i.e., (! + c)(x) = f(x) + e, para todo x E X. Exemplo 1.9. Sendo f e g as funções de R em R dadas por f(x) = x2: 1 e g(x) = -x + 3, temos

(! + g)(x)

=

f(x) + g(x)

=

X

+ (-x + 3) X+ 1 2

x + (x2 + 1) (-x + 3) x2

+1

-x 3 + 3x2 + 3 x2

+1

Tópicos de Matemática Elementar 3

10

x

(f g)(x) = f(x)g(x) = X

2

· (-x + 3) = +1

-x 2 + 3x X

2

+1

e

(

y13

f) (x) =

y13 y13 X xy3 3f (x) = 3 . x2 + 1 = x2 + 1 .

Deixamos ao leitor a verificação de que as operações de adição e multiplicação para funções acima definidas satisfazem propriedades análogas àquelas das operações correspondentes com números reais. Mais precisamente, para f, g, h : X -+ II e a, b, c E II, temos:

• Comutatividade: f

+ g = g + f;

• Associatividade: f

+ (g + h)

• Distributividade: f(g

+ h) =

=

fg = gf.

(! + g) + h; f(gh)

=

(f g)h.

f g + fh.

Por fim, note que a associatividade da adição e da multiplicação de funções permite definir, de modo inteiramente análogo, a soma ou o produto de um número finito qualquer de funções de X em R Sugerimos ao leitor consultar o problema 4 para uma extensão adicional da discussão acima. Voltemos ao estudo do contradomínio de uma função f: X-+ Y. É importante notar que o contradomínio Y em geral não coincide com o conjunto formado pelas imagens dos elementos de X. Ilustremos essa diferença utilizando novamente a função f do exemplo 1.7. Já observamos que o contradomínio da mesma é o conjunto II dos números reais. Por outro lado, o conjunto formado pelas imagens f (x) dos elementos do domínio Q de f certamente não contém números reais menores que 1. De fato, para um racional x qualquer, temos x 2 + 1 2: 1 e, daí,

f (x) = Vx 2 + 1 2: v'i = 1; por outro lado, para um racional x > O, temos f(x) = x maneira que {f(x); x E Q} e [1, +oo).

+1 >

1, de

Antonio Caminha M. Neto

11

Note, por fim, que o intervalo [1, +oo) é um subconjunto próprio do contradomínio II de f. Mais geralmente, dada uma função f: X-+ Y, o conjunto imagem, ou simplesmente a imagem da função f é o conjunto Im (!), cujos elementos são as imagens f(x) E Y dos elementos x E X:

.Im(f) = {f(x)

E

Y; x

E

X}.

Em particular, temos sempre Im (!) e Y, e a discussão acima mostra que pode ocorrer Im (!) # Y. No exemplo discutido acima, mostramos que a imagem da função era um subconjunto próprio do contradomínio da mesma. No entanto, não chegamos a explicitar precisamente tal conjunto-imagem. Em situações específicas essa tarefa pode ser consideravelmente difícil, como atesta o seguinte Exemplo 1.10. Explicitemos a imagem da função f : Q \ {O} -+ Q · dada por f (r) = } , se o racional r estiver escrito da forma r = onde a E Z, b E N e mdc (a, b) = 1 (para maiores detalhes sobre o mdc de dois inteiros, veja a introdução ao capítulo 1 do volume 1 ou a seção 1.2 do volume 5). Certamente a função f está definida de maneira não ambígua, uma vez que todo racional não nulo admite uma representação única como quociente de dois inteiros primos entre si, sendo o denominador natu= -./ e, daí, f ( = Por outro lado, como ral. Por exemplo, b E N, vemos imediatamente que

i,

-! ) ! .

-!

lm (!)

e { ~; b E N}

= { 1, ~,

1, ~, ... }.

Ademais, é imediato que todos os números do último conjunto acima pertencem à imagem de f, já que f (}) =} para todo b EN. Portanto,

1 1 1 } Im(f)= { 1,2'3'4'"' .

Tópicos de Matemática Elementar 3

12

Infelizmente não existe um algoritmo 1 que nos permita encontrar explicitamente a imagem de uma função qualquer dada. No entanto, ao longo dos capítulos subsequentes resolveremos o problema de encontrar o conjunto-imagem para vários tipos importantes de funções. Terminemos esta seção discutindo a igualdade de duas funções. Em relação à função f do exemplo 1.7, não faz sentido considerarmos f( v'2), uma vez que v'2 (/:. Q e f tem domínio Q. O que poderíamos fazer seria considerar, em vez de f, a função g : IR. -+ IR. dada por 9 ( x)

={

v'x 2 + 1,

se x ::; O . x + 1, se x > O

Apesar das fórmulas que definem f(x) e g(x) serem as mesmas, para f elas só podem ser aplicadas a x E Q, ao passo que para gelas podem ser aplicadas a todo x real; assim, não faz sentido pensarmos em f e g como funções iguais, apenas denotadas de duas maneiras distintas. Mais geralmente, temos a seguinte

Antonio Caminha M. Neto

13

Problemas - Seção 1.1

1. Considere a função f : IR. -+ IR. definida por f (x) = x 3 - 2x 2 + 5x. Prove que f(x) tem o mesmo sinal de x, para todo real x =1- O.

2. A função f : IR.-+ IR. é tal que f(l) = 2, f( v'2) = 4 e f(x + y) f(x)f(y), para todos x, y E R Calcule o valor de f(3 + v'2).

3. Seja f : IR.-+ IR. uma função tal que f(x + y) = f(x)f(y) para todos x, y reais. Se (ak)k::::: 1 é uma PA de razão r e f(a 1) =/:- O, prove que a sequência (f(ak))k::::: 1 é uma PG de razão f(r).

* Dadas funções

4.

reais de variável real f, g : X -t IR., estenda a discussão do texto, apresentando definições apropriadas para a diferença f - g e o quociente tg das funções f e g.

5.

*A

Definição 1.11. Duas funções f : X -+ Y e g : W -+ Z são iguais se X= W, Y = Z e f(x) = g(x), para todo x E X.

parte inteira de um real x é definida como o maior inteiro menor ou igual a x. Explicite a imagem da função parte inteira l J : IR. ----+ IR. (1.2) X

Se duas funções f: X-+ Y e g: W-+ Z forem iguais, escrevemos f = g. Frisamos que, de acordo com a definição acima, tal notação encerra mais significado que escrevermos f(x) = g(x): ela significa a igualdade dos domínios, X = W, e dos contradomínios, Y = Z, assim como a validade da igualdade f(x) = g(x), para todo x E X. Se funções f e g como acima não forem iguais, escreveremos f =/:- g e diremos que f e g são funções diferentes ou distintas.

=

1----t

lxj '

que associa a cada x E IR. sua parte inteira l x J. 6.

* A parte fracionária de um real x é definida por {x} = x- l x J, onde l x J denota a parte inteira de x (veja o problema anterior). Explicite a imagem da função parte fracionária { } : IR. ----+ X

1----t

IR.

{x} '

(1.3)

que associa a cada x E IR. sua parte inteira {x}. 1 Um

algoritmo é uma sequência finita e bem determinada de procedimentos executáveis que, rigorosamente seguidos, fornecem a solução de um certo problema.

7. (Torneio das Cidades.) Prove que o n-ésimo natural que não é quadrado perfeito é igual a ln+ y'n +

!J.

r :~

Tópicos de Matemática Elementar 3

14 8.

* Seja

f : (Q --+ (Q uma função tal que f(x + y) = f(x) para todos x, y E (Q. Prove os seguintes itens:

+ f(y)

(a) f(O) = O e f(-x) = - f(x) para todo x E (Q. (b) f(x - y) = f(x) - f(y) para todos x, y E (Q. (c) f(kx) = kf(x) para todo x E (Q e todo k E Z.

f (~) =~para todo n E Z \ {O}. (e) f (~) = 7:f(l) para todos m,n E Z, n # O.

(d)

9. Seja f: R--+ Ruma função tal que f(x+y) = f(x) + f(y), para todos x, y E R. Se (ak)k~l é uma PA de razão r, prove que a sequência (f(ak))k~I é uma PA de razão f(r). 10. (IMO.) Sejam f, g: R--+ R funções tais que, para todos x, y E R, tenhamos f(x + y) + f(x - y) = 2f(x)g(y).

Se f não é identicamente nula e If (x) 1 prove que lg(x)I :::; 1, para todo x E R

:::;

1 para todo x E R,

~: L

Antonio Caminha M. Neto

15

Desta forma, se os valores f (x) forem dados por uma expressão que dependa de x E X, poderemos encarar o problema de encontrar a imagem de f como aquele de encontrar os y E R para os quais a equação f(x) = y tenha pelo menos uma solução x E X. Vejamos alguns exemplos.

Exemplo 1.12. Uma função afim é uma função f: R--+ R tal que f(x) = ax + b para todo x real, onde a e b são números reais dados, com a# O. Uma função linear é uma função afim f como acima, tal que b = O. De acordo com a discussão acima, a imagem de uma função afim f como acima pode ser encontrada procurando o conjunto dos y E R tais que a equação ax + b = y tenha alguma solução x E JR. Mas, como tal equação sempre admite a solução x = ~, co11d~::.,.;:.;;os que todo y E R pertence à imagem de f, de sorte que Im (!) = R Exemplo 1.13. A função de proporcionalidade inversa é a função f: R \{O}--+ R \ {O} dada por f(x) = para todo x E R \ {O}. Para encontrar sua imagem, é suficiente encontrar os y E R para os quais exista x # O real (i.e., x pertencente ao domínio de !) , tal que f(x) = y, i.e., tal que = y. Se y = O, tal equação claramente não admite solução; por outro lado, se y # O, a mesma equação admite a de sorte que Im (!) = R \ {O}. solução x =

i,

i

1.2

Monotonicidade, extremos e imagem

Comecemos esta seção concentrando-nos no problema de encontrar a imagem de uma função dada. Para tanto, recorde que a imagem de uma função f: X--+ Y é o conjunto lm (!) = {f(x) E Y; x E X}. Se a função f é real de uma variável real, i.e., se f : X --+ R, com X e R, uma maneira particularmente útil de declararmos sua imagem é escrevermos Im (!) = {y E Y; y = f(x) para algum x E X}.

!,

A continuação, precisamos da definição a seguir.

Definição 1.14. Uma função quadrática ou de segundo grau é uma função f : R --+ R tal que f (x) = ax 2 + bx + e, para todo x real, onde a, b e e são números reais dados, com a# O. O discriminante~ da função fé o discriminante do trinômio de segundo grau ax 2 +bx+c, i.e., ~ = b2 - 4ac. O problema da determinação da imagem de uma função quadrática é suficientemente importante para ser colecionado na seguinte

Tópicos de Matemática Elementar 3

16

Proposição 1.15. Em relação à função quadrática f (x) temos que:

(a) Se a> O, então Im(f) =

=

ax 2 +bx+c,

Antonio Caminha M. Neto

de maneira que Im (!)

[-!,+oo).

(b) Se a< O, então Im (!) = ( -oo, -!]

.

ax 2

f (x)

= - 4a {::}

x

= -

b 2a ·

Prova. Seguindo a ideia geral esboçada anteriormente, basta encontrar os y E IR para os quais a equação ax 2 + bx +e= y, i.e., a equação de segundo grau ax 2 + bx + (e - y) = O, tenha solução. Também, assim como antes, uma condição necessária e suficiente para a existência de raízes 2 é que o discriminante dessa última equação seja não negativo, i.e., que b2 - 4a(c - y) 2:: O. Mas, como b2 - 4ac = ~' os y que procuramos são exatamente as soluções da inequação de primeiro grau ~ + 4ay 2:: O.

Consideramos agora os casos a > O e a < O separadamente. Se a> O, então

e segue daí

= {

y E IR; y S - ~}

-

2a

-

2a

(l.4)

·

Portanto, a equação f(x) = y admite uma solução única se, e só se, ~ + 4ay = O, ou, o que é o mesmo, se, e só se, y = sendo esse o • caso, temos, a partir de (1.4), que x = - 2ba·

!;

Para o que segue, convencionamos dizer que a função quadrática f ( x) = ax 2 + bx + e tem sinal constante quando f (x) 2:: O para todo x E IR ou f (x) O para todo x E R

s

Corolário 1.16. A função quadrática f(x) = ax 2 + bx + e tem sinal constante se, e só se, ~ O. Neste caso, temos af(x) 2:: O, para todo x E R De outro modo:

s

s O e a> O, então f(x) 2:: O para todo x E R Se~ s O e a< O, então f(x) s O para todo x E IR.

(a) Se~ (b)

Prova. Analisemos o caso a > O, sendo o outro caso análogo. Se ~ S O, temos da proposição anterior que ~

4

ª 2:: O,

Vx

E

R

Reciprocamente, suponha que a > Oe f tem sinal constante. Segue novamente da proposição anterior que a imagem de f contém números positivos, e a constância de sinal de f garante que deve ser f(x) 2:: O, para todo x E R Em particular, devemos ter

2 Aqui e em vários outros pontos do texto assumimos que o leitor possua certa familiaridade com a noção de raiz quadrada, sem nos preocuparmos em estabelecer sua existência rigorosamente. A esse respeito, veja, por exemplo, o capítulo 3 de

[34].

-oo, - ~]

x - -b±Jb2 -4a(c-y) _ -b±J~+4ay

f (x) 2:: se a< O, então

= (

Para o que falta, veja que, para y E Im (!), as soluções da equação + bx + e = y ({::} ax 2 + bx + (e - y) = O) são

Ademais, em qualquer um dos casos acima, temos ~

17

Logo,~ S O.

~ =f

4a

(-!!__) 2:: O. 2a



Tópicos de Matemática Elementar 3

18

Observação 1.17. Uma pequena modificação do argumento apresentado no corolário acima permite concluir que i.

Se~< O e a> O, então f(x) > O para todo x E IR.

ii. Se~< O e a< O, então f(x)

< O para todo x

E IR.

O corolário acima pode ser utilizado para dar uma prova mais direta da desigualdade de Cauchy (teorema 7.14 do volume 1), conforme ensina o seguinte

Exemplo 1.18. Dados n > 1 inteiro e números reais a1, a2, ... , an não todos nulos e b1 , b2, ... , bn também não todos nulos, considere a função quadrática

J(x)

(a1x - b1) 2 + (a2x - b2) 2 + · · · + (anx - bn) 2

Ax 2 -2Bx+C, onde A = a~ + a~ + · · · + a;,, B = a1b1 + a2b2 + · · · + anbn, C = b~ + b~ + ... + b;,. Uma vez que f(x) é uma soma de quadrados, devemos ter J(x) 2 O para todo x E IR. Por outro lado, como A > O, o corolário 1.16 garante que ~ = 4( B 2 - AC) :::; O. Portanto, B 2 :::; AC, ou, o que é o mesmo, IBI :::; v1A.Jc. Substituindo os valores de A, B e Cem tal desigualdade, obtemos a desigualdade de Cauchy. De acordo com a dedução acima, a igualdade na desigualdade de Cauchy equivale à igualdade ~ = O para a função, que por sua vez equivale à existência de um único a E IR tal que f(a) = O. Mas, como f(a) é uma soma de quadrados, temos J(a) = O se, e só se, cada um de tais quadrados for zero, i.e., se, e só se,

a1n - b1 = a2a - b2 = · · · = anil - bn = O. Por fim, como ao menos um dos bi é não nulo, temos a =/:- O e, escrevendo À= l, obtemos a

f º

Antonio Caminha M. Neto

19

f,

como condição necessária e suficiente para a igualdade. A fim de prosseguirmos nosso estudo de funções, precisamos agora da seguinte

Definição 1.19. Seja I dita:

e IR um intervalo. Uma função f : I--+ IR é

(a) crescente se, para todos x 1 < x2 em I, tivermos J(xi) < f(x2). (b) decrescente se, para todos x1 < x 2 em I, tivermos f(x1) > f(x2). (c) não decrescente se, para todos x1 < x 2 em I, tivermos J(x1):::;

f (x2). (d) não crescente se, para todos x1 < x 2 em I, tivermos f(x1) 2 f(x2). Ademais, em um qualquer dos casos acima, dizemos que a função monótona em ! 3 .

f é

A respeito da definição acima, um problema interessante é o de encontrar os intervalos de monotonicidade de uma função, i.e., dada uma função f : I --+ IR, onde I e IR é um intervalo, pede-se investigar em que intervalos f é crescente (resp. decrescente). Vejamos alguns exemplos elementares, postergando uma análise mais geral para a seção 6.3.

Exemplo 1.20. A função afim f : IR --+ IR, dada para x E IR por f(x) = ax + b é crescente se a> O e decrescente se a< O. 3 Nas

notações desta definição, vale observar que, para alguns autores, uma função f satisfazendo a condição do item (a) (resp. (b), (e), (d)) é dita estritamente crescentes (resp. estritamente decrescentes, crescentes, decrescentes).

rm 1,,1

'i[' 1,

'I

,,!1

Tópicos de Matemática Elementar 3

20

Antonio Caminha M. Neto

21

1

Verifiquemos tal afirmação quando a > O, sendo a análise do caso a < O totalmente análoga. Sendo x 1 < x 2 números reais quaisquer, segue de a > O que

e

f é crescente.

f(b) - J(a)

b2 ª2 ----b+2 a+2 (a+ 2!(b + 2) [b2(a + 2) - a2(b + 2)],

e, uma vez que (a+ 2)(b + 2) > O, basta mostrarmos que b2(a + 2) a2 (b + 2) > O. Para tanto, veja que

(a) Se a > O, então

(b - a)[ab + 2(b + a)]; como O ~ a < b, segue que ambos os fatores do último produto acima são positivos e, daí, b2 (a + 2) - a2 (b + 2) > O.

Exemplo 1.22. A função f : R -+ R dada por f(x) = x 3 + 2x é crescente. De fato, para números reais quaisquer a < b, temos

a3 + 2b - 2a (b - a)(b2 + ba + a2) + 2(b - a) (b - a)(b2 + ab + a2 + 2).

b3

-

Como b - a > O, basta mostrarmos que a2 + ab + b2 + 2 > O; uma possibilidade é usar a desigualdade entre as médias para dois numeras:

a2 + b2 + ab + 2 ~ 2labl + ab + 2 ~ labl + 2 > O,

O,

2:]

=

ax 2+bx+c.

e crescente em

[-;ª, +oo). (b) Se a < O, então

f é crescente em ( -oo, - ;ª] e decrescente em

[- ;a' +oo) . Prova. Façamos a prova do item (a), sendo a prova do item (b) análoga. Para x 2 > x 1 ~ temos

;ª,

a(x~ - Xi)+ b(x2 - x1) a(x2 -

ab(b - a)+ 2(b - a)(b + a)

J(b) - f(a)

f é decrescente em ( -oo, -

b2a - a2b + 2(b2 - a 2)

b2(a + 2) - a2(b + 2)

~

A proposição a seguir resolve, para funções quadráticas, o problema da determinação dos intervalos de monotonicidade.

Proposição 1.23. Sejam a, b, e E R, com a=/:- O, e J(x)

Exemplo 1.21. A função f : [O, +oo) -+ R dada por f(x) x~ 2 é crescente em R. Para verificar tal afirmação, tome números reais O ~ a < b. Então

lod + a

onde na penúltima passagem utilizamos o fato de que para todo a E R.

X1) ( X2

+ X1 + ~) > o,

-;ª

uma vez que X2 > X1 ~ implica X2 - X1 > O e X2 + X1 + ~ > O. O caso x 1 < x 2 ~ - 2: é análogo e também será deixado ao leitor. • A próxima definição é, de certa maneira, complementar à definição 1.19.

Definição 1.24. Sejam I e Rum intervalo e f: I-+ Ruma função dada. Dizemos que y0 E R é o valor mínimo de f em I se as duas condições a seguir forem satisfeitas:

(a) Im (!) e [Yo, +oo). (b) Yo E Im (!). Nesse caso, os reais x 0 E I tais que f(x 0 ) = y0 são denominados os pontos de mínimo da função f.

Tópicos de Matemática Elementar 3

Antonio Caminha M. Neto

Analogamente, definimos o que se entende por valor máximo e ponto de máximo de uma função f : I --+ li (I e li intervalo). Genericamente, os pontos de máximo (resp. mínimo) de uma função são denominados seus pontos extremos; da mesma forma, os valores que a função assume em tais pontos são seus valores extremos. Veremos na seção 6.3 como procurar pontos extremos para uma classe importante de funções, ditas deriváveis. Por ora, contentamonos com alguns exemplos elementares, o primeiro dos quais sendo uma consequência imediata da proposição 1.15.

ao triângulo OQ R que x 2 + y 2

22

Proposição 1.25. Em relação à função quadrática J(x) = ax 2+bx+c, se a > O (resp. a < O), então - 2: é o único ponto de mínimo (resp. máximo) de f. Ademais, o valor mínimo (máximo) de f é - _t. A proposição acima tem várias aplicações interessantes, duas das quais colecionadas abaixo à guisa de ilustração. Exemplo 1.26. Temos um semicírculo de diâmetro AB, centro O e raio lcm (figura 1.4). O retângulo PQRS tem o lado PQ situado sobre o diâmetro do semicírculo e os vértices R e S situados sobre o mesmo. Calcule o maior valor possível para sua área.

23 =

1 e, daí,

2xy = 2xvl - x 2 = 2Jx 2(1- x 2) = 2vx 2 - x 4 . Fazendo a substituição z = x 2 , segue da expressão acima para a área que basta maximizar a função de segundo grau f (z) = z - z 2 , com a condição de que O < z < 1 (uma vez que x < OR = 1). Pela proposição 1.25, tal função admite z = ! como seu único ponto de máximo; como! E (O, 1), segue que o valor máximo desejado é f =

0)

!· Portanto, o valor máximo para a área é 2/'f =

1.



Exemplo 1.27. Dado um triângulo ABC no plano, mostre que seu baricentro G é o único ponto P do plano de ABC para o qual a soma -2 -2 -2 AP + BP + CP é a menor possível. Prova. Escolha um sistema Cartesiano de coordenadas, em relação ao qual tenhamos A(x 1 , y 1 ), B(x 2, y 2 ) e C(x 3, y3). Se P(x, y), então a fórmula para a distância entre dois pontos do plano Cartesiano fornece -2 -2 -2 AP + BP + CP

=

f(x)

+ g(y),

onde 3

J(x) = l ) x - xi) 2 = 3x 2 - 2(x1

+ X2 + x3)x + (x~ + x~ + x~)

i=l

A

p

Q

X

Q

B

Figura 1.4: maximizando a área de PQRS.

Solução. Sendo OQ = x e QR = y, temos que a área de PQRS é igual a 2xy. Por outro lado, segue do teorema de Pitágoras aplicado

e, analogamente, g(y) = 3y2 - 2(y1 + Y2 + y3)y + (y~ + Y~ + Yi). Como x e y são variáveis independentes, a fim de minimizarmos a soma AP2 + B P 2 + C P 2 é suficiente minimizarmos as funções quadráticas f e g. Para tanto, recorrendo à proposição 1.25 concluímos que f e g atingem seus valores mínimos somente nos pontos x = Hx1 +x2+x3) e y = }(Y1 +y2+y3), respectivamente, os quais são (cf. equação (6.3) do volume 2) precisamente as coordenadas do baricentro • G do triângulo ABC.

Tópicos de Matemática Elementar 3

24

í I

r~

' C amm "hMN Antomo a . eto

25

I Outra estratégia elementar, por vezes útil, para abordar o problema de encontrar os valores máximo e/ ou mínimo de uma função dada é a utilização de desigualdades. A seguir, vemos dois exemplos nesse sentido.

x::(

Solução. Novamente pela desigualdade entre as médias, temos

x 2 + 16

16

2

16

16

x+3+3+3

> 4 4/x2. 16. 16. 16 =

V

3

3

3

~vx m '

Exemplo 1.28. Seja f : [O, +oo) ~ IR a função dada por f(x) = Qual o valor mínimo que f assume? A função f assume um valor máximo?

e daí

Solução. Inicialmente, note que

A igualdade ocorre se, e só se, x 2 = \6 , ou seja, se, e só se, x =

Js

(uma vez que x ~ O). Assim, ~ é o valor máximo de f.



f(x) =

x2 + 1

x+l X -

x2

-

1+2

f(x)

=

vx

x 2 + 16 :::;

m

32·

x+l

2 2 1 + - - = (x + 1) + - - - 2.

x+l

x+l

Problemas - Seção 1.2

Portanto, aplicando a desigualdade entre as médias para dois números, obtemos

(x + 1) + - 2x+l

1. Para reais dados a e b, com a

O, considere a função afim f : IR~ IR, definida por f(x) = ax + b. Se (ak)k:::: 1 é uma PA de razão r, prove que a sequência (bk)k>I, dada para k ~ 1 inteiro por bk = f(ak), é uma PA de razão ar.

2 - = 2V2, ~ 2J(x + 1) · -x+l

x!i,

ocorrendo a igualdade se, e só se, x + 1 = i.e., se, e só se, x 2 + 2x - 1 = O. Mas, como x ~ O, concluímos que haverá igualdade na desigualdade acima se, e só se, x = v12 - 1. Assim, para x ~ O temos

2 f(x) = (x + 1) + x + 1 - 2 ~ 2V2 - 2, de sorte que 2v12 - 2 é o valor mínimo de f, o qual é atingido se, e só se, x = v12 - 1. Para o que falta observe que, para n E N, temos f (n) = n - 1 + ~ n - 1 e, daí, f não assume valor máximo. •

n!i

Exemplo 1.29. Encontre o máximo da função por f(x) = x2~ 6 .

f : [O, +oo)

~

IR dada

#

2. Ache a imagem da função 3.

f:

IR~ IR dada por f(x)

=

x2~ 1 .

* Ache a imagem da função f: IR*~ IR, dada por f(x) = x + ~-

4. Sejam I e IR um intervalo, a E I e f : I ~ IR uma função dada. Se f é crescente (resp. decrescente) em (-oo, a] n I e decrescente (resp. crescente) em [a, +oo) n J, então a é o único ponto de máximo (resp. de mínimo) para f em I.

5.

* Sejam X

e IR um conjunto não vazio, f : X

~

IR uma função dada e e E IR também dado. Relacione as imagens das funções f e f + e. Mais precisamente, se Y = Im (!), prove que Im (! + e) = Y + e, onde Y + e denota o conjunto

Y +e= {y + e; y E Y}.

Tópicos de Matemática Elementar 3

26

6. Motivados pela forma canônica do trinômio de segundo grau ax 2 + bx + c, diremos doravante que

f (x)

=

a { (x + ~) 2a

2 -

~} 4a2

7. Sejam f(x) = ax 2 +bx+c uma função quadrática tal que~> O, e x 1 < x 2 as raízes de J(x) = O. Prove os seguintes itens: (a) Se a> O, então f(x) Oe Xo E (x1, x2), onde x 1 < x 2 são as raízes da equação f(x) = O. 9. Dentre todos os retângulos de mesmo perímetro, prove que o de maior área é o quadrado.

10. A seção reta de um túnel tem o formato de um semicírculo de raio 5m, e o túnel está dividido em duas faixas de trânsito, de sentidos contrários, separadas por um canteiro muito estreito. Os caminhões de uma companhia de transportes têm de atravessar o túnel para levar mercadorias de uma cidade a outra. Se o comprimento máximo permitido de um caminhão é 18m, quais devem ser sua largura e altura a fim de que a companhia transporte o máximo possível de carga em cada caminhão? 11. Calcule o valor máximo da função 5x-1 x 2 +1'

27

12. Sejam a 1 < a 2 < · · · < an reais dados e dada por

f : lR --+ lR a função

(1.5)

é a forma canônica da função quadrática f(x) = ax 2 + bx + c. Utilize tal forma canônica para dar uma outra demonstração da proposição 1.15.

(b) Se a< O, então f(x) m::::} f(n) > f(m). Se f possui infinitos pontos de estrangulamento, mostre que ela é crescente.

1.3

Composição de funções

Antonio Caminha M. Neto

29

Grosso modo, a definição acima significa que, para encontrarmos a imagem de x E X por g o f, basta encontrarmos a imagem de f (x) E Y por g. É fácil verificar que g o f, como definida acima, é de fato uma função. Observe também que, para formarmos a composta de f e g, devemos ter o domínio de g igual ao contradomínio de f. Vejamos alguns exemplos. Exemplo 1.31. Se f : X --+ Y é uma função arbitrária e ldx : X --+ X e ldy : Y--+ Y são, respectivamente, as funções identidade de X e Y, então f o ldx = f e ldy o f = f. Verifiquemos a igualdade f o ldx = f, sendo a outra totalmente análoga. Para tanto, basta notarmos que f o ldx é uma função de X em Y tal que, para todo x E X,

(! o ldx)(x)

=

f(Idx(x))

Definição 1.30. Dadas as funções f: X--+ Y e g: Y--+ Z, a função composta de f e g (nessa ordem) é a função g o f: X--+ Z definida, para cada x E X, por

(g o f)(x)

=

g(f (x)).

f(x).

Exemplo 1.32. Considere as funções f,g: ~--+~dadas por f(x) = x 2 e g(x) = x 2~ 1 . Temos g o f e f o g funções de~ em~' com 1

Dadas as funções f : X --+ Y e g : Y --+ Z temos, em última análise, regras bem definidas para, partindo de x E X via f, obter y = f(x) E Y e, via g, obter z = g(y) E Z. Parece, então, razoável que possamos formar u,ma função que nos permita sair de X diretamente para Z. Este é de fato o caso, e a função resultante é denominada a função composta de f e g, de acordo com a seguinte

=

(g

O

f)(x) = g(f(x)) = (f(x)) 2 + 1

1

(x 2 ) 2 + 1

1

x4 + 1

e

(J o g)(x) - f(g(x)) - (g(x))' -

(x, ~ J

1 x4

+ 2x2 + 1 ·

O exemplo acima mostra algo interessante: podemos ter g o f =/=! og. Bem entendido, pode mesmo acontecer que possamos formar gof mas não f o g (ou vice-versa); basta termos, por exemplo, f: X--+ Y e g : Y --+ Z, com X =/=- Z. Contudo, mesmo que possamos formar ambas as funções, o exemplo mostra que, ainda assim, pode ocorrer que g o f =/=- f o g.

Tópicos de Matemática Elementar 3

30

f, g : (O, +oo) --+ (O, +oo) funções tais que

Exemplo 1.33. Sejam

x2 + 1

f(x) = 3x 2

X+

2

Encontre a expressão da função g. Solução. Segue da definição de composta que X+

3

de mo do que

2

= (f

39 (x) 2

-

-

Exemplo 1.35. Seja f : R \ {-1, 1} --+ R \ {-1, 1} a função dada por f(x) = Para cada n EN, encontre a expressão que define a função

)( ) = f( ( )) = g(x) 2 + 1 og X gX 3g(x)2 '

g(x)2+1 _ x+2 3-

31

A proposição acima é muito importante, na medida em que nos assegura que, se tivermos funções f, g eh e pudermos compô-las (nessa ordem), podemos denotar a função composta por h o g o f simplesmente, não nos preocupando com qual composição efetuar primeiro. É também claro que vale uma observação análoga para mais de três funções.

= - 3 -.

e (! o g)(x)

Antonio Caminha M. Neto

i~:-

· d ou, ain a,

j(n)

=

f

O

f

Solução. Veja primeiro que Olhando essa expressão como uma equação do primeiro graú em g(x)2, obtemos g(x) 2 = e, daí, g(x) = para cada x > O. Mas, como g deve ter imagem não negativa, deve ser g(x) = para todo x > O. •

±k,

Proposição 1.34. Dadas funções f: X--+ Y, g: Y--+ Z eh: Z--+ W, temos h o (g o !) = ( h o g) o f. Prova. Veja primeiro que ambas h o (g o!) e (h o g) o f são funções de X em W. Portanto, para serem iguais, é suficiente que associem, a cada x E A, um mesmo elemento de W. Para ver isto, basta notar que

h((g o f)(x)) (h o g)(f(x))

= =

h((g(f(x))) ((h o g) o f)(x).



j(n) :

R \ { -1, 1} --+ R \ {-1, 1}. Agora

J< 2)(x) = (f o f)(x) = J(f(x)) =

k,

Apesar de não ser comutativa, a operação de composição de funções é associativa, conforme ensina a seguinte

(h o (g o f))(x)

f.

n

3g(x) 2 + 3 = 3(x + 2)g(x) 2 •

x!i

O ••• O

1 - f(x) 1 + f(x)

=

l - ~ 1+

i~:

=x

'

isto é, j< 2) = ldx, a função identidade de X= R \ {-1, l}. Segue daí que j O, as proposições 1.15 e 1.23 garantem que a função quadrática l(x) = ax 2 + bx + e, vista como função

l :

[-..!!._ +oo) 2a'

----+ [-

~ +oo) '

4a'

é uma bijeção. Calcule a expressão de sua invera. Solução. De acordo com a discussão que precedeu o exemplo 1.42, a fim de obter a expressão da inversa 1- 1 : [ - t,, +oo) ----+ [- 2bª, +oo) de f, devemos fixar y E [- t,, +oo) e resolver, para x E [- 2bª, +oo), a equação l (x) = y, isto é, a equação ax 2 + bx + e - y = O. Ao fazê-lo, a condição x ~ - 2bª garante (lembre-se de que a > O) que

xonde~= b2

-

-b + Jb 2

-

2a

4a(c - y)

-b + J ~ + 4ac ------2a '

4ac é o discriminante de

f.

Logo,

• Exemplo 1.43. Uma discussão análoga à do exemplo acima garante que a inversa da função identidade ldx do conjunto X =/=- 0 é ela



Tópicos de Matemática Elementar 3

42

Exemplo 1.45. Como caso particular do exemplo anterior4, a função I : [O, +oo) --+ [O, +oo) dada por I (x) = x 2 é uma bijeção, tendo como inversa a função raiz quadrada

1-

1 :

[O, +oo)

---+ [O, +oo)

VX

f-t

X

Terminemos esta seção explicitando uma relação útil entre composição e inversão de funções. Proposição 1.46. Se I : X --+ Y e 9 : Y --+ Z são funções bijetoras, então 9 o I : X --+ Z é bijetora e ( 90 1) -1

=

1-1

(f- 1 o 9- 1 ) o (9 o!)= Idx e (9 o!) o (f- 1 o 9- 1 ) = Idz. Mas tal verificação é imediata e será deixada a cargo do leitor.



* Seja I : X

4 Aqui

rn,

2. Seja I: [!, +oo)--+ +oo) a função definida por l(x) = x 2 x + 1. Mostre que I é uma bijeção e obtenha a expressão para sua inversa.

:.:::t

3. Seja I : IR\ {2} --+ IR\ {3} a função definida por l(x) = 3 Mostre que I é uma bijeção e obtenha a expressão para sua inversa.

* Sejam n

4.

um inteiro positivo e I : IR --+ IR a função dada por l(x) = xn. Prove que I é uma bijeção se, e só se, n for ímpar. Nesse caso, calcule a expressão da inversa de I.

5.

X C IR um intervalo e I : X --+ X uma bijeção. Se for crescente (resp. decrescente), prove que 1- 1 também será crescente (resp. decrescente).

* Sejam

I

6. Sejam I : IR --+ IR uma bijeção e 9 : IR x IR --+ IR x IR a função definida por 9( x, y) = (x 3 , x - I (y)). Prove que 9 é bijetora e encontre a expressão de sua inversa em termos da função inversa 1-1 de 1-

I, 9 E G, então I o 9 E G. Se I E G, então 1- 1 E G.

(a) Se

--+ Y uma função dada.

(a) Se 9: Y--+ X é uma função tal que 9 o I = Idx e ldy, prove que I é uma bijeção.

Y --+ X tal

7. (IMO.) Seja G um conjunto (não vazio) de funções afins, possuindo as seguintes propriedades:

Problemas - Seção 1.4

1.

(b) Prove que existe no máximo uma função 9 que 9 o I = Idx e I o 9 = ldy.

09 -1 .

Prova. Já sabemos, pelo item (e) da proposição 1.39, que 9 ó I é bijetora. Por outro lado, como (9 o f)- 1 e 1- 1 o 9- 1 são ambas funções de Z em X, a fim de verificar que (9 o f)- 1 = 1- 1 o 9- 1 é suficiente, pela unicidade da inversa (cf. problema 1), mostrar que

43

Antonio Caminha M. Neto

I

o9 =

e no exemplo anterior estamos nos apoiando no conhecimento anterior do leitor. A rigor, a discussão apresentada só pode ser rigorosamente justificada com a introdução do conceito de função contínua, o que faremos no capítulo 4. A esse respeito, veja também o exemplo 4.22.

(b)

(c) Para toda I E G, existe x f E IR tal que I (x f) = x f. Prove que existe um real x 0 tal que l(x 0 ) = x 0 para toda I E G. 8. (França.) Seja I : N --+ N uma bijeção. Prove que existem naturais a< b < e tais que l(a) + l(c) = 2l(b).

Tópicos de Matemática Elementar 3

44

1.5

Funções definidas implicitamente

Uma função pode ser definida implicitamente por um conjunto de propriedades. Por exemplo, sendo g(x) = x + 1 e h(x) = x - 1, a função f: ~--+~dada por f(x) = x 2 é tal que

f(g(x))

=

g(x) 2 e f(h(x))

=

h(x)2,

ou seja, ela é tal que

f(x+l) = (x+1) 2 =x 2 +2x+l e J(x+l) = (x-1) 2 =x 2 -2x+l. Daí, temos que a função acima satisfaz, para todo x E~' a relação

f (X + 1) - f (X

-

1) = 4x.

Podemos tentar reverter os passos acima, perguntando agora quais são as funções f : ~ --+ ~ tais que

f (x + 1) - f (x - 1) = 4x, V x

E R

(1. 7)

É claro que a função f (x) = x 2 não é a única, pois, como é fácil verificar, para qualquer constante real e a função fc(x) = x 2 + e também satisfaz ( 1. 7). Corno uma função f : ~ --+ ~ satisfazendo (1. 7) não está dada por seus valores, e sim por uma relação que deve satisfazer, dizemos que a função está definida implicitamente. Note que, a partir de ( 1. 7), podemos descobrir outras relações que a função satisfaz. Por exemplo, se g: ~--+~é dada por g(x) = x 2 , temos

f(g(x)

+ 1) - f(g(x) -

1) = 4g(x)

ou, ainda,

f(x 2 + 1) - f(x 2 - 1) = 4x 2 , V x

E~-

(1.8)

Assim, qualquer função que satisfizer (1.7) também satisfará (1.8). Entretanto, a relação (1.8) pode não ser muito útil para ajudar a

Antonio Caminha M. Neto

45

determinar as funções f que satisfazem (1.7). Só a experiência dirá que relações obtidas a partir de uma relação inicialmente dada serão úteis nesse sentido. Em geral, um problema interessante é o de encontrar todas as funções definidas implicitamente por um certo conjunto de relações dadas. Uma vez que não há uma teoria geral a esse respeito, no que segue, veremos alguns exemplos que ilustram um certo número de técnicas úteis no trato de funções definidas implicitamente. Exemplo 1.47 (Canadá). Ache todas as funções f: N--+ N, crescentes e tais que f(2) = 2 e f(mn) = f(m)f(n), para todos m, n EN. Solução. De 1 ~ J(l) < J(2) = 2 obtemos J(l) = 1. Agora J(4) = f(2)J(2) = 4 e J(8) = J(4)J(2) = 8. Suponha pois, por hipótese de indução, que J(2k) = 2k para um certo natural k. Então

e segue que J(2n) = 2n para todo inteiro não negativo n. Portanto, fixado n natural, segue de f ser crescente que

Mas, uma vez que J(2n + 1), J(2n + 2), ... , J(2n+1 - 1) são naturais, a única possibilidade é termos

f(2n + 1) = 2n + 1, J(2n + 2) = 2n + 2, ... , J(2n+l - 1) = 2n+1 - 1. Finalmente, como esse raciocínio é válido para todo n natural, segue • que f (m) = m para todo m natural. Exemplo 1.48 (OIM). Se D = ~ - {-1, O, 1}, encontre todas as funções f : D --+ ~ tais que, para todo x E D, tenhamos

f(x)2 f ( 1 - x) l+x

=

64x.

Tópicos de Matemática Elementar 3

46

Solução. Note antes de tudo que, como X=/=- o, temos f(x) 2 f e~;) =/=Ü para todo x E D. Em particular, J(x) =/=- O para todo x E D. Seja agora g(x) = para x E D. A definição de D garante facilmente que g(D) e D, de modo que podemos compor f com g. Assim, para todo x E D, temos

i~;

(1.9) Substituindo a expressão de g na relação acima, chegamos a

f

(

1-x 1+ X

ou, ainda,

)

f (

-1-x) = (1+ 1+ ~ 1

64

(1-x) + f (X) 2

f

1

X

=

64

1 -x )

(1-x) + 1

X

X

'

para todo x E D. Elevando ao quadrado ambos os membros da relação do enunciado e dividindo o resultado pela relação acima, obtemos

J(x)3 = 64x2

(1-x) +x 1

e, daí, J(x) = 4 3 x 2 (i~;). Até este ponto, mostramos apenas que, se f existir, deve ser dada por essa expressão. Temos, pois, de verificar que f, assim definida, realmente satisfaz a relação do enunciado para todo x E D. Mas tal verificação é imediata e será deixada a cargo do leitor. • Ainda em relação ao exemplo anterior, com um pouco mais de prática poderíamos prescindir de definir a função g para em seguida compô-la com f a fim de obter (1.9). Ao invés disso, poderíamos apenas ter dito Substituindo x por

i~;

na relação do enunciado, obtemos ... ,

Antonio Caminha M. Neto

47

tendo em mente que essa substituição é meramente uma composição de funções. Doravante, sempre que não houver perigo de confusão, adotaremos essa simplificação de linguagem, a qual já aparece no exemplo a seguir. Ao lê-lo, tente identificar as composições que foram mascaradas por substituições.

Exemplo 1.49 (Polônia). Encontre todas as funções que, para todos x, y E R, tenhamos

(x - y)f(x + y) - (x + y)f(x - y) = 4xy(x 2

f : R--+ R -

tais

y2 ).

Solução. Como essa relação deve ser válida para todos os x, y E R, e y = com a, b E R. ela deve ser válida se fizermos x = Substituindo esses valores de x e y na relação do enunciado chegamos à relação bf(a) - af(b) = (a 2 - b2 )ab,

ª!b

ª;b,

a qual deve ser satisfeita para todos a, b E R Em particular, quando ab =/=- O, dividindo ambos os membros dessa relação por ab segue que devemos ter f(a) _ f(b) = ª2 _ b2 a b ' para todos a, b E R \ {O}. Portanto, sendo g : R \ {O} --+ R a função dada por g(x) = 1C:) - x 2 , a relação acima diz que g(a) = g(b), para todos a, b E R \ {O}. Em outras palavras, g deve ser constante, isto é, deve existir um real k tal que g(x) = k para todo x E R \ {O}. Mas isso é o mesmo que ser f(x) = x 3 + kx, para todo x E R \ {O}. Por outro lado, fazendo x = y = 1 na relação do enunciado, obtemos f(O) = O para qualquer função que satisfaça aquelas condições. Como 03 + k · O = O, concluímos que qualquer função que satisfaça as condições do enunciado deve ser da forma f (x) = x 3 + kx, para todo xER Novamente temos de verificar que toda função desse tipo satisfaz as condições do enunciado, o que é imediato e será, uma vez mais, deixado a cargo do leitor. •

Antonio Caminha M. Neto

Tópicos de Matemática Elementar 3

48

49

(b) f(xy) = f(x)f(y).

Para o próximo exemplo precisamos da seguinte

Solução. Seja f uma função satisfazendo as condições do enunciado. Fazendo x = y = O em (a), obtemos

Definição 1.50. Se X é um conjunto não vazio e f: X--+ X é uma função dada, um elemento x 0 E X é dito um ponto fixo de f se f(xo) = Xo.

f(O) = f(O + O) = f(O) + f(O) = 2f(O),

Se I e lR é um intervalo, uma função decrescente f : I--+ I admite no máximo um ponto fixo. De fato, se x 1 , x 2 E I fossem pontos fixos de f, com x 1 < x 2 , seguiria de f ser decrescente que

de modo que segue que f(O) = O. Fazendo y = x em (a), obtemos

f(2x) = f(x + x) = f(x) + f(x) = 2f(x), para todo x E JR. Fazendo agora y = 2x em (a), segue que

uma contradição à hipótese x 1 < x 2 .

f(3x) = f(x + 2x) = f(x) + f(2x) = f(x) + 2f(x) = 3f(x),

Exemplo 1.51 (Argentina). Seja f: lR--+ lR uma função decrescente e tal que f(x+ f(x)) = x+ f(x) para todo real x. Prove que J(f(x)) = x para todo real x.

para todo x E lR. Repetindo o argumento acima concluímos, por indução sobre n EN, que

Prova. As hipóteses sobre f garantem que x + f(x) é ponto fixo de f para todo x E lR. Por outro lado, o caráter decrescente de f garante, de acordo com a discussão anterior, a existência de no máximo um ponto fixo para f, de sorte que deve existir a E lR tal que x + f (x) = a para todo x E JR, o que é o mesmo que f(x) = a - x para todo x E JR. Portanto

f(nx) = nf(x), 'í/ n EN, x E lR

Em particular, fazendo x = 1 em (1.10), obtemos f(n) = n, para todo n EN. Fazendo agora x = em (1.10), segue que

i

1 = f(l) = f ( n · ~) = nf (~),

J(f(x)) = f(a - x) = a - (a - x) = x, para todo x E lR.



de modo que fornece

O próximo exemplo desenvolve ideias úteis em muitas outras situações, e a primeira parte do argumento que apresentamos a seguir resolve o problema 8, página 14.

f (i) =



Finalmente, x =

i

em (1.10), com m EN,

!(:) =f(n· !) =nf(!) =n· ! = ;. Vamos ver o que ocorre com os racionais negativos. Para isso, façamos y = -x no item (a), obtendo

Exemplo 1.52. Encontre todas as funções f: lR--+ lR tais que f(l) = 1 e, para todos x, y E JR, tenhamos

(a) f(x

(1.10)

O= f(O) = f(x + (-x)) = f(x) + f(-x),

+ y) = f(x) + f(y).

1

Tópicos de Matemática Elementar 3

50

ou ainda

f(x)

-f(-x), \/ x

=

ER

(1.11)

Em particular, sendo x < O racional, segue de (1.11) e do fato de ser -x um racional positivo que f(x) = -f(-x) = -(-x) = x; portanto, f(x) = x, para todo x E (Q. Como f(x) = x para todo x E (Q, desconfiamos que a função identidade seja a única satisfazendo as condições do enunciado. Para confirmar tal suposição, voltemos nossa atenção à condição do item (b). Inicialmente, mostremos que se para um certo x E R tivermos f(x) = O, então x = O. De fato, caso fosse x-=/:- O, fazendo y = em (b) teríamos

i

o que é uma contradição. Agora, fazendo y = x -=/:- O em (b), obtemos

f(x 2 ) = f(x · x) = f(x) · f(x) = J(x) 2 > O; portanto, se x,y E R, com x < y, e a-=/:- O for tal que y - x então, aplicando sucessivamente (a), (1.11) e (1.12), obtemos

f(y) - f(x) = f(y)

+ f(-x) =

=

51

Exemplo 1.53 (Lituânia). Encontre todas as funções que, para todos os naturais m e n, tenhamos

J(f(m)

+ f(n))

=

f : N---+ N tais

m + n.

Solução. Provemos primeiramente que f é injetiva. Para tanto, sejam m e n naturais tais que f(m) = f(n) = k. Então f(2k) = J(f(n) + f(n)) = 2n e, analogamente, f(2k) = 2m, de modo que deve ser m = n. Seja agora k > 1 natural. De (k - 1) + 2 = k + 1, segue que

J(f(k - 1) + f(2)) = k + 1 = J(f(k)

+ f(l)).

Pela injetividade de f, temos então f(k - 1) + f(2) = f(k) + f(l) ou, ainda, f(k) - f(k - 1) = f(2) - f(l), para todo natural k > 1. Escrevendo essa relação para k = 2, 3, ... , n e somando as igualdades assim obtidas, chegamos a

(1.12) =

f(n)

a2 ,

f(y - x) = f(a 2 ) = f(a) 2 > O,

de sorte que f é crescente. Suponha, por fim, que existe a E R tal que f(a) < a e tome (cf. problema 1.5.2, volume 1) um racional r tal que f(a) < r < a; o caráter crescente de f fornece

r

Antonio Caminha M. Neto

f(r) < f(a),

o que é uma contradição. Analogamente, não podemos ter f (a) > a, e a única possibilidade é f (a) = a. Mas, como a E R foi escolhido • arbitrariamente, devemos ter f(x) = x para todo x E R. Nosso último exemplo mostra que, para funções f : N---+ N, argumentos elementares de divisibilidade serão por vezes úteis.

=

(n - 1)(!(2) - f(l))

+ f(l),

para todo natural n > 1. Fazendo n = 2f(l) na relação acima, segue então que

2 = J(f(l)

+ f(l)) = f(2f(l)) = (2f(l) -

l)(f (2) - f(l))

+ f(l)

ou, ainda,

f(2) - f(l)

=

2 - f(l) . 2f(l) - 1

Mas f(2) - f(l) é inteiro, de modo que 2f(l) - 1 divide 2 - f(l). Assim, deve ser 2f(l) - 1 :::; 12 - f(l)I e é fácil concluir, a partir daí, que a única possibilidade é f(l) = 1, de modo que f(2) = 2. Segue então que f (n) = n, para todo n natural. •

Tópicos de Matemática Elementar 3

52

* Generalize a discussão do parágrafo anterior ao exemplo 1.51, mostrando que, se I e :IR. é um intervalo e f, g : I -+ I são funções tais que f é decrescente e g é crescente, então existe no máximo um x 0 E I tal que f(xo) = g(xo).

2. Ache todos os reais positivos x para os quais J2_+ 3. Encontre todas as funções

53

8. (Áustria-Polônia.) Prove que não existe função que, para todos x, y E Z, tenhamos

Problemas - Seção 1.5 1.

Antonio Caminha M. Neto

Jx =

~-

f : Q -+ Q tais que

f(x + f(y))

=

f : Z -+ Z tal

f(x) - y.

9. (Romênia.) Ache todas as funções f : Z-+ Z tais que f(O) e f(f(k)) + f(k) = 2k + 3,

=

1

para todo k E Z. 10. (Romênia.) Sejam k > 1 um inteiro ímpar e A= {x 1 , x 2, ... , xk} um conjunto de k números reais. Obtenha todas as funções injetivas f : A -+ A tais que

para todos x, y E Q. 4. (Áustria.) Encontre todas as funções f : Z \ {O} -+ Q tais que, para todos x, y E Z \ {O} para os quais x + y seja múltiplo de 3, tenhamos

5. (Vietnã.) Ache todas as funções 1

1

f:

11. Sejam a um real dado e f: :IR.-+ :IR. uma função tal que f(O) = e, para todos x, y E :IR., tenhamos

f(x

!

+ y) = f(x)f(a - y) + f(y)f(a - x).

:IR.-+ :IR. tais que ·

2J(xy) + 2J(xz) - f(x)f(yz)

2'.:

1

4,

para todos x, y, z E :IR.. 6. (Espanha.) Encontre todas as funções crescentes f : N -+ N tais que, para todo n EN, tenhamos f(n +f(n)) = 2f(n).

Prove que

f

é constante.

12. Ache todas as funções para todos x, y E Q.

f:

Q-+ Q~ tais que f(x+y) = f(x)f(y),

13. Ache todas as funções f : [O, 1] -+ [O, 1] tais que f(O) f(l) = 1 e

o,

+ y) + f(x - y) = 2f(x), para todos os x, y E [O, 1] tais que x - y, x + y E [O, l]. f(x

7. Encontre todas as funções (a) f(x +a)= f(x)

f : :IR. -+ Z tais que:

+ a, para todo x E :IR. e todo a E Z.

(b) J(f(x)) = O para x E [O, 1).

14. (Lituânia.) Seja f: Z-+ Z uma função tal que f(m 2 + f(n)) = f(m) 2 + n, para todos m, n inteiros.

Tópicos de Matemática Elementar 3

54

(a) Prove que f(O)

= O e f(l) =

1.

(b) Ache todas tais funções. 15. (OIM.) Encontre todas as funções crescentes que f(yf(x)) = x 2 f(xy), para todos x, y EN. 16. (IMO.) Ache todas as funções duas condições a seguir: (a) f(xf(y))f(y) (b)

f (2)

=

Oe f (x)

=

f : N -+ N, tais

f: [O, +oo)-+ IR satisfazendo as

f(x + y), para todos x, y

E

CAPÍTULO 2

[O, +oo).

# Opara O~ x < 2.

17. (IMO.) Seja S = {x E IR; x > -1}. Obtenha todas as funções f : S -+ S que satisfaçam as duas condições a seguir:

Gráficos de Funções

(a) f(x+ f(y)+xf(y)) = y+ f(x)+yf(x)), para todos x, y E S. (b)

f~)

é crescente em cada um dos intervalos (-1, O) e (O, +oo).

18. (IMO.) Decida se existe uma função condições a seguir:

f : N-+ N satisfazendo as

(a) f(l) = 2. (b) f(n) < f(n + 1), para todo n EN. (c) J(f(n))

=

f(n)

+ n,

=

f : IR-+ IR tais que, para todos

f(x + y) + f(x)f(y) - xy.

20. (Polônia.) Encontre todas as funções f: Q~-+ Q~ satisfazendo, para todo racional positivo x, as seguintes condições:

(a) f(x

+ 1) =

(b) f(x 3 )

=

f(x) f(x)3.

+ 1.

Gt

para todo n EN.

19. (Irã.) Obtenha todas as funções x, y reais, tenhamos

J(f (x + y))

Dada uma função f : X -+ Y, o gráfico de do produto Cartesiano X x Y definido por

= {(x,y)

E X

f é o subconjunto G f

x Y; y = f(x)}.

(2.1)

Quando f : X -+ IR for uma função real de variável real, com X e IR uma união finita de intervalos (possivelmente X = IR), o gráfico de f se reveste de significativa importância geométrica, uma vez que G J e X x Y e IR x IR, e esse último conjunto pode ser identificado com o plano, munido de um sistema Cartesiano de coordenadas fixado 1 . 1 Referimos

o leitor ao capítulo 6 do volume 2 para uma discussão sobre sistemas Cartesianos de coordenadas.

56

Tópicos de Matemática Elementar 3

Antonio Caminha M. Neto

57

Nosso propósito neste capítulo é examinar alguns exemplos e propriedades simples de gráficos de funções f : X ---+ ~' quando X e ~ for uma união finita de intervalos, postergando para os capítulos 4, 5 e 6 a discussão das propriedades dos gráficos de funções contínuas e deriváveis. Em tudo o que segue supomos fixado, no plano, um sistema Cartesiano de coordenadas.

um ponto (x 0 , y0 ) no plano Cartesiano, com x0 E [a, b). Claramente, (xo, y0 ) E r; além disso, se x 0 for uma solução da equação f(x) = y0 , i.e., se J(x 0 ) = y0 , então teremos também (x 0 , y0 ) E Gt. Assim, a reta horizontal de ordenada y0 intersectará o gráfico exatamente quando y0 pertencer à imagem de f. O raciocínio para uma função qualquer f : X ---+ ~' com X e ~' é inteiramente análogo e nos permite concluir que

2.1

A imagem de f é precisamente o conjunto dos y0 E ~ tais que a reta horizontal de ordenada y0 intersecta o gráfico de f.

Generalidades e exemplos

Há uma interpretação geométrica bastante simples para a imagem de uma função real de uma variável real em termos de seu gráfico. Para exibi-la, marquemos, no plano Cartesiano da figura 2.1, os pontos de interseção do gráfico de uma função f : [a, b) ---+ ·~ com uma reta horizontal r, de ordenada y = y0 . Seja (x 0 , y0 ) um ponto comum

Figura 2.1: imagem x gráfico.

à reta e ao gráfico. Por pertencer ao gráfico de f, o ponto (x 0 , y0 ) deve ser tal que Xo E [a, b) e f(x 0 ) = y0 . Reciprocamente, seja dado

Se I e ~ é um intervalo, a monotonicidade de uma função f I ---+ ~ também nos diz muito sobre o comportamento de seu gráfico. Por exemplo, supondo que f seja crescente (resp. decrescente) em I, concluímos que, à medida que a variável x aumenta em I, os valores J(x) aumentam (resp. diminuem) em~' de maneira que o gráfico de f sobe (resp. desce). Por outro lado, se y0 E ~ é o valor mínimo de f : I ---+ ~ e x 0 E I é um ponto de mínimo de f (cf. definição 1.24), então o ponto (x, f(x)) está acima ou coincide com o ponto (x, y0 ), para todo x E I (cf. figura 2.2). De outra forma, o gráfico de f está contido no semiplano superior fechado determinado pela reta horizontal y = y0 , tocando tal reta no ponto (xo, Yo). Observe que os conceitos de valor máximo e ponto de máximo de uma função f : I ---+ ~ admitem interpretações geométricas análogas às discutidas acima. Vale ainda observar que nem todo subconjunto do plano (munido de um sistema Cartesiano xOy) pode ser visto como gráfico de uma função. De fato, suponha dada uma função real de uma variável real f : X ---+ ~' tal que X é uma união finita de intervalos. Se (x 0 , y0 ) E Gt, então x 0 E X, pela definição de gráfico; por outro lado (e mais importante), fixado x 0 E X, se A 1 (x 0 , y1 ) e A 2 (x 0 , y 2 ) são pontos sobre

58

Tópicos de Matemática Elementar 3

Antonio Caminha M. Neto

59 y

(-3, O)

X

Y = Yo

Figura 2.2: ponto de mínimo de f : I--+ IR.

o gráfico de f, então, novamente pela definição de gráfico, temos Y1

= f(xo) =

Figura 2.3: subconjuntos do plano que não são gráficos de funções.

i.e., o gráfico de f é a reta paralela ao eixo-x e passando pelo ponto (O, e) do eixo-y (figura 2.4).

Y2,

de maneira que A1 = A2 . Em resumo, para x 0 E JR, a reta vertical x = x 0 do sistema Cartesiano em questão intersecta o gráfico de f se, e somente se, x 0 E X; ademais, nesse caso tal reta intersecta o gráfico em exatamente um ponto. Assim, o subconjunto C do plano Cartesiano esboçado na figura 2.3 não representa o gráfico de função alguma f : [-3, 3] --+ JR, uma vez que toda reta vertical paralela às retas tracejadas e situada na faixa cinza intersecta Cem mais de um ponto. Por fim, vejamos alguns exemplos importantes de gráficos de funçoes. Exemplo 2.1. Seja f : lR --+ lR a função constante e igual a e. O gráfico de f é o conjunto G1 = {(x,y); x E lR e y =e}= {(x,c); x E JR},

Exemplo 2.2. Lembre-se (cf. definição 1.2) de que a função identidade IdIR : lR --+ lR é tal que IdIR (x) = x, para todo x E R Seu gráfico é, portanto, o conjunto Gld]R = {(x,y); x E lR e y = x} = {(x,x); x E JR}. É um exercício fácil de geometria Euclidiana verificar que os pontos da forma (x, x) e (-x, x) são os pontos do plano Cartesiano situados sobre as bissetrizes dos ângulos formados pelos eixos coordenados, sendo que os da forma (x, x) pertencem ao primeiro ou terceiro quadrantes. Portanto, o gráfico da função ldIR é a reta da figura 2.5, denominada a bissetriz dos quadrantes ímpares. Observe que o conjunto dos pontos da forma (x, -x), i.e., a bissetriz dos quadrantes pares, é o gráfico da função f : lR --+ lR dada por f (x) = - x.

Exemplo 2.3. A função modular é a função f : lR --+ lR dada por f(x) = !xi. Segue imediatamente da definição de módulo de um

!'r' :

1

Tópicos de Matemática Elementar 3

60

61

Antonio Caminha M. Neto

y

y

(O, e) (O, a)

o

X

(a, O)

Figura 2.4: gráfico da função constante f(x)

=

e, V x E R

Figura 2.5: gráfico da função identidade ldJR..

número real (cf. seção 2.2 do volume 1) que

Gt

X

{(x, lxl); x E IR} {(x, lxl); x E IR+} U {(x, lxl);

X

E

IR_}

{(X, X); X E IR+} U {(X, - X); X E IR_}. Como os pontos (x, -x) e (x, x) são simétricos em relação ao eixo-x, o gráfico da função modular é obtido refletindo a porção do gráfico da função ldJR. situada no terceiro quadrante em relação ao eixo-x (cf. figura 2.6). Exemplo 2.4. SeJ(x) = ax+b é uma função afim, então seu gráfico é o subconjunto do plano Cartesiano dado por G1

= {(x,y);x,y

E

IR e y

= ax + b}.

De acordo com a discussão da seção 6.2 do volume 2, o gráfico de fé a reta de equação y-ax-b = O, com coeficiente angular a e passando pelos pontos A(-~, ü) e B = (O, b). A figura 2.7 esboça o gráfico de f(x) = ax + b para a, b > O.

Para o que segue, recorde (cf. problema 6.3.13 do volume 2) que, dados um ponto F e uma reta d no plano, com F ~ d, a parábola de foco F e diretriz d (cf. figura 2.8) é o lugar geométrico dos pontos P do plano tais que P F = dist(P, d). Por fim, o eixo da parábola é a reta que passa por F e é perpendicular à diretriz d, e o vértice da mesma é seu ponto V de interseção com o eixo. Mostraremos, no que segue, que o gráfico de toda função quadrática é uma parábola. Mais precisamente, temos o seguinte Teorema 2.5. Para a, b, e E IR, com a =J. O, o gráfico da função quadrática f (x) = ax 2 + bx + e é a parábola de eixo {x = - la} e vértice V (-;ª, - fa), "aberta para cima" se a > O, e "aberta para baixo" se a< O. Prova. Procuremos x 0 , y0 , k E IR tais que Yo =J. k e, sendo F(xo, Yo) e

Tópicos de Matemática Elementar 3

62

Antonio Caminha M. Neto

63

y

1 / 1 / 1 / 1 /

X

X

(x,x).-/ '/ / /

Figura 2.6: gráfico da função modular f(x) =

d: {y

=

lxl.

Figura 2.7: gráfico da função afim f(x) = ax + b.

o que é imediato: dividindo as duas primeiras equações, obtemos x 0 = - 2: ; em seguida, substituindo a primeira equação e o valor de x 0 na terceira equação, segue que

k }, tenhamos P E G1 ~ PF = dist(P,d).

Para tanto, sendo P(x, y), observemos que

P E Gf ~ y

=

ax 2 + bx + e

por fim, resolvendo o sistema

e

PF

= dist(P, d)~

J(x - xo) 2 + (y - Yo) 2

Yo - k

= IY - kl,

= ax2 + bx + e {::}

(x - xo) 2

{::} y=

+ (y -

1

2(yo - k)

X

2

Yo) 2 -

= (y .Xo

k) 2 2

Yo - k

+

k2

2

Xo Yo x+-----

1(

2(yo - k)

+ 2 Yo +

+ k = - 2ª,



Basta, então, resolvermos em x 0 , y0 e k o sistema de equações

1 = a _ Xo = b Xõ 2(yo - k) ' Yo - k ' 2(y0 - k)

Yo

obtém-se y0 = l-.ó. e k = - l+.ó.. 4a 4a Finalmente, uma vez que o vértice V da parábola é a interseção da reta x = com o gráfico, temos

de sorte que queremos que y

~

1

= 2ª,

k) =

e,

y

=a

(-!_) 2a

2

+ b (-.!!__)+e= - 4a ~. 2a

• Terminamos esta seção estabelecendo, na proposição a seguir, uma importante relação entre os gráficos de uma bijeção e de sua inversa.

Tópicos de Matemática Elementar 3

64

Antonio Caminha M. Neto

65

Exemplo 2. 7. Esboce o gráfico da função raiz quadrada

f : [O, +oo) -----+ [O, +oo)

VX

f-----+

X

Solução. Vimos no exemplo 1.45 que f é a inversa da função g [ü,+oo) -+ [ü,+oo) dada por g(x) = x 2 . Como já conhecemos o gráfico de g, segue da proposição anterior que o gráfico de f é obtido como o simétrico do gráfico de g em relação à reta y = x (figura 2.9) .



y

g

I I I I

/

I

/

I

Figura 2.8: parábola de foco F e diretriz d.

/ /

I

/

I I / I /

/

f

1/

Em seguida, ilustramos tal resultado na construção dos gráficos de duas funções importantes. X

/ /

Proposição 2.6. Se os conjuntos não vazios I, J C IR são uniões finitas de intervalos e f : I -+ J é uma bijeção, então os gráficos de f e 1- 1 são simétricos em relação à bissetriz dos quadrantes ímpares do plano Cartesiano.

/

/ / / /

Figura 2.9: gráfico de x

f---+

y'x.

Prova. Fixe a E I e b E J. Pela definição de função inversa, temos que

(a, b)

E G1

{::} b = J(a)

{::} a=

1- 1 (b)

{::} (b,a)

E

G1-1.

Mas, como os pontos (a, b) e (b, a) são simétricos em relação à reta y = x, nada mais há a fazer. •

Exemplo 2.8. Recorde que a função de proporcionalidade inversa é a para todo x E IR\ {O}. função f: IR\ {O}-+ IR\ {O}, tal que f(x) = De posse da discussão desenvolvida até o momento, podemos esboçar muito acuradamente seu gráfico. De fato, f é claramente decrescente em (O, +oo); também já sabemos que fé ímpar (cf. problema 10, página 37), de sorte que, pelo problema 5, seu gráfico é simétrico

i,

.1

Tópicos de Matemática Elementar 3

66

em relação à origem do plano Cartesiano; por outro lado, f é a inversa de si mesma, e a proposição anterior garante que seu gráfico também é simétrico em relação à bissetriz dos quadrantes ímpares; por fim, mostraremos na seção 5.3 que seu gráfico é emborcado para cima em

(0,+oo).

Antonio Caminha M. Neto Problemas - Seção 2.1

1. Seja f : IR -+ IR a função quadrática dada por f (x) = ax 2 +bx+c, onde a-=/:- O. Sabendo que x 1 = -1 e x 2 = 5 são as raízes de f, e que f(l) = -8, pede-se:

i

(a) Encontrar a, b, e.

Observando que f(x) = se aproxima cada vez mais de zero à medida que x aumenta, chegamos ao esboço do gráfico de f constante da figura 2.10, construído com o auxílio das observações acima e dos pontos auxiliares (n, ~), para 1 ::::; n :S 4 inteiro.

(b) Calcular f(O). (c) Calcular as coordenadas do vértice de (d) Esboçar o gráfico de

f.

f.

2. Sejam I e IR um intervalo e f : I -+ IR uma função dada. Dizemos que fé limitada se existe M > O tal que lf(x)I : : ; M, para todo x E J. Prove que, nesse caso, o gráfico de f está contido na faixa horizontal do plano Cartesiano delimitada pelas retas y = ±M.

y / y=

67

X

/

/ /

/ / / / / /

3. X

* Se

e IR é um intervalo e f : I -+ I é uma função dada, mostre que os pontos fixos de f são, precisamente, as abscissas dos pontos de interseção do gráfico de f com a bissetriz dos I

quadrantes ímpares.

/ / / / /

4.

/ /

/ /

Figura 2.10: gráfico da função f(x) =

* Se

I e IR é um intervalo e f, g : I -+ IR são funções dadas,

explique como identificar os pontos comuns aos gráficos de g, traçados em um mesmo sistema Cartesiano.

/

f e

Para o próximo problema, sugerimos ao leitor reler o enunciado do problema 10, página 37.

i· 5.

* Sejam I e

IR uma união de intervalos, simétrica em relação a O E IR, e f: I-+ IR uma função dada. Prove que: (a) Se f for par, então G f é simétrico em relação ao eixo das ordenadas.

Tópicos de Matemática Elementar 3

68

(b) Se

Antonio Caminha M. Neto

f for ímpar, então GI é simétrico em relação à origem.

(c) g ( x) = - f (x) é obtido refletindo o gráfico de eixo das abscissas.

6. Em cada um dos itens a seguir, esboce, num mesmo sistema Cartesiano, os gráficos das funções reais de uma variável real listadas:

(e) g(x) = af(x) é obtido alongando2 o gráfico de f verticalmente do fator a, se a > O.

= X, Í2 (X) = x 3 e Í3 (X) = x 5.

7. Esboce o gráfico da função todo x E R

f:

(f) g(x)

IR----+ IR tal que f(x) = ijx, para

8. Esboce, com justificativa, o gráfico da função parte inteira, IR----+ IR (cf. problema 5, página 13).

l J:

Para o próximo problema sugerimos ao leitor reler o enunciado do problema 15, página 37. 9.

f ao longo do

(d) g(x) = f(-x) é obtido refletindo o gráfico de f ao longo do eixo das ordenadas.

(a) fi(x) = x 2 , h(x) = x 4 e h(x) = x 3 . (b) Íi (X)

69

f(ax) é obtido alongando o gráfico de f horizontal-

mente do fator a, se a> O. 11. Sejam I um intervalo da reta e f : I ----+ IR uma função dada. Que relação existe entre os gráficos de f e da função g : I ----+ IR dada por g(x) = lf(x)I? Utilize suas conclusões, juntamente com o resultado do problema anterior, para esboçar os gráficos das funções abaixo listadas: (a) g(x)

* Faça os seguintes itens:

=

(b) g(x) =

(a) se f : IR----+ IR é periódica de período p > O, explique como construir o gráfico de f conhecendo a porção do mesmo para OS x < p; (b) use o item (a) para construir o gráfico da função parte fracionária, { } : IR ----+ IR (cf. problema 6, página 13).

=

(c) g( X)

lx~ll' para x E IR\ {-1}.

lx2 - 41, para x

= 1x 2 - 1X + 21 + 21, para X E R

(d) g(x) = 112.

E R

(x~ 2) 2

'

para todo real x-=/=- 2.

* Esboce o gráfico da função f:

IR\ {2}----+ IR dada por f(x)

=

X

10.

* Sejam f : IR

----+ IR uma função dada e a Prove que o gráfico de:

-=/=-

2-x·

O um real dado.

(a) g(x) = f(x + a) é obtido transladando o gráfico de f de -a, paralelamente ao eixo das abscissas. (b) g(x) = f(x) + a é obtido transladando o gráfico de f de a, paralelamente ao eixo das ordenadas.

13. Prove que o gráfico da função de proporcionalidade inversa é obtido pela rotação trigonométrica da hipérbole de equação x 2 y2 = 2, do ângulo trigonométrico de ~ radianos. 20

leitor deve ter cuidado com o sentido em que a palavra alongando é utilizada aqui; para tanto, compare os casos O < a < 1 e a > 1.

70

2. 2

Tópicos de Matemática Elementar 3

Funções trigonométricas

A função seno é a função sen : lR ----+ JR, que associa a cada x E lR o seno de um arco de x radianos: lR sen : lR --+ x f-------t sen x Analogamente, definimos a função cosseno por cos : lR --+ X

f-------t

lR COS X

As propriedades básicas das funções seno e cosseno são estabelecidas na proposição a seguir, para a qual o leitor pode achar útil recordar os enunciados do problema 15, página 37 e do problema 10, página 37. Proposição 2.9. As funções seno e cosseno têm como imagem o intervalo [-1, 1] e são periódicas de período 21r. Ademais, a função seno é ímpar e a função cosseno é par. Prova. Imediata da discussão das seções 7.1 e 7.2 do volume 2.

seção 5.3 mostraremos (cf. exemplo 5.19) que tal gráfico é "emborcado para baixo" no intervalo [O, n]. Assumindo por enquanto a validade dessas afirmações, podemos finalmente esboçar o gráfico da função seno. Exemplo 2.10. Reunindo as informações de que dispomos até o momento sobre a função seno em [O, n] (imagem, continuidade e concavidade), juntamente com o fato de que a mesma é crescente em [O, iJ, decrescente em [i, n] e satisfaz sen (n - x) = senx, a fim de esboçarmos razoavelmente o gráfico da mesma nesse intervalo basta tabelarmos alguns valores de senx para x E [O, iJ, o que fazemos a segmr:

o o

7r

7r

7r

7r

6

4

3

2

1/2 \1'2/2 v13/2 1

De posse das informações acima e utilizando a periodicidade do seno, obtemos imediatamente a figura 2.11, primeiro no intervalo [O, n] e, em seguida, no intervalo [-n, 1r].



De acordo com a proposição acima e a discussão contida no problema 15, página 37, a fim de esboçarmos o gráfico da função seno, é suficiente fazê-lo no intervalo [-n, 1r], copiando em seguida essa porção do gráfico em cada um dos intervalos da forma [-n + 2kn, 1r + 2kn], onde k E Z. Por outro lado, uma vez que a função seno é ímpar, a fim de obtermos seu gráfico no intervalo [-n, n], é suficiente construí-lo no intervalo [O, n]; feito isto, ao refleti-lo em torno da origem do plano Cartesiano obtemos (de acordo com o item (c) do problema 10, página 37, uma vez que a função seno é ímpar) o gráfico no intervalo [-n, n]. Provaremos na seção 4.1 (cf. exemplo 4.10) que o gráfico da função seno é uma curva contínua, i.e., sem interrupções. Por outro lado, na

71

Antonio Caminha M. Neto

y

-------------.,..1 /

/ /

/

-7r

~' 2 ' , .....

X

________ ____ _ -1

Figura 2.11: gráfico das funções seno e cosseno.

Observemos agora que, a partir das fórmulas de adição de arcos,

Tópicos de Matemâtica Elementar 3

72 temos

cosx = sen

73

Antonio Caminha M. Neto

(x+ i). a

Portanto, o item (a) do problema 10, página 68, garante que, urna vez esboçado o gráfico da função seno, obtemos o esboço correspondente ao gráfico da função cosseno transladando o gráfico da função seno de -i paralelamente ao eixo das abscissas. Na figura 2.11, o gráfico da função cosseno no intervalo [-1f, 7í] é representado pela curva pontilhada, situada na faixa do plano Cartesiano entre as retas y = -1 e

b Figura 2 .12: definindo o ângulo a.

y = 1. (figura 2.12). Assim, ternos das fórmulas de adição de arcos que

I

l'I'

,,','I

Vejamos, agora, um exemplo relevante de aplicação das fórmulas de adição de arcos da proposição 7.18 do volume 2, o qual nos diz como proceder para estudar as funções construídas corno uma certa sorna de múltiplos das funções seno e cosseno. Exemplo 2.11. Dados reais positivos a e b, seja f: R--+ R a função

dada por

f (x)

a cos x + b sen x J a 2 + b2(cos a cosx +sena senx) J a 2 + b2 cos(x - a).

Em particular, segue de I cos(x - a)I :::; 1 que lf(x)I = Va2 + b2Icos(x - a)I:::; Ja 2 + b2,

f(x) = acosx + bsenx.

e não é difícil provar, a partir daí, que a imagem de

Escrevendo

f é precisamente

o intervalo [-e, e], onde e= Ja 2 + b2 (veja o problema 1).

acosx + bsenx = Ja 2 + b2 (

ª

J a2 +

b2

cosx + .

b

J a2 +

b2

senx),

observemos que

Voltemo-nos, por fim, ao estudo da função tangente, i.e., da função que associa, a cada real x em seu domínio, o número real tg x. Urna vez que cos x =

o{:: }- x = 27f + k1f,

:3 k E Z,

o domínio (maximal) da função tangente é o conjunto Portanto, o ponto P ( v'a~+b2 , v'a:+b2 ) pertence à porção do ciclo trigonométrico r situada no primeiro quadrante e, daí, existe um real a E (O, i) tal que cosa i

(2.2)

=

a b e sena = . 1 J a2 + b2 v a2 b2

+

7f

D= R \ { 2 + k1f; k E Z},

de sorte que a função desejada é tg: D --+ X

f--t

R tgx

74

Tópicos de Matemática Elementar 3

Antonio Caminha M. Neto y

Para x E D, temos tg (X+ 7r )

75

=

sen (x + 1r) cos(x + 1r)

=

-senx - cosx

= tg X, 1

e é imediato verificar que nao existe um real O < p < 1r tal que tg (x + p) = tgx, para todo x E D. Portanto, a função tangente é periódica de período 1r. Ademais, uma vez que D é um subconjunto de lR simétrico em relação a O e







-21-4I 1

1

-1

1

1

=

sen (-x) cos(-x)

=

-senx cosx

1 1 1 1 1

= - tg X

para todo x E D, concluímos que a função tangente é ímpar. Dado o aci~a exposto, para esboçar o gráfico da função tangente é suficiente tê-lo à mão no intervalo [O,~). De fato,"8eu caráter í~par garante que o gráfico no intervalo ( - ~, O] é obtido por reflexão em torno da origem do sistema Cartesiano da porção do mesmo no intervalo [O,~). Por outro lado, uma vez esboçado o gráfico no intervalo ( - ~, ~), a periodicidade da função tangente nos permite esboçá-lo em todos os intervalos da forma ( - ~ + k1r, ~ + k1r), com k E Z: basta transladar o gráfico no ( - ~, ~) de k1r unidades paralelamente ao eixo das abscissas, para todo k E Z. Provaremos na seção 4.1 (cf. problema 3, página 127) que o gráfico da função tangente, restrita ao intervalo ( - ~, ~), é uma curva contínua, i.e., sem interrupções. Por outro lado, na seção 5.3 mostraremos (cf. exemplo 5.20) que tal gráfico é "emborcado para cimd' no intervalo [O,~). Assumindo por enquanto a validade dessas afirmações, e de posse da discussão dos parágrafos anteriores, podemos esboçar o gráfico da função tangente de forma análoga ao feito no exemplo 2.10, obtendo aproximadamente a figura 2.13.

X

1

1 1 1 1 1

1

tg (-X )



4 12

1 1 1

Figura 2.13: gráfico da função tangente.

Problemas - Seção 2.2 1.

* Sejam a e b reais não ambos nulos, e f : lR -t lR a função dada por

f(x) = acosx + bsenx. (a) Obtenha, com justificativa, o conjunto Im (!). (b) Prove que

f é periódica de período 21r.

(c) Esboce os gráficos de sistema Cartesiano.

f

e da função seno em um mesmo

2. Seja f : lR -t lR a função dada por f(x) = 2 senx + cos 2x. Calcule os valores máximo e mínimo de f, bem como os números reais x para os quais f assume tais valores. 3. (Canadá.) Calcule o número de soluções reais da equação sen x = X

10·

76

Tópicos de Matemática Elementar 3

4. Encontre o valor máximo assumido pela função definida por f(x) = 3x + 4J5 - x 2 .

f : [-1, 1] -+ lR

5. (Nova Zelândia.) Seja a um número irracional dado. Prove que a função f : lR -+ JR, definida por

f(x) = cosx + cos(ax),

CAPÍTULO 3

para x E JR, não é periódica. 6. Encontre valores inteiros de n para os quais a função dada para x E lR por

f (x)

=

f : lR -+ JR,

cos (nx) sen ( 5: ) ,

Mais sobre Números Reais

é periódica de período 31r. 7. (Canadá.) Prove que a função f sen (x 2 ) não é periódica.

lR -+ lR dada por f(x)

Antes de prosseguirmos em nosso estudo de funções, precisamos de um pequeno interlúdio técnico, a fim de apresentar as noções de limite de uma sequência (infinita) de números reais e de série convergente. Dentre outras aplicações, tais noções nos permitirão apresentar o importante exemplo de uma série geométrica convergente, bem como introduzir o segundo dos dois mais famosos números da Matemática1 , o número e. Apresentamos ainda um famoso teorema de Kronecker sobre subconjuntos densos da reta, o qual encontrará várias aplicações interessantes, aqui e em capítulos subsequentes. 1 Como

o leitor deve suspeitar, o outro é o número 1r, definido no volume 2 como o valor numérico da área de um círculo de raio 1.

78

3.1

Tópicos de Matemática Elementar 3

Limites de sequências

79

Exemplos 3.2.

Dada uma sequência (an)n2'.I (em~), estamos interessados em reconhecer se os números reais an se aproximam cada vez mais de um certo número real l, à medida que n aumenta; por exemplo, se an = ~' então é razoável dizer que os números an se aproximam de O à medida que n aumenta, haja vista que o resultado da divisão de 1 por n é cada vez menor à medida que n aumenta. Temos então a definição a seguir.

Definição 3.1. Dizemos que uma sequência (an)n:::: 1 converge para um real l quando, uma vez prescrito um erro E > O para o valor de l, existir um índice n 0 EN tal que lan - li < E, para todo n > n 0 . Alternativamente, se (an )n2'.l convergir para l, diremos que a sequência é convergente e que l é um limite da sequência, o que denotamos escrevendo an ~ l OU lim an = l. n--++oo Por fim, uma sequência que não é convergente é dita divergente. Em geral, diminuindo a aproximação E > O, é de se esperar que tenhamos de aumentar o natural n 0 da definição de convergência. Em outras palavras, é de se esperar que n 0 dependa de E > O. De qualquer modo, o importante para assegurar a convergência da sequência (an)n2'.I é o fato de que, fixada arbitrariamente uma aproximação E> o,, sejamos capazes de escolher n 0 E N tal que

n >no::::} lan - li
n 0 . (b) Se an = (-1)\ então (an)n2:1 não converge: de fato, como os termos da sequência são alternadamente iguais a 1 ou -1, não podem aproximar-se todos de um mesmo real l (formalize esse argumento intuitivo). (c) Se an = 1 + (-~)n, então an ~ 1: isso porque lan maneira que lan - li < E para n > !.. E (d) Se (an)n2:1 é uma sequência constante, com an n ~ 1, então an ---+ e.

Exemplo 3.3. Se an = qn, com O
1




!

4

2 ,

vn+Í+vn·

Assim, dado E> O, tome no EN tal

de sorte que

n >no==} vn + 1 +

vn > vno + 1 +../no> 2.,/nõ > -1

n

> no

==}

lan -

Proposição 3.6. Se (an)n~l é uma sequência com limite l, então:

E

e, daí,

OI

1 = .~

. r;;;

vn+l+vn


no==} lan - li < E; em particular, para n > n 0, temos

Proposição 3.5. Se a sequência (an)n~l convergir, então seu limite é único.

an < l +E= l + (a - l) = a,

Prova. Sejam li e l 2 reais distintos e suponha que a sequência convergisse simultaneamente para li e h- Tomando E= !Ili - hl > O, a definição de limite garante a existência de n1, n2 E N tais que

o que é um absurdo. (b) Seja dado E > O. Como an ~ l, existe um natural n 0 tal que lan - li < E, para n > no. Mas, como n 1 < n2 < n3 < ···,existe um índice ni na subsequência tal que nj > n 0 para j 2:: i; portanto, para tais j, temos lani-ll
O. Existe no E N tal que n > n 0 :::} lan - ai < ~

Até agora, exceto por alguns exemplos bastante simples, não vimos ainda como seria possível descobrir o limite de uma sequência que saibamos ser convergente. Para tanto, precisamos entender como é possível operar com limites de sequências, problema que examinamos a partir de agora. Precisaremos, inicialmente, do seguinte resultado auxiliar.

Lema 3.8. Toda sequência convergente é limitada.

1

'! i

. 1

n >no:::} lcan - cal= lcllan - ai< lei· 1; 1 = E. (b) Provemos que an + bn ---+ a + b (provar que an - bn ---+ a - b é análogo). Dado E> O, existem n 1, n 2 EN tais que

n > n1:::} lan - ai
n2:::} lbn - bl
O, escolha n 0

lbl 2
O dado; como l - E não é mais cota superior de A, algum elemento de A é maior que l - E, digamos ano > l - E. Mas, como ªno :::; ªno+l :::; ano+ 2 :::; • • • , concluímos que an > l - E, para n 2: no. Assim, para n 2: no, temos

E N tal que

n >no=}

Antonio Caminha M. Neto

bl < 2E .

Para n > n 0 , temos então que

l -

• Para o que segue, recordemos que uma sequência (an)n21 de números reais é simplesmente uma função f : N --+ JR, para a qual convencionamos a notação an = J(n). Portanto, (an)n21 é: • monótona crescente (resp. decrescente, não decrescente, não crescente) se an < ªn+l (resp. an > ªn+i, an :::;· ªn+l, an 2: ªn+1), para todo n 2: 1. • limitada, se existe um real positivo M tal que lanl < M, para todo n 2: 1.

O resultado mais importante sobre limites de sequências é o teorema a seguir, conhecido na literatura como o teorema de BolzanoWeierstrass2. Teorema 3.10 (Bolzano-Weierstrass). Toda sequência monótona e limitada é convergente. Prova. Suponhamos que (an)n 21 é uma sequência monótona não decrescente e limitada, i.e., que

como desejado.

E

< an :::; l < l + E,



O teorema acima e a definição de convergência garantem que, se uma sequência limitada for monótona a partir de um determinado termo, então ela ainda será convergente. Exploramos essa observação nos dois exemplos a seguir. Exemplo 3.11. Dado um real positivo a, a sequência (an)n21 dada por an = efa, é convergente e seu limite é igual a 1. Prova. Se a > 1, então a 1 > a 2 > a3 > · · · > 1, de sorte que, pelo teorema de Bolzano-Weierstrass, existe l = limn-++oo ªn· Para mostrarmos que l = 1, seja an = 1 + bn. Então

de sorte que O < bn < ~- Portanto, bn --+ O quando n --+ +oo, o que por sua vez garante que an = 1 + bn --+ 1. Se O < a < 1 e an = efa,, provamos analogamente que an --+ 1. • Exemplo 3.12. A sequência (an)n21 dada por an = {/ri, é convergente e seu limite é igual a 1.

para algum M > O (os demais casos são análogos). Então M é uma cota superior para o conjunto A = { a1, a2, a3, ... }, de sorte que a 2 Após

Karl Weierstrass, matemático alemão do século XIX.

Prova. Os termos iniciais da sequência são v'2, ?'3, ~' ... , e é fácil verificar diretamente que v'2 < ,v'3 e ,v'3 > ~ > {15. Como 2n 2: n 2 para n 2: 4 (por indução, por exemplo), temos a 2 2: an para n 2: 4, de

Tópicos de Matemática Elementar 3

86

sorte que a sequência é limitada; portanto, se mostrarmos que ela é decrescente a partir de seu terceiro termo sua convergência seguirá do teorema de Bolzano-Weierstrass. Para o que falta, dado n > 2 inteiro, temos

v1n > n+«n + 1 {:} n n+ 1 > (n + 1r

{:} n > ( 1 + ~) n

Provemos a última desigualdade acima. Para n = 3 ela é de verificação imediata; para n > 3, basta mostrarmos que ( 1 + i) n < 3. Para tanto, observemos que (l

+ ! )n n

=

1+

(n) ! + (n) 2_ + ... + (n) 2_ 1 n

2 n2

n

nn

e

(n)k 2_nk = k!(n -n! k)!nk = ~k! . n(n - 1) · ·nk· (n - k + 1) < ~k! M, temos



Vejamos um exemplo interessante de aplicação do teorema acima. Exemplo 3.17. Seja (an)n>l uma sequência de números reais tal que

para todo n E N, onde O < e < 1 é um real fixado. Mostre que tal sequência é convergente.

1

Tópicos de Matemática Elementar 3

90

91

Antonio Caminha M. Neto

Prova. Pelo teorema 3.16, basta mostrarmos que se trata de uma sequência de Cauchy. Para tanto, iterando a propriedade satisfeita pela sequência, é imediato que, para todo k E N, tenhamos

4. (Torneio das Cidades.) Seja (an)n 21 uma sequência de naturais dois a dois distintos, todos maiores que 1. Prove que há infinitos naturais k tais que ak > k.

Sejam, agora, n e p naturais dados. Temos

5. Sejam (an)n21 e (bn)n21 sequências de números reais e, para cada n E N, seja tn E [O, 1] um real fixado. Denote por (Cn)n21 a sequência definida por

n+p-1 lan+p - anl ::;

L

n+p-1 lak+l - akl ::;

k=n

L

ck-lla2 - a1I

k=n

la2 - a1lcn-l ( 1 ;

~p:l),

e a última expressão acima tende a O quando n -t +oo, pelo exemplo 3.3. Portanto, (an)n 21 é, realmente, uma sequência ·de Cauchy. •

para todo n EN. Se ak, bk -te, prove que Ck -te. 6.

* Prove

o teorema do confronto: sejam (an)n21, (bn)n21 e (Cn)n 21 sequências tais que an ::; bn ::; Cn, para todo n E N. Se an, Cn -t l, para algum l E R, então Cn -t l.

7. Seja (an)n 21 uma sequência de números reais tais que, para todos os m, n E N, tenhamos Iam - anl::;

Problemas - Seção 3.1

2mn 2

m +n



Prove que a sequência é constante. 1.

* Sejam

(an)n21 e (bn)n21 sequências convergentes de números reais, com limn-Hoo an = a e limn-Hoo bn = b. Generalize o item (a) da proposição 3.6, mostrando que se an ::; bn para todo n ~ 1, então a ::; b.

2. (Hungria.) Seja (Rn)n 21 uma sequência infinita de retângulos dois a dois distintos no plano Cartesiano, cada um deles com vértices (O, O), (an, O), (O, bn) e (an, bn), para algum par de inteiros positivos an, bn. Prove que há dois desses retângulos tais que um contém o outro.

3. Generalize o resultado do exemplo 3.3, mostrando que, dados k k E N e a E R, com ial > 1, temos -t O quando n -t +oo.

;n

8. (Áustria-Polônia.) Seja (an)n 21 uma sequência de reais positivos, tais que ªk+2 =

para todo k seu limite.

~

.Jam + Ja,.,

1. Prove que a sequência é convergente e calcule

9. Sejam n > 1 um inteiro fixado e t 0 , t 1, ... , tn reais também fixados, tais que t 0 + ti + · · · + tn = O. Prove que a sequência (ak)k21, dada por

ak = toVk + kv'k+l + ... + tnvk + n converge para O.

Tópicos de Matemática Elementar 3

92

10. (Romênia.) A sequência (xn)n>l é tal que ,Jxn+l + 2:::; Xn:::; 2, para todo n :2'.: 1. Encontre todos os possíveis valores de x 1986 . 11. (Leningrado.) Seja (an)n::::: 1 uma sequência de números reais tal que, para todos m, n E N, tenhamos

Antonio Caminha M. Neto

93

15. (Turquia.) Seja (an)n:::::i uma sequência de inteiros tal que O < an+l - an < ,Ja;;,, para todo n natural. Dados números reais x e y, com O :::; x < y :::; 1, prove que existem naturais m e n tais que

16. Seja (an)n>l uma sequência de reais positivos. Mostre que a desigualdade

Prove que a sequência é uma PA. 12. (Bulgária.) Para cada n EN, seja

se verifica para infinitos inteiros positivos n.

=

a n

+ 1 (2 2n+l 1

1

n

+ 2 + ... + 2n) 2

2

n

.

Prove que a sequência assim definida é decrescente, conclua sua convergência e calcule o limite correspondente.

3.2

Séries de números reais

Seja (an)n:::::i uma sequência de números reais. Pela série

13. (Romênia.) Sejam k um natural fixado e (an)n::::: 1 a sequência definida por an =

J + J + ··· + v'k, k

k

com exatamente n raízes quadradas. (a) Mostre que (an)n:::::i é convergente. (b) Mostre que, quando k é ímpar, o limite da sequência é um número irracional. (c) Encontre todos os valores naturais de k para os quais o limite da sequência seja um número inteiro. 14. Para cada real positivo a, considere a sequência (an)n>l definida por a 1 = 1 e, para k :2'.: 1 inteiro,

ªk+1

=

!2 (ak + .!!._) . ªk

Prove que a sequência converge para y'a,.

ou simplesmente Ln:::::i an, entendemos a sequência (sn)n::::: 1 , onde sn = a 1 + a2 + · · · + an para n :2'.: 1. O número real Sn é denominado a n-ésirna sorna parcial da série Ln:::::i an, e dizemos que tal série converge para s E IR. se a sequência (sn)n::::: 1 de suas somas parciais converge para s. Nesse caso, dizemos· que s é a sorna da série e escrevemos (3.1) = s.

Lªn n2:1

Em outras palavras, quando escrevermos Lk2:l ak = s, estaremos dizendo que as somas finitas sn = a 1 + a2 + · · · + an se aproximam mais e mais do número real s, à medida que n---+ +oo. É nesse sentido que a igualdade (3.1) deve ser pensada, como um limite. Por vezes, teremos em mãos uma sequência (an)n>O de números reais, em cujo caso a série correspondente será denotada por Ln:::::o ªn·

T !

Tópicos de Matemática Elementar 3

94

Antonio Caminha M. Neto

Deixamos ao leitor a tarefa (imediata) de adaptar as discussões acima e porvir a tal situação. Nosso interesse primordial nesta seção é encontrar critérios que permitam decidir se uma dada série é ou não convergente. Caso não o seja, diremos que se trata de uma série divergente. Vejamos dois exemplos de séries divergentes.

95

A recíproca da proposição acima não é válida, i.e., há séries divergentes Lk>l ak para as quais ak --+ O. O exemplo clássico é o da série harmô~ica, i.e., da série Lk:::,:i f;, cuja divergência é discutida no exemplo a segufr e encontrará uso posterior nestas notas. Exemplo 3.20. Dado n E N, se m é o único natural tal que 2m < n < 2m+1, então n

Exemplo 3.18. As séries Lk:::,:l k e Lk:::,:I (-1 )k são divergentes.

1

(3.2)

I:k2:;+1. k=l

Prova. A primeira série diverge por ter n-ésima soma parcial sn = 1+ 2 + · · · + n = n(ntl), logo divergente. No segundo caso, a n-ésima soma parcial Sn da série é tal que Sn = O, se n for par, e sn = -1, se n for ímpar, de sorte que também é uma sequência divergente. • Dada uma série Ln:::,:l an, referimo-nos a um termo genérico an como o termo geral da série. A proposição a seguir dá uma condição necessária para a convergência de uma série em função de seu termo geral.

Em particular, a série harmônica é divergente.



Prova. Veja que, para todo inteiro k > 1,

1 1 1 1 1 1 + 1 + 2k-l + 2 + ... + 2k > 2k + 2k + ... + 2k

2k-l

2k-l

=

1 2·

vezes

Portanto, 1

n

1

1+2+1:-: j=3

J

Proposição 3.19. Se a série Lk:::,:i ak é convergente, então ak--+ O. Prova. Dado E > O, queremos provar que existe n 0 E N tal que n > no =} lanl < E. Seja l = Lk:::,:I ªk· Pela definição de convergência de uma série, existe n 0 E N tal que

n

2: no=} l(a1 + a2 + · · · + an) - li < ~-



Ir

No que segue mostraremos que, parar > 1 racional, a série Lk:::,:l converge. Para tanto, precisamos examinar a convergência de uma série geométrica, i.e., uma série da forma

Daí, pela desigualdade triangular, temos, para n > n 0 , que

lanl :::; l(a1 + a2 + · · · + an) - li+ ll - (a1 + a2 + · · · + ªn-1)1 E E < +-=E. - 2 2

"\""""

L.tq

k-1

'

k21



para um certo real não nulo q. Nesse sentido, temos o seguinte resultado importante .

T Tópicos de Matemática Elementar 3

96

Proposição 3.21. Dado q E IR\ {O}, a série geométrica Lk2':l qk-l converge se, e só se, O < JqJ < 1. Neste último caso, sua soma é igual a

1

Antonio Caminha M. Neto 1

Mas, como r > 1, temos O < anterior que 1

L

1-q·

Prova. Se JqJ ~ 1 a série geométrica não converge, uma vez que seu termo geral qk-l não converge para O. Suponha agora que O< JqJ < 1, e seja sn = 1 + q + · · · + qn-l. Pela fórmula para a soma dos termos

97

2(r-l)k

2r-l

< 1, e segue da proposição

2r-l

=

2r-l _



k20

Portanto, concluímos que O < sn < 2;~~~ 1 , de sorte que a sequência (sn)n2:1 das somas parciais é realmente limitada. li

de uma PG finita, temos 1 - qn 1 qn s = =-----. n 1-q 1-q 1-q

Portanto, para mostrarmos que a série converge para 1 ~q, basta mostrarmos que qn ----+ O quando n----+ +oo, o que fizemos no exemplo li 3.3.

Lk2':l

Jr

converge, qualquer que (3.3)

Prova. Pelo teorema de Bolzano-Weierstrass, basta mostrarmos que a sequência (sn)n2':l das somas parciais Sn = L~=l Jr é limitada. Para tanto, dado n EN, tomem EN tal que 2m > n. Então Sn

'.Ô 1 +

( ;. + :. ) + · · · + ( {2m~l )" + · .. + {2"' ~

< 1+ 2 o a sequência das somas parciais da série do enunciado, i.e., 1 1 1 s =1+-+-+···+n

1!

2!

n!"

Para tal sequência, temos claramente 1 = s 0 < s 1 < s 2 < · · · ; por outro lado, como k! > 2k-l para todo inteiro k > 2, temos, para n ~ 4 inteiro, que

(2(m-l)(r-1)

1 2 (r-l)k ·

k20

já tivéssemos definido potências kr com r > O real (o que só vamos fazer na seção 5.2), o argumento da prova apresentada funcionaria igualmente bem para 3 Se

mostrar que a série

~k2':l

fr

é convergente.

onde utilizamos a fórmula para a soma de uma série geométrica na última igualdade acima. Portanto, a sequência (sn)n>o é monótona e limitada, logo convergente, pelo teorema de Bolzano-Weierstrass.

Tópicos de Matemática Elementar 3

98

Agora, segue do item (b) da proposição 3.6, juntamente com s 2 = 2 e sn < ~~ para todo inteiro n 2: 4, que 2 < e < 3. Mostremos, agora, que e é irracional (de modo que, em particular, e =/=- 3). Para tanto, observe inicialmente que, para naturais 1 < n < m, temos

1 k!
l ak e Lk>l bk são séries convergentes e e é um número real qualquer~ então: -

n+2

+ -n + nn+ ( l) 2 ,

(a) a série Lk2'.l cak converge e Lk>l cak

=

e Lk>l ªk·

T I

i

Tópicos de Matemática Elementar 3

100

Exemplo 3.26. Existe uma sequência (ak)k2:l de números reais positivos tal que ambas as séries Lk>l ak e Lk>l kla k convirjam? -

Prova. (a) Se sn é a n-ésima soma parcial da série Lk2:l ak, então a n-ésima soma parcial da série Lk2:l cak é csn· Portanto, segue do item (a) da proposição 3.9 que

Solução. Suponha que sim. Então, o item (b) da proposição 3.24, juntamente com a desigualdade entre as médias, forneceria

converge e Lk 21 (ak

+ bk) =

Lk2:l ak

Lk21 bk.

~ cak = lim CSn = e lim Sn = e L ªk· L....J n--++oo n--++oo k>l

k>l

(b) Se sn e tn são respectivamente as somas parciais das séries Lk2:l ak e Lk>l bk, então a n-ésima soma parcial da série Lk>l (ak + bk) é sn + t~. Portanto, o item (b) da proposição 3.9 fornece

~(ak+bk) = lim (sn+tn) = lim sn+ lim tn = Lªk+ Lbk. ' I'

L....J k2:l

n--++oo

n--++oo

n--++oo

·

k2:1

k2:1

• Uma rápida análise dos argumentos apresentados nos exemplos 3.22 e 3.23 fornece o seguinte resultado mais geral, conhecido como o critério de comparação para a convergência de séries. i! !

101

+

(b) a série Lk 21 (ak

+ bk)

Antonio Caminha M. Neto

Proposição 3.25. Sejam (ak)k 21 e (bk)k21 sequências de números reais positivos, tais que ak :S bk para todo k ·:2'. 1. Se a série Lk2:l bk convergir, então a série Lk2:l ak também convergirá e

Logo, pelo critério de comparação para séries, a série harmônica seria convergente, o que é um absurdo. • A seguir, discutimos um critério muito útil para a convergência de uma série de números reais positivos com base no comportamento de seus termos, critério este conhecido como o teste da razão. Proposição 3.27. Sejam (an)n>l uma sequência de reais positivos tal 1 -+ l. que Se l < 1, a série Lk2:l ak converge; se l > 1, a série Lk2:l ak diverge.

ª::

Prova. Provemos que, se l < 1, então a série Lk2:l ak converge (a prova da divergência da série no caso l > 1 é análoga). Sendo l < 1, podemos tomar l < q < 1. Por definição, a convergência ªn+i an -+ l garante a existência de n 0 E N tal que

Portanto, para n :2: n 0 , temos k2:1

k2:1

Prova. Se Sn = L~=l ak e tn L~=l bk, temos O < Sn :S tn para todo n E N. Como a sequência (tn)n>l é convergente, ela é limitada. Portanto, a sequência (sn)n2:l é monótona e limitada, logo convergente pelo teorema de Bolzano-Weierstrass. Para o que falta, basta fazer n-+ +oo na desigualdade sn :S tn, utilizando em seguida o resultado • do problema 1, página 90.

Assim, para n :2: n 0 , os termos da série Lk2:l ak são majorados pelos termos da Lk2:l an 0 qn-no, a qual converge, pelas proposições 3.24 e 3.21. Portanto, segue do critério de comparação que a série Lk2:l ak é convergente. •

Tópicos de Matemática Elementar 3

102

Nas notações da proposição anterior, observamos que, se l = 1, a série Lk2:l ak pode convergir ou divergir. De fato, para an = ~ temos

ªk+l ªk mas a série Lk2:l

i

=

_k_ -+ l k +1 '

diverge; por outro lado, para an =

(k

+ 1)2

,;2

temos

-+ 1,

, . '"""" mas a sene L..,k2:l k12 converge. Para séries Lk2:l ak com infinitos termos negativos e infinitos termos positivos, os resultados obtidos até o momento nada dizem acerca de sua convergência. Remediamos esta situação a partir de agora, começando com a seguinte

Definição 3.28. Uma série Lk2:l ak é absolutamente convergente se a série Lk2:l lakl é convergente. A utilidade do conceito de série absolutamente convergente é evidenciada na proposição a seguir e no exemplo subsequente.

Proposição 3.29. Toda série absolutamente convergente é convergente. Prova. Seja Lk2:l ak uma sene absolutamente convergente e, para cada n 2:: 1, sejam Sn = a1 + a2 + · · · + an e tn = la1I + la2I + · · · + lanl· Dados inteiros m > n 2:: 1, temos

lsm - snl

Antonio Caminha M. Neto

103

de sorte que a sequência (sn)n>l também é de Cauchy. Logo, pelo teorema 3.16 a sequência (sn)n2 1 é convergente, conforme queríamos • demonstrar. A recíproca da proposição acima não é válida, quer dizer, há séries convergentes que não são absolutamente convergentes, sendo o exemplo clássico desse fenômeno fornecido pela série Lk2:l (-l~n-i, a qual é convergente (como caso particular do exemplo mais geral a seguir) e tal que a série formada pelos valores absolutos de seus termos (a série harmônica) é divergente. O exemplo a seguir é devido a G. W. Leibniz4, sendo conhecido na literatura como o critério de Leibniz para a convergência de séries alternadas.

Exemplo 3.30 (Leibniz). Se (an)n>l é uma sequência não crescente de reais positivos, tal que an -+ O, então a série Lk2:l (-l)k-lak é convergente. Prova. Para cada n E N, seja Sn = a 1 + a 2 + a1 2:: a2 2:: a3 2:: · · · > O garante facilmente que

· · · + ªn· A condição (3.5)

Por outro lado, para cada m E N, temos

lan+l + an+2 + · · · + aml

< lan+il + lan+2I + · · · + laml Como a sequência (tn)n 21 converge, ela é de Cauchy; portanto, dado E> O, existe n 0 EN tal quem> n >no=} ltm - tnl < E. Com tais E e n 0 , segue da desigualdade acima que

o que claramente garante, em conjunção com (3.5), que a sequência (sn)n21 é de Cauchy. Logo, (sn)n 21 é convergente, conforme desejado . 4 Gottfried



Wilhelm Leibniz, matemático e filósofo do século XVII, é considerado, juntamente com I. Newton, um dos criadores do Cálculo Diferencial e Integral. De fato, algumas das notações que utilizamos até hoje foram criadas já por Leibniz, e resistiram ao teste do tempo.

11 1'

1

1.'

Tópicos de Matemática Elementar 3

104

Problemas - Seção 3.2 , , . L...k """ 2k-1 1. D a do um numero rea1 a > 1, prove que a sene 21 aJc é convergente e calcule sua soma.

2. Se (an)n 21 é uma PA infinita e não constante de termos positivos, prove que: (a) A série Ek 21 ª1k é divergente. !

(b) A série Lk>l - 1 é convergente. -

ª2k

3. (NMC.) Seja A um conjunto finito de naturais da forma 2a3b5c, para algum terno (a, b, e) de inteiros não negativos. Prove que

105

Antonio Caminha M. Neto 6.

* Dada uma sequência (a1, a 2, a 3, ... ) de algarismos, prove que existe um único elemento x E ~ satisfazendo a seguinte condição: fixado um erro máximo 1~n , n E N, temos

para todo natural k 2:: n. 7. Seja (an)n 21 a sequência de números reais positivos definida por a1 =!e an+I = a~+an, para n EN. Prove que a série Lk>l 1+1 ªk converge e 1 """ L..J

li]!

k21

ak

+1=



1

I:- 1, prove que a série Ek 21 ak diverge.

(c) Conclua, a partir de (b), que 2,71828 é uma aproximação de e com cinco casas decimais corretas.

(c) Se l = 1, dê exemplos mostrando que a série Ek 21 ak pode convergir ou divergir.

'r 1

Tópicos de Matemática Elementar 3

106

O critério de convergência de séries dado pelo caso l < 1 é conhecido na literatura como o teste da raiz.

Antonio Caminha M. Neto

15. Seja Tn um triângulo retângulo cujos lados medem 4n 2 , 4n4 -1 e 4n4 + 1, onde n é um número inteiro positivo. Seja an a medida, em radianos, do ângulo oposto ao lado de medida 4n2 . Mostre que

O critério de convergência de séries dado pelo próximo problema é devido ao matemático norueguês do século XIX Niels Henrik Abel, sendo conhecido na literatura como o critério de Abel. 11. Sejam (an)n;::,:l e (bn)n;::,:l duas sequências de números reais satisfazendo as seguintes condições: (a) A sequência (sn)n;::,: 1, definida para n E N por sn · · · + an, é limitada.

=

a1

+

Prove que a série Lk;::,:i akbk converge. 12. Use o critério de Abel para provar que a série Lk;::,:l (-~l é convergente. 13. Mostre que todo O < x < 1 admite uma única expansão decimal da forma x = O, a 1 a 2 a 3 ... , com ak i= O para infinitos valores de k. Em seguida, use este fato para dar um exemplo de função sobrejetora f : (O, 1) --+ (O, 1) x (O, 1). 14. No plano Cartesiano, considere a sequência (An)n;::,:1 de pontos tal que A 1 = (1, O) e satisfazendo as seguintes condições: (i) OAnAn+l é retângulo em An, tal que AnAn+l

=

l.

(ii) OAn+1An+2 tem interior disjunto de OAnAn+l, para todo

n?:_l. ---+

Prove que, à medida que n--+ +oo, a semirreta OAn dá infinitas voltas em torno da origem.

107

3.3

O lema de Kronecker

O resultado principal desta seção poderia ter sido visto já no capítulo 1 do volume 1. Postergamo-lo pelo simples fato de que sua demonstração (bem como as aplicações que dele faremos, aqui e a posteriori) é bem mais refinada que o material constante daquele capítulo introdutório, cabendo melhor aqui. Comecemos recordando a definição a seguir. Definição 3.31. Um subconjunto X de IR é denso (em IR) se, para todo a E IR e toda aproximação E > O dada, tivermos

X

n (a - E, a+ E) i= 0.

Um resultado bastante útil sobre densidade de subconjuntos de IR é o corolário 3.34 a seguir, conhecido na literatura como o lema de Kronecker 5 . A demonstração que apresentamos para o mesmo não é a mais simples possível, mas tem a vantagem de apresentar algumas ideias interessantes em si mesmas 6 . Precisamos primeiro de outra definição. Definição 3.32. Um subconjunto não vazio G de IR é dito um subgrupo aditivo de IR se, para todos x, y E G, tivermos x - y E G. 5 Após

6 Aqui,

Leopold Kronecker, matemático alemão do século XIX. seguimos, essencialmente, [34].

F 1

!

Tópicos de Matemática Elementar 3

108

Evidentemente, {O}, Z, (Q e o próprio lR. são subgrupos aditivos de JR.. Para um exemplo menos óbvio, dados números reais x 1, ... , xk, é imediato verificar (cf. problema 1) que o conjunto

(3.6) é um subgrupo aditivo de lR.. Seja, agora, G um subgrupo aditivo arbitrário de lR. e tome x E G. Pela definição acima, temos O = x - x E G. Segue então daí que, para x, y E G, temos -y = O - y E G e, portanto, x + y = x - (-y) E G. Logo, G é fechado para a operação de adição, e uma fácil indução permite mostrar que {ma:; m E Z}

e

G, V a: E G.

(3.7)

O teorema a seguir fornece a caracterização dos subgrupos aditivos de lR. que nos será útil.

Antonio Caminha M. Neto

109

afirmamos que (m+l)x E Gn(a-E,a+E). De fato, se fosse (m+l)x 2: a+ E, teríamos

mx ::;: a -

E

< a + E ::;: ( m + 1 )x,

de sorte que

x = (m + l)x - mx 2: (a+ E) - (a - E) = 2E, uma contradição à escolha de x. Portanto, (m + l)x E (a - E, a+ E) e, como x, mx E G, temos também (m + l)x = mx + x E G. (b) Se inf(G~) = a: > O, afirmamos inicialmente que a: E G~. Para tanto suponha, por contradição, que a: (/:. G~. Então a definição de ínfimo de um conjunto de números reais garantiria a existência de elementos /3, 1 E G~ tais que a: < /3 < 1 < 20:. Mas, como G é um subgrupo aditivo de JR., seguiria daí que 1 - f3 E G~, com

O< 1 -

/3 < 20: - a: =

O:,

o que, por sua vez, contradiria o fato de que a: = inf(G~). Assim,

Teorema 3.33. Seja G G n JR.~.

# {O} um subgrupo aditivo de lR. e G~

(a) Se inf(G~) = O, então G é denso em JR.. (b) Se inf(G~) =a:> O, então G = Ga. Prova. (a) Suponhamos que inf(G~) = O, e sejam a um real arbitrário e E> O uma aproximação também arbitrária, e provemos que Gn(a-E, a+E) # 0. Como x E G se, e só se, -x E G, basta analisarmos o caso a 2: O. Se a - E < O temos O E G n (a - E, a+ E) e nada mais há a fazer. Suponhamos, pois, que a - E 2: O.

A condição inf(G~) = O garante a existência de x E G~ tal que x < 2E. Sendo m o maior inteiro não negativo tal que mx ::;: a - E,

a: E G~. Tome, agora, x E G~ qualquer e faça q = e r = x - a: de sorte que q E Z+, O ::;: r < a: e x = qa + r. Se r > O, então o fato de G ser um subgrupo aditivo de lR. implica r = x - qa E G~, com O< r < 0:. Mas, como isso contradiz o fato de que a:= inf(G~), concluímos quer= O e, daí, x = qa E Ga. Portanto,

l~J

G~

e {na; n

E

l~J,

N}

e, como a inclusão oposta já foi estabelecida em (3.7), temos mesmo G~

= {na; n EN}.

Finalmente, como G = G~ U {O} U G*:_, com G*:_ = {-x; x E G~}, é imediato que G = {ma:; m E Z} = Gª.



Tópicos de Matemática Elementar 3

110

No corolário a seguir, utilizamos novamente a notação estabelecida em (3.6). Corolário 3.34 (Kronecker). Se a é um número irracional, então o subgrupo aditivo G 1,a = {m + na; m, n E Z} de lR. é denso em R Prova. Por simplicidade de notação, seja G = G1,a· Para provar que G é denso em JR., pelo teorema anterior é suficiente provar que inf(G~) = O. Se este não fosse o caso, novamente pelo teorema anterior existiria um real positivo f3 tal que inf(G~) = f3 > O e G = Gf3. Mas, uma vez que a é irracional e tanto a quanto 1 + a são elementos de G* existiriam inteiros não nulos e distintos m e n tais que +'

a = n/3 e 1 + a = m/3. Logo, por um lado teríamos f3 teríamos i.e., f3

=

= ; (/:. Q,

ao passo que, por outro,

(m - n)/3 = (1 + a) - a= 1, m~n E

Q, o que é um absurdo.



Afirmamos primeiro que existem m, n E Z tais que m < O < n e m + na E A n (O, 8). Por contradição, suponha que

m

seguir são densos em JR.: (a) A= {m + na; m, n E Z em< O< n}.

(b) B={m+na;m,nEZen O, queremos provar a existência de x E A tal que a - E < x < a + E. Suponha que a - E 2 O (os demais casos são análogos) e seja 8 = min{ a, 2E} > O.

+ na E A n (O, 8)

:::} n :::;; O.

Escolha (pelo corolário anterior) x 0 = m 0 + n 0 a E A n (O, 8), com no :S; O o maior possível. Como A n (O, x 0 ) é infinito pelo lema de Kronecker, podemos tomar x 1 = m 1 + n 1a E A n (O, x 0 ), com n 1 < no. Então

xo - x1 = (mo - m1)

+ (no

- n1)a

E

A n (O, x 0 ) e A n (O, 8),

o que é uma contradição, uma vez que n 0 - n 1 > O. Portanto, podemos escolher m + na E A n (O, 8), com n > O. Isto posto se fossem > O ' ' teríamos m + na 2 a 2 8, outra contradição. Logo, m < O e, daí,

A n (O, 8)

#- 0.

Tome, agora, x E A n (O, 8) e considere os números da forma kx, com k E Z+. Sendo k 0 o maior inteiro não negativo para o qual kox :S; a - E, afirmamos que (k 0 + l)x E (a - E, a+ E). De fato, se fosse (ko + l)x 2 a+ E, teríamos

kox :::;; a - E< a+ E:::;; (k 0 + l)x

Nosso próximo corolário refina a conclusão do corolário anterior. Corolário 3.35. Se a é um número irracional, então os conjuntos a

111

Antonio Caminha M. Neto

e, daí,

8> x

=

(ko

+ l)x -

k0 x 2 (a+ E) - (a - E) = 2E,

uma contradição à escolha de 8. Exemplo 3.36 (OBM). Sejam uma função tal que

d(P, Q) = 1

*

7r

um plano Euclidiano e

f : 7f ---+

• 7f

d(f(P), f(Q)) = 1,

para todos P, Q E 7f. Prove que f é uma isometria de 7f, i.e., prove que, para todos P, Q E 7r, tem-se d(P, Q) = d(f(P), f(Q)).

Tópicos de Matemática Elementar 3

112

Prova. Para P E 1r, denotemos J(P) sistematicamente por P', de sorte que d(P, Q) = PQ e d(f(P), J(Q)) = P'Q'. Mostremos primeiro que f deve preservar distâncias -vf:3.

Antonio Caminha M. Neto Afirmação 2. Para todo inteiro positivo n, temos

PQ = n

P'Q' = n.

==}

Basta mostrarmos o caso n = 2, sendo o caso geral totalmente análogo. Sejam P e Q pontos tais que PQ = 2, e R o ponto médio do segmento PQ, tal que PR= RQ = 1 (figura 3.2). Considere pontos

Afirmação 1.

De fato, dados dois pontos P e Q do plano tais que PQ = .J:3, construa pontos R e S tais que os triângulos Q RS e P RS sejam equiláteros de lado 1 (figura 3 .1). Gire o losango P RQ S, com centro em P e no

S

Q

p

u

~: .... 1 1 1 1 1

T

/V\ R

Q

Figura 3.2: PQ = 2

....

s

vl, .... l1'

113

*

P'Q' = 2.

R .... ....

Se T tais que PRS, RST e QRT sejam triângulos equiláteros de lado +------+ 1, situados em um mesmo semiplano dos determinados pela reta PQ . Utilizando a Afirmação 1 duas vezes~, é imediato que P'Q' = 2 .

....

p

Figura 3.1: PQ =

J3 *

Analogamente, podemos provar que P'Q' =

J3.

sentido anti-horário, até obter um losango PTUV para o qual QU

PQ =

= 1.

Observe que as imagens P', R' e S' de P, R e S formam um triângulo equilátero de lado 1. Como Q' R' = Q' S' = 1, segue que Q' = P' ou P' R' Q' S' é um losango congruente a P RQ S (de modo que P'Q' = -vf:3). Para descartar a primeira possibilidade basta observar que, se P' = Q', então T', U' e V' estão todos sobre o círculo de centro P' = Q' e são vértices de um triângulo equilátero de lado 1, o que é um absurdo.

nV3 ==}

P'Q'

=

nV3.

Afirmação 3. P'Q' 2 PQ, para todos os pontos P e Q do plano. A fim de provar essa afirmação, seja PQ = l, tal que l não é nem natural nem da forma n-vf:3, para algum n E N. Pelo corolário 3.35 (veja também o problema 2), podemos tomar sequências (mk)k2'. 1 e (nk)k2'.I de inteiros satisfazendo as seguintes condições: 1.

mk < O < nk, para todo k 2 1;

ii. limk-Hoo(mk

+ nk-vf:3) = l;

iii. max{O, l - 1} < mk

+ nk-vf:3 < l,

para todo k 2 1.

Tópicos de Matemática Elementar 3

114

Vamos mostrar primeiramente que existe um triângulo de lados l, -mk e nn.13. Para tanto, como mk + nk.J3 < l, temos l + (-mk) > nkv'3; também l + (mk + nkv'3) > O, o que implica l + nkv'3 > -mk; finalmente, a partir de l - 1 < mk + nk.J3, temos

Antonio Caminha M. Neto

115

XQ. Mas, como isto vale para todo vértice da triangulação, devemos ter necessariamente P' Q' = l.

X Portanto, a desigualdade triangular garante a existência de um ponto +------+

-

R E 1r \ PQ tal que PR = nk.J3 e RQ = -mk; como já temos PQ = l, nada mais há a fazer. Segue do que fizemos acima que (cf. figura 3.3) P'Q' + R'Q' 2: P'R' ou, ainda, P'Q' 2: P'R' - R'Q'. Mas, como P'R' = nkv'3 e R'Q' = -mk, obtemos P'Q' 2: nk.J3 + mk. Por outro lado, como nk.J3 + mk-+ l quando k-+ +oo, segue daí que P'Q' 2: l = PQ. Figura 3.4: PQ = l

*

P'Q' = l.

R'

R

f nkv'3 p

Q

P'

Problemas - Seção 3.3 1.

Figura 3.3: P'Q' 2: PQ.

Considere novamente pontos P e Q do plano, com PQ +------+



Q'

= l. Com

base na reta PQ e tendo o ponto P como origem, triangule o plano com triângulos equiláteros de lado 1. Pelo que fizemos acima, as imagens por f dos vértices dessa triangulação formam uma triangulação análoga do plano. Por outro lado, se X é um vértice arbitrário da triangulação original (cf. figura 3.4), temos X'Q' 2: XQ, para todo ponto Q do plano. Geometricamente, isto significa que Q' não está no interior do círculo de centro X' e raio

* Dados números reais x 1 , ... , xk, verifique que o conjunto é um subgrupo aditivo de IR.

2.

* Dados a, l E IR,

com a irracional, mostre que existem sequências (mkh?.1 e (nkk::::1 de inteiros satisfazendo as seguintes condições: (a) mk ~e, daí, para todo x E R •

Tópicos de Matemática Elementar 3

126

De posse do lema acima, o lema 4.6 garante a continuidade das funções seno e cosseno, conforme ensina o próximo exemplo. Exemplo 4.10. As funções seno e cosseno são contínuas. De fato, a partir das fórmulas de transformação em produto e do lema anterior, temos para x, y E lR que I

Antonio Caminha M. Neto

O exemplo a seguir mostra um uso típico da regra da cadeia para funções contínuas. Exemplo 4.12. A função f : lR --+ lR dada por f(x) = sen (x 2 ) é contínua. De fato, se g, h : lR--+ lR são as funções dadas por g(x) = senx e h(x) = x 2 , então g eh são contínuas e f = g oh; portanto, f é contínua pela regra da cadeia.

cos x - cos y 1 = 21 sen ( x ; y) 11 sen ( x ; y) 1

y) 1 ~ 21 x ; yI

< 21 sen ( x ;

Problemas - Seção 4.1

lx-yl.

1. Prove que:

Portanto, o lema 4.6 garante a continuidade da função cosseno, e o argumento para a função seno é totalmente análogo.

i

possui (a) A função f : lR \ {O} --+ lR dada por J(x) = a propriedade do valor intermediário e é contínua, mas a função f : lR --+ lR dada por

Terminamos esta seção examinando a continuidade de uma composição. O resultado da proposição a seguir é conhecido como a regra da cadeia para funções contínuas.

IY - Yol < 8::::} lg(y) - g(yo)I < E.

(b) A função

.._,.,-,

..__.,

Y

YO

1

Portanto, segue das relações acima (com y = f(x) em (4.4)) que E X, lx - xol

< 8'::::} lf(x) - J(xo)I < 8::::} lg(f(x)) - g(f(xo))I
O, satisfaz uma desigualdade do tipo (4.3) em cada intervalo [a,b] C (0,+oo). Conclua, a partir daí, que f é contínua.

Mas, a última desigualdade acima acarreta que

f(xo) f(x) - f(xo) > - 2 -,

5. Se I e IR é um intervalo e f : I ---+ IR é uma função contínua, explique por que a função lfl : I---+ IR também é contínua.

o que é o mesmo que f(x) > ô.

. - f( x ) = 6. E xp11que por que a f unçao

Uma aplicação judiciosa do lema acima é o que nos permite provar a propriedade do valor intermediário para funções contínuas definidas num intervalo [a, b]. Começamos com um caso especial da mesma, conhecido como o teorema de Bolzano 3 , para o qual o leitor pode achar conveniente revisar a definição e as propriedades elementares do supremo de um conjunto não vazio e limitado superiormente de números reais (cf. capítulo 1 do volume 1).

3 x4-2x3+1 x 2 +1 ,

7. Justifique a continuidade da função para x > -1, por f(x) = ~-

x E

TI]) m.,

e cont'mua. ,

f : (-1, +oo) ---+ IR dada,

O teorema do valor intermediário

4.2

Nesta seção, mostramos que funções contínuas definidas num intervalo [a, b] satisfazem a propriedade do valor intermediário e apresentamos várias aplicações desse fato. Comecemos enunciando e provando o lema de permanência do sinal para funções contínuas. Lema 4.13. Sejam I e IR um intervalo e f : I ---+ IR uma função contínua. Se x 0 E I é tal que f(xo) > O (resp. f(xo) < O), então existe c5 > O tal que

x

E J,

lx - x0 1< c5 ~

Em particular,

f

f(xo) f(xo) f(x) > - 2 - (resp. f(x) < - 2 -).

ainda é positiva (resp. negativa) em In(xo-ô, x 0 +c5).

Prova. Façamos a prova no caso em que f(x 0 ) > O, sendo a prova no outro caso totalmente análoga. A definição de continuidade garante que, para E = f(;o), existe c5 > O tal que 1

129

Antonio Caminha M. Neto

1

x

E J,

lx - xol < c5 ~

f(xo) lf(x) - f(xo)I O) e, novamente pelo lema de permanência do sinal, existiria O < c5 < b - e tal que f seria negativa em (e - ô, e+ c5) n [a, b]. Mas, como e = sup A, podemos 3 Após

Bernard Bolzano, matemático alemão dos séculos XVIII e XIX.

Tópicos de Matemática Elementar 3

130

tomar d E ( e - ô, e) n A, de sorte que f < O em [a, c1]; portanto, teríamos f < O em [a, d] U (e- ó, e+ = [a, e+ contradizendo o fato de ser e = sup A. Por outro lado, se f (e) > O, existiria ô > O tal que f seria positiva em (e- ô, e+ 6) n [a, b]; em particular, A n (e- ô, e]= 0 e, daí, teríamos sup A e - ô, uma nova contradição. Logo, a única possibilidade é termos f(c) = O. •

!J

!L

s

O teorema a seguir é conhecido como o teorema do valor intermediário. Como é costume na literatura, doravante nos referiremos ao mesmo simplesmente como o TVI.

Antonio Caminha M. Neto

com ao, a1, ... , an E lR, an =/=- O e n ímpar, então a imagem de f é o conjunto de todos os números reais. Em particular, f possui pelo menos uma raiz real. Prova. Dado d E JR, faça g(x) = f (x )-d. Então g : lR--+ lR também é polinomial, e basta garantirmos a existência de e E lR tal que g(c) = O. O argumento do parágrafo anterior reduz nosso problema a mostrar a existência de e E lR tal que f(c) = O. Para tanto, suponhamos, sem perda de generalidade, que an > O. Então, para x =/=- O, repetidas aplicações da desigualdade triangular fornecem

Teorema 4.15 (TVI). Sejam f, g : [a, b] --+ lR funções contínuas. Se f(a) < g(a) e f(b) > g(b) (ou vice-versa), então existe e E (a, b) tal que f(c) = g(c). Em particular, se um real d pertence ao intervalo de extremos f(a) e f(b), então existe e E (a, b) tal que f(c) = d.

i: 1

Prova. Para a primeira afirmação, note que a função h = f - g é contínua e tal que h(a)h(b) = (f (a)-g(a) )(f (b)-g(b)) < O. Portanto, o teorema de Bolzano garante a existência de e E ( a, b) tal que h( e) = O, i.e., tal que J(c) = g(c). O caso particular em questão é obtido fazendo g igual à função constante e igual a d em [a, b]. •

131

f(x)

Portanto, se lxl 2 1, então lxl S lxl 2

s ··· s lxln e, daí,

No que segue, discutimos algumas aplicações interessantes do TVI. Exemplo 4.16. Seja f : [O, 1] --+ [O, 1] uma função contínua. Prove que existe um real O e 1 tal que f(c) = e (i.e., que f tem pelo menos um ponto fixo).

s s

1 la·I. o qual, por sua vez, é positivo para lxl > .l :E~-=an J-0 J Em resumo, se

Prova. Se f(O) = O ou f(l) = 1, nada há a fazer; senão, f(O) > O e f(l) < 1. Considerando a função g : [O, 1] --+ lR dada por g(x) = x, temos, então, f(a) > g(a) e f(b) < g(b), e o TVI garante a existência de O< e< 1 tal que f(c) = g(c) (ou, o que é o mesmo, f(c) = e). • Exemplo 4.1 7. Se

f : lR --+ lR é uma função

polinomial da forma

A > max { 1, al

I:

1

aj

1}

,

n j=O

O para x = ±A. Mas, como n 1mpar, , - f(x) ent ao~> segue que f(-A) < O< J(A) e o TVI garante a existência de e E [-A, A] tal que f(c) =

O.



Tópicos de Matemática Elementar 3

132

Exemplo 4.18 (Romênia). Existe uma função contínua tal que

J(x)

E Q {:}

f(x

f : IR. --r IR.

+ 1) tf_ Q?

f(x + 2) - f(x) = f(x + 2) - f(x + 1) + f(x + 1) - f(x) = 2a, (4.5) também para todo x E R Afirmamos, agora, que existe x 0 E IR. tal que f(x 0 ) E Q; de fato, tome um real qualquer a; se f (a) não for racional, segue de nossas hipóteses que f(a + 1) será racional e basta, então, tomar x 0 = a ou x 0 = a+ 1. Por outro lado, fixado um tal x 0 , nossas hipóteses asseguram que E

Q

*

f(xo

+ 1)

tf_ Q

*

f(xo

+ 2)

Agora, se f(a)

* * * *

J(f(f(x))) = f(x 2 + 1) f(x) 2 + 1 = f(x 2 + 1) f(x)2+1=f(-x)2+1

f(x) = ±f(-x)

= f(/3) = O, então

a 2 + 1 = J(f(a)) = f(O) de modo que

= f(f(/3)) = /3 2 + 1,

f tem no máximo dois zeros. Há três possibilidades:

• f(x) =1- O, para todo real x: pelo TVI, f tem sinal constante em R Mas, como f(x) = ±f(-x) para todo real x, segue que f(x) = J(-x) para todo real x, e

f é par.

• f tem um único zero, digamos em x = a: como f(-a) = ±f(a) = devemos ter a = -a, de modo que a = O. Mas, daí, f(j(O)) = 02 + 1 = 1

*

o,

f(O) =1- O,

o que é uma contradição. E

Q.

Logo, f(x 0 + 2} - f(x 0 ) E Q, o que contradiz (4.5) e termina a demonstração. • Para o exemplo a seguir, recorde que uma função se f(-x) = f(x), para todo real x.

133

Prova. Note inicialmente que, para cada x E IR., temos

f(f(x)) = x 2 + 1

Solução. Suponha que exista uma tal f e defina g : IR. --r IR. por g(x) = f(x+ 1)- f(x). Então, g é contínua (pela regra da cadeia para funções contínuas) e, pela condição do enunciado, transforma todo número real num irracional. Mas, como todo intervalo não degenerado contém números racionais (cf. problema 1.5.2 do volume 1), a fim de não obtermos uma contradição ao TVI a única possibilidade é g ser constante. Assim, existe um número irracional a tal que f (x + 1) f(x) =apara todo x E IR. e, daí,

f(xo)

Antonio Caminha M. Neto

f : IR. --r IR. é par

Exemplo 4.19. Seja f : IR. --r IR. uma função contínua e tal que J(f(x)) = x 2 + 1, para todo x E R Prove que fé par.

• f tem exatamente dois zeros, em x = a ex= -a, para algum a> O: pelo TVI, f tem sinal constante no intervalo (-a, a). Considere a função g(x) = f(x) - x. Se f > O em (-a, a), então

g(O) = f(O) - O> O e g(a) = f(a) - a= -a< O, de modo que existe O< e< a tal que g(c) = O. Se f < O em (-a, a), então g(O) = f(O) - O< O e g(-a) = f(-a) - (-a)= a> O,

Tópicos de Matemática Elementar 3

134

de modo que existe -a< e< O tal que g(c) = O. Em qualquer caso, f admite um ponto fixo e. Mas

f(c) =e==} e= J(f(c)) = c2 + 1 ==} c2

-



Exemplo 4.20. Sejam m, n ~ 1 inteiros fixados. Calcule, para x > O, o número de soluções da equação

x

1

1

são, respectivamente, decrescente e crescente, Como as soluções da equação do enunciado correspondem aos valores positivos de x tais que f(x) = g(x), o problema 1, página 52, garante que tal equação tem, no máximo, uma solução. Para mostrarmos que há de fato alguma solução, vamos usar o TVI, observando inicialmente que f e g são claramente contínuas em (O, +oo ). Para tanto, consideremos dois casos separadamente: • para x > 1, temos 1

Portanto, f(x) < g(x) para

• para O < x < 1, temos 1

1

·

1

1

-xm > - > · · · > -x2 > -X e xm-1

X

< Jx < · · ·
g(a) e f(b) < g(b), de maneira que o TVI garante a existência de e E (a, b) tal que f(c) = g(c). • Terminamos esta seção utilizando o TVI para estudar a continuidade da inversa de uma função contínua cujo domínio é um intervalo.

1

J(x)=-+-+-+···+e g(x)=x+Jx+Tx+···+v'x X X2 x3 xm

1

se ~ < ny'x, i.e., se x > (~) n~i.

m nG · se -;;> nyx, 1.e., se x
Oe k inteiro positivo, a função x M x\ é decrescente, ao passo que a função x M ijx é crescente. Mas, como uma soma de funções crescentes de mesmo domínio é crescente e uma soma de funções decrescentes de mesmo domínio é decrescente (prove este fato!), concluímos que as funções f, g : (O, +oo) -+ IR tais que 1

de maneira que f(x) < ~ e g(x) > ny'x. Em particular, f(x) < g(x)

de maneira que f(x) > ~ e g(x) < ny'x. Em particular, f(x) > g(x)

3 -1 + -12 + -13 + ... + - 1 = X+ Jx + \7X + ... + y'x.

x

135

x > max { 1, (~) n~l } ·

e+ 1 = O,

o que é um absurdo, haja vista tal equação não possuir raízes reais.

x

Antonio Caminha M. Neto

1

1

-xm < - < · · · < -x2 < -X e xm-1

X

> Jx > · · · >

n-VX >

y'x,

Teorema 4.21. Se I e IR é um intervalo (resp. aberto, fechado ou semiaberto) e f : I -+ IR é uma função contínua, então f é injetiva se, e somente se, f é crescente ou decrescente. Ademais, em um qualquer desses casos temos que: (a) a imagem de fé um intervalo J (resp. aberto, fechado ou semiaberto); (b)

J- 1 : J

-+ I é contínua.

Prova. Se f não é injetiva, então f claramente não pode ser nem crescente, nem decrescente. Reciprocamente, se f não é nem crescente, nem decrescente, então existem a< b < e em I tais que f(a) ::::; f(b) ~

ri: •

1

Tópicos de Matemática Elementar 3

136

i

f(c) ou f(a) 2:: f(b) ::; f(c). Suponha que f(a) ::; f(b) 2:: f(c) (o outro ' caso é análogo) e escolha d E R tal que

max{f(a), f(c)} ::; d::; f(b). O TVI garante a existência de x 0 E (a, b) e x 1 E (b, e) (logo x 0 , x 1 E J) tais que f(xo) = d e f(x1) = d, de modo que f(xo) = f(x1). Em particular, f não é injetiva. Para o item (a), suponha f crescente e I = (a, b), com a, b E R (os demais casos são totalmente análogos). Para cada intervalo [e, d] e (a, b), o TVI e o fato de f ser crescente garantem que a imagem por f do intervalo [e, d] é o intervalo [f(c), f(d)]. Mas, como

(a,b) i,

=

LJ [a+ ~,b- ~],

Antonio Caminha M. Neto

137

Observe que podemos supor E > O tão pequeno que x 0 ± E E ( a, b) (senão, diminua o E > O dado, de forma que essa condição seja satisfeita). Lembrando que fé crescente, tome

O< ô< min{ ôo, f(xo

+ E) -

f(xo), f(xo) - f(xo - E)}.

Então,

. f(x) - f(xo) < ô :::} f(x) - f(xo) < f(xo + E) - f(xo) :::} f(x) < f(xo + E) :::} x < Xo + E e, analogamente,

f(x) - f(xo) > -8:::} x > xo -

E.

n21

1i

Em qualquer caso, a escolha acima para ô garante que

é imediato verificar que

lf(x) - f(xo)I < ô :::} -8 < f(x) - f(xo) < ô :::} -E < X - Xo < E onde e= inf{f(x); x E (a, b)} e d= sup{f(x); x E (a, b)}. Mostremos por fim o item (b), supondo ainda que f : (a, b) -+ (e, d) é crescente. Então (cf. problema 5, página 43), J- 1 : (e, d)-+ (a, b) é crescente. Agora, fixado y0 E (e, d), seja xo = J- 1(yo). Dado E > O, queremos ô > O tal que

Y E (e, d), IY - Yol 1 inteiro e a 1 , a 2 , ... , an reais positivos dados. Prove que a equação

possui exatamente uma solução real positiva. 6. Sejam x 1 , x 2 , ... , Xn reais escolhidos do intervalo [O, l]. Prove que existe x E [O, 1] tal que

Tópicos de Matemática Elementar 3

140

7. (Leningrado.) Seja f : IR ---+ IR uma função contínua satisfazendo, para todo x E IR, a relação f(x)f(x + 2) + f(x + 1) = O. Mostre que existem infinitos valores reais de x tais que f (x) = O. 8. (Leningrado.) Seja f : IR ---+ IR uma função contínua tal que f(x)f(f(x)) = 1, para todo real x. Se !(1000) = 999, calcule !(500). 9. (Austrália.) Encontre todos os a E IR tais que, para toda função contínua f : [O, 1] ---+ IR satisfazendo a condição f(O) = f(l), exista x 0 E [O, 1 - a] para o qual f(x 0 ) = f(x 0 + a).

12.

16. (Torneio das Cidades.) Prove que, para cada natural n, o gráfico de qualquer função contínua e crescente f : [O, 1] ---+ [O, 1] pode ser coberto por n retângulos, cada um dos quais de área -\ e n lados paralelos aos eixos coordenados.

+ 1) + 1 = O,

f : [O, 1] ---+ [O, 1] tais que

18. (Bielorrússia.) Encontre todas as funções f, g, h : IR ---+ IR tais que, para todos x, y E IR, tenhamos

J(x + y3 ) + g(x 3 + y) = h(xy).

* Sejam dados um intervalo I

e um subconjunto não vazio J de I, tendo as seguintes propriedades: (a) Para todo x 0 E J, existe ô> Otal que (x-ô, x 0 +'5)nJ

e

I.

(b) Se (an)n2:1 é uma sequência de pontos de J e l E I é tal que an ---+ l, então l E J. Mostre que J

13.

15. (Crux.) Seja f : IR ---+ IR uma função contínua que assume valores positivos e negativos. Dado k > 2 natural, prove que existem reais a 1 , a 2 , ... , ak em progressão aritmética e tais que

17. (Leningrado.) Seja f : IR ---+ IR uma função contínua tal que, para todo real x, tenhamos f(x + f(x)) = f(x). Prove que fé constante.

10. Seja f: IR---+ IR uma função tal que J(x + l)J(f(x) para todo real x. Prove que f não é contínua. 11. Encontre todas as funções contínuas (! o J)(x) = x, para todo x E [O, l].

=

I.

* Se f:

[O, 1] ---+ IR é uma função contínua, tal que f(r) 2:: O para todo racional diádico r E [O, 1], prove que f(x) 2:: O, para todo X E [0,1].

14. Se a é um irracional dado, encontre todas as funções contínuas f : IR ---+ IR tais que

J(x) para todo x E R

=

141

Antonio Caminha M. Neto

J(x + 1) = J(x + a),

4.3

Continuidade sequencial

Nesta seção, relacionamos os conceitos de limite de uma sequência convergente e continuidade de uma função. O resultado principal é a proposição a seguir, a qual será utilizada diversas vezes no que segue. Em palavras, ela assegura que funções contínuas são caracterizadas pela propriedade de transformarem sequências convergentes em sequências convergentes. Em tudo o que segue, I denota um intervalo da reta.

Proposição 4.24. Uma função f : I ---+ IR é contínua se, e só se, a seguinte condição for satisfeita: para todo a E I e toda sequência (an)n2:1 de elementos de I, temos lim an =a:::} lim f(an)

n~+=

n~+=

=

f(a).

Tópicos de Matemática Elementar 3

142

Prova. Inicialmente, suponha que existe ó > O tal que

x

E J,

f

é contínua. Então, dado

)x - a) O,

E.

Seja, agora, (an)n2'.l uma sequência de elementos de I, convergindo para a E J. Como limn---Hoo an = a, existe no EN tal que n > no

=}

)an - a) < ó.

Logo, a conjunção das duas condições acima garante que

ai

no=} lan -

Antonio Caminha M. Neto

E,

o que é o mesmo que dizer que limn-Hoo f(an) = f(a). Reciprocamente, suponha que f não é contínua em a E J. Então, a negação da definição de continuidade garante a existência de E > O tal que, para todo ó > O, temos )f(x) - f(a)) 2 E para algum x satisfazendo )x - ai < ó. Em particular, tomando n E N e ó = ;, verificamos a existência de an E I tal que

E/

Exemplo 4.26 (Suécia). Ache todas as funções contínuas f: lR-+ lR tais que f(x) + f(x 2) = O, para todo x E R Solução. Seja f uma função qualquer satisfazendo as condições do enunciado. Fazendo respectivamente x = O ex = 1, obtemos f(O) = f(l) = O. Por outro lado, uma fácil indução garante que, para x > O en EN,

/f ( vx) / = lf(x)I. 2

lan -

1

ai < -n e lf(an) -

f(a)) 2

E.

Em particular, segue das condições acima que a sequência (an)n2'.1 assim construída converge para a, ao passo que a sequência (f(an))n2'.1 não converge para f (a). • Ilustramos a importância da proposição acima em dois exemplos, O primeiro dos quais fornece outra prova para o resultado do exemplo 3.12.

Exemplo 4.25. Se a > O e an =

via, então an ~ 1.

1

1:

Prova. Façamos a prova no caso em que a 2 1, sendo a prova para o caso O < a < 1 totalmente análoga. Se a 2 1, então an 2 ªn+1 2 1,

Fixe um real positivo X e seja ªn a função lfl também é contínua e an anterior, segue da proposição 4.24 que

= =

,vx para n E N_ Como ,vx ~ 1 pelo exemplo

2

2

IJ(an)) ~ )f(l)j = O. Por outro lado, )f(an)) = IJ(x)) para todo n garante que IJ(an)) ~ IJ(x)), de maneira que lf(x)I = O, pela unicidade do limite de sequências. Finalmente, dado x > O, temos

O= f(-x)

+ f((-x)2) = f(-x) + f(x 2) = f(x),

uma vez que x 2 > O =} f(x 2) = O. Logo, a única função satisfazendo as condições dadas é a função identicamente nula. •

Tópicos de Matemática Elementar 3

144

A proposição acima permite provar facilmente as afirmações feitas no parágrafo sucedâneo à definição 4.2. Antes, contudo, vale observarmos o seguinte: se g : I --t lR. é uma função contínua em x 0 E I e g(x 0 ) -=/= O, o lema 4.13 garante a existência de O tal que a função g não se anula no intervalo J = I n (x 0 - n : 1 . i

I' 11

_n_ < log n+l ou, ainda,

(1 + !)n 1
O, com igualdade se, e só se, x = y, temos

Tópicos de Matemática Elementar 3

180

Antonio Caminha M. Neto

• cos (x;y) = 1: a condição x, y E [O, 7r] garante que -~ :::; x? :::; ~; nesse intervalo, temos cos (x;y) = 1 se, e só se, x? = O, i.e., se, e só se, x = y.

Por outro lado,

{::} (xn + yn)(x + y) :::; 2(xn+l + yn+l) {::} xny

1

1

1

;'

~

O,

o que é verdade; ademais, uma rápida inspeção das desigualdades acima garante que há igualdade se, e só se, x = y. Exemplo 5.18. A função log : (O, +oo) --+ IR é estritamente côncava. ~ yfxfj; mas como log é De fato, para todos x, y > O, temos crescente, segue que

x;y

1

1

i

• sen (x+y) = O· segue de x+y E [O 7r] que x+y = O ou 7r· mas 2 ' 2 ' 2 ' ' como x, y E [O, 7r], isso é o mesmo que pedir que x = y = O ou X =y = 7!'.

+ xyn :::; Xn+l + yn+l

{::} (x - y)(xn - yn) 1

I

181

y)

x+ log ( - 2 -

~

Exemplo 5.20. A função tangente é estritamente convexa no intervalo [O,~). De fato, para x, y E [O,~) não ambos nulos, as fórmulas de adição de arcos fornecem t g

;-; :; ; log x + log y, logyxy =

(x+y) < tgx+ tgy 2 2

2

e claramente a igualdade ocorre se, e só se, x

COS

= y.

=

2 sen ( x ; y) cos ( x ; y) .

(x;y)

~

< sen (~) cos (~)

(X?) -

{::} 2 COS X

y: :; cos

COS

2 COS X COS Y 2 (

x;

y :::; 1 + COS ( X

y) + y)

(x?) =

{::} cos(x - y) :::; 1, o que é sempre verdade. Por outro lado, os cálculos acima também mostram que

y) =

X+ tg ( - 2 -

tg X+ tg 2

y{::} cos(x -

y) = 1 {::} x = y,

de sorte que a função é estritamente convexa no intervalo em questão.

Y) cos ( x ; Y) : :; 2 sen ( x ; Y) ,

ocorrendo a igualdade se, e só se, cos (x?) = 1 ou sen Analisemos essas duas possibilidades separadamente:

2 COS X COS y

{::} cos x cos y :::; 1 - sen xsen y

A condição x, y E [O, 7r] garante que x? E [O, 7r] e, daí, que sen O; mas, como sempre temos cos (x;y) :::; 1, segue que 2 sen ( x ;

< sen(x+y)

x;y) -

{::} cos x cos

y: :; 2 sen ( x ; y) ,

com igualdade se, e só se, x = y. Para tanto, transformando o primeiro membro em produto, obtemos sen x + sen y

COS (

{::} sen (~)

Exemplo 5.19. A função seno é estritamente côncava no intervalo [O, 7r]. De fato, para O:::; x, y:::; 7l', é suficiente mostrarmos que sen x + sen

{::} sen (~)

O.

A continuação, provamos uma versão da proposição 5.13 para funções estritamente convexas e estritamente côncavas, a qual encontrará utilidade na próxima seção.

Tópicos de Matemática Elementar 3

182

Proposição 5.21. Uma função contínua f : I ----+ IR é estritamente convexa (resp. estritamente côncava) se, e só se,

J((l - t)x + ty)) < (> )(1 - t)f(x)

+ tf(y),

Antonio Caminha M. Neto

183

Problemas - Seção 5.3 1. Sejam I, J e IR intervalos e

f :I

----+ J uma bijeção contínua. Se fé convexa (resp. estritamente convexa), prove que 1- 1 : J----+ I

(5.13)

é côncava (resp. estritamente côncava), e vice-versa. para todos x, y E I distintos e todo t E (O, 1).

11

2.

Prova. Examinemos a convexidade estrita de f (a análise do outro caso sendo totalmente análoga), observando inicialmente que se (5.13) for válida para todos x, y E I e todo t E (O, 1), então (tomando t = f é estritamente convexa. Reciprocamente, suponha que fé estritamente convexa e fixe x, y E I distintos e O < t < 1. Se z = (1 - t)x + ty, então x < z < y, de modo que podemos escolher a E (x, z) e b E (z, y) tais que z = Ademais, sendo a= (l-s)x+sy e b = (l-u)x+uy, com s, u E (O, 1), segue de z = que t = Portanto, a convexidade estrita de f, juntamente com a proposição 5.13, garante que

convexas (resp. estritamente côncavas) e de mesmo domínio também é uma função estritamente convexa (resp. estritamente côncava).

!)

ª!b·

ª!b

ª!u.

J((l - t)x + ty) = f(z) = f (a; b) < f(a); f(b)

* Prove que a soma de um número finito de funções estritamente

3. Esboce o gráfico da função por f (x) = x + ~. 4.

* Prove que a função

f : IR\ {O}

----+ IR, dada para x

f : (O, 1) ----+ IR, tal que f(x)

=

v1f=x

-=/=-

O

para

x E (O, 1), é estritamente convexa. 5. Sejam I, J e IR intervalos e g : I ----+ J, f : J ----+ IR funções estritamente convexas (resp. estritamente côncavas). Se f for crescente, prove que f o g : I ----+ IR também é estritamente convexa (resp. estritamente côncava).

1

= 2[J((l - s)x + sy) + J((l - u)x + uy)]

6.

1

:S: 2[((1 - s)f(x) + sf(y)) +((1 - u)f(x) + uf(y))] =

J(x) ( 1 - -s+u) 2-

+ (s+u) - 2- f(y)

= (1 - t)J(x) + tf(y).



* Prove que

a função f : (O, 7r) ----+ IR, dada para x E (O, 7r) por f(x) = logsenx, é estritamente côncava.

7. Seja f : (a, b) ----+ IR uma função contínua, não negativa e crescente (resp. decrescente). Prove que toda integral indefinida de f (cf. definição 5.4) é estritamente convexa (resp. estritamente côncava) . 8. Use o resultado do problema anterior para provar que: (a) a função logaritmo natural é estritamente côncava. (b) para n > 1 inteiro, a função x H- xn, x 2 O, é estritamente convexa.

Tópicos de Matemática Elementar 3

184

9. Se f : (O, +oo) --"? lR. é uma função contínua, crescente (resp. não decrescente) e convexa (resp. estritamente convexa), prove que a função g: (O, +oo)--"? JR., dada para x >Opor g(x) = xf(x), também é estritamente convexa.

A desigualdade de Jensen

5.4

Para nós, a importância da discussão sobre funções côncavas e convexas levada a cabo na seção anterior reside no seguinte teorema, conhecido como a desigualdade de J ensen 1 . Teorema 5.22 (Jensen). Sejam I e lR. um intervalo aberto e f : I-? lR. uma função contínua. Se x 1, ... , Xn E I e ti, ... , tn E (O, 1), com t 1 + · · · + tn = 1, então t1X1 + · · · + tnXn E J. Ademais: (a) Se

f

for convexa, então

Antonio Caminha M. Neto

185

Suponha agora que, para um certo n > 1 e todos x 1 , ... , Xn E I e t 1, ... , tn E (O, 1), com t1 + · · ·+tn = 1, tenhamos t1X1 + · · ·+tnXn E I e com igualdade se, e só se, x 1 = · · · = Xn· Consideremos elementos x 1 , ... 'Xn, Xn+1 E I e ti, ... ' tn, tn+l E (O, 1) tais que t1 + ... +tn +tn+I = 1. Defina t1X1 + · · · + tnXn y= l = S1X1 + · · · + SnXn, -tn+l com Sj = i-t+i, para 1 :S: j :S: n. Como s1 + · · · + sn = 1 e Sj E (O, 1) para 1 :S: j :S: n, segue da hipótese de indução que y E J. Daí,

t1X1 + · · · + tnXn + tn+1Xn+1 = (1 - tn+1)Y + tn+1Xn+1 e a convexidade estrita de

E

J

f

f(t1X1 + · · · + tn+1Xn+1)

f((l - tn+I)Y + tn+1Xn+1)

< (1 - tn+1)f(y) + tn+if(xn+I), ocorrendo a igualdade no caso em que se, e só se, X1 = · · · = Xn·

f

é estritamente convexa

com igualdade se, e só se, y = Xn+1 · Aplicando agora a outra metade da hipótese de indução, obtemos

f(y) (b) Se

f

for côncava, então

+ · · · + SnXn) < sif(x1) + · · · + snf(xn) f(s1X1

l

f

1

t

-

n+l

(tif(x1)

+ · · · + tnf(xn)),

é estritamente côncava

com igualdade se, e só se, x 1 = · · · = XnJuntando as duas desigualdades acima, concluímos que

Prova. Suponhamos f estritamente convexa (os demais casos são totalmente análogos) e façamos a prova por indução sobre n > 1. O caso n = 2 segue da hipótese de convexidade estrita e da proposição 5.21.

f(t1X1 + · · · + tn+1Xn+1) < (1 - tn+1)f(y) + tn+IÍ(Xn+I)

ocorrendo a igualdade no caso em que se, e SÓ se, X1 = · · · = Xn·

1 Após

Jensen.

o matemático e engenheiro dinamarquês dos séculos XIX e XX Johan

< (tif(x1) + · · · + tnf(xn)) + tn+IÍ(Xn+1), ocorrendo a igualdade se, e só se, y = Xn+1 e x 1 = · · · = Xn· Mas é imediato verificar que tais condições equivalem a x 1 = · · · = Xn = Xn+1, conforme desejado. •

Tópicos de Matemática Elementar 3

186

Os exemplos a seguir elencam algumas aplicações da desigualdade de Jensen.

Exemplo 5.23 (BMO). Seja n > 1 e a 1 , ... , an reais positivos com soma igual a 1. Para cada 1 :::; i :::; n, defina bi = a 1 + · · · + ªi-l + ai+l + · · · + an. Prove que a1 ª2 an n --+--+ .. ·+ >-1 + b1 1 + b2 1 + bn - 2n - 1 '

com igualdade se, e só se, a1

Prova. Substituindo bi

=

= a2 = · · · = an =

Antonio Caminha M. Neto

187

Exemplo 5.24. Sejam dados, no plano, um semicírculo r de raio R e um diâmetro A 0 A 1 de r. Para cada inteiro n > 2, mostre que existe um único n-ágono A 0 A 1 A 2 ... An-l satisfazendo as seguintes condições:

(a) A2, ... , An-1 E

r.

(b) A área de A 0 À 1 A 2 .•• An-l é a maior possível.

Solução. Considere a figura 5. 7 como representativa da situação do problema, e seja AiÔAi+l = ai, para 1 :::; i :::; n - 1 (com An = A0 ). Então a 1 + a 2 + · · ·+ ªn-l = 1r e a fórmula do seno para a área de um triângulo fornece

i.

1 - ai para 1 :::; i :::; n, basta provar que



Para tanto, afirmamos inicialmente que a função f : (-oo, 2) ---+ lR dada por f (x) = 2 _:x é estritamente convexa. De fato, como

f(x)

= -1

2

+2-x -,

a convexidade estrita de f é uma consequência imediata da convexidade estrita da função de proporcionalidade inversa e do problema 12, página 69. Portanto, aplicando a desigualdade de Jensen, obtemos

(l

(1)

n n ) Lf(ai) ~nf -2:ai =nf i=l n i=l n

ocorrendo a igualdade se, e só se, a 1 1 e so, se, ª1 = ª2 = · · · = an = -. n

=

Ao

2 , 2n - 1

= a 2 = · · · = an

o

Figura 5.7: polígono de área máxima inscrito em um semicírculo.

e, portanto, se, •

Nosso próximo exemplo utiliza a desigualdade de Jensen para resolver um interessante problema de geometria.

Agora, uma vez que a função seno é estritamente côncava no intervalo [O, 1r], segue da desigualdade de Jensen que n-l

I:senai:::; (n - l)sen i=l

=

7r

(n - l)sen - - ,

n-l

Tópicos de Matemática Elementar 3

188

com igualdade se, e só se, a 1 = · · · = ªn-l = n:_ 1. Logo, há um único polígono de área máxima satisfazendo as condições do enunciado. •

li

Antonio Caminha M. Neto Lema 5.26 (Young). Sejam p e q reais positivos tais que .!.p Dados a, b > O, temos aP bq ab S - + -,

(5.15)

Exemplo 5.25. Demonstre a desigualdade entre as médias aritmética e geométrica, para n > 2 reais positivos, como corolário da desigualdade de Jensen.

Prova. Já sabemos que a função logaritmo natural é estritamente côncava em (O, +oo). Portanto, dados reais positivos x 1 e x 2 , e t 1, t 2 > O tais que t1 + t2 = 1, temos pela desigualdade de Jensen que

Prova. Dados n > 2 reais positivos a 1, a2 , ... , an, a sobrejetividade da função logaritmo natural garante a existência de números reais x 1, x 2 , ... , Xn tais que aj = logxj para 1 S j S n. De acordo com o exemplo 5.18, a função log : (O, +oo) --+ IR é estritamente côncava. Portanto, temos pela desigualdade de Jensen que

+ log x~+

· · · + log Xn S log ( X1 +

X2 :

···

+

log(t1x1

= bº.

com igualdade se, e só se, x 1 = x 2 • Fazendo t 1 = .!., t 2 = .!., x 1 = aP e p q X2 = bº, obtemos log (ªP p

+ bº) 2: q

~p log aP + ~q log bº = log ab '

com igualdade se, e só se, aP = bº. Por fim, como a função logaritmo natural é crescente, segue da desigualdade acima que

Xn)

bq

-p +-q 2: ab, com igualdade se, e só se, aP (5.14)

com igualdade se, e só se, x 1 = x 2 = · · · = XnPor fim, como log é uma função crescente, a desigualdade entre as médias segue prontamente de (5.14).



Podemos obter outras desigualdades interessantes refinando o uso da desigualdade de Jensen em conexão com a função logaritmo natural. Em particular, a desigualdade (5.15) a seguir é conhecida como a desigualdade de Young 2 .

!

q

+ t2x2) 2: t1 log X1 + t 2 log x 2 ,

ou, ainda,

1

= 1.

com igualdade se, e só se, aP

p

aP

1

+ .!.q

O exemplo a seguir utiliza a desigualdade de Jensen para dar outra prova da desigualdade entre as médias aritmética e geométrica.

log X1

I •

189

= bº.

A desigualdade da proposição a seguir é uma consequência da desigualdade de Young, sendo conhecida na literatura como a desigualdade de Hõlder 3 . Observe que, quando p = q = 2, tal desigualdade se reduz à desigualdade de Cauchy (7.13) do volume 1. Proposição 5.27 (Holder). Sejam a 1, a2 , •.. , an e b1, b2 , ... , bn reais positivos dados, e p, q > O tais que .!.p + .!.q = 1. Então

n ( n ) l/p ( n ) l/q ~aibi S ~af · ~bf , --::--~~~~~~~~~

2 Após

William H. Young, matemático inglês dos séculos XIX e XX.



3 Após

Otto Hõlder, matemático alemão dos séculos XIX e XX.

Tópicos de Matemática Elementar 3

190

Problemas - Seção 5.4

com igualdade se, e só se,

aPn

1. (Romênia.) Sejam I e R um intervalo e f : I ---+ R uma função estritamente convexa e crescente. Prove que a sequência (f (n) )n~l não contém uma PA infinita.

bK.

Prova. Fazendo A= CE:= 1 af) 11P e B = (I:::= 1 b{) 11 q, temos

2. Dados n

n n a· b· ~ < AB {::} ~ ....!:. • .....: L.....t a·b· iiL.....tA B 1 inteiro e a 1 , a2 , ..• , an reais positivos, prove que

xf = y'f

com igualdade se, e só se,

1.

a1

= a2 = · · · = ªn·

q i=l

para 1


O dado (o :ue é o mesmo que dar a vizinhança J)' deve existir 8 > O (o que e o mesmo que existir a vizinhança I) tal que

x

E

Xo ·

(a) limx--+ 2 (-2x+7) = 3: seja dado E> O. Partindo de x E IR sujeito a um erro do tipo O< lx - 21 < ô, temos

l(-2x + 7) - 31 = 1- 2x + 41 = 2lx - 21 < 25.

(5 ,5)

f(x)EJ

Veja a figura 6.2. ( E m pa1avras, (6 .6) Ocorre quando ' fixado um erro E > O .para- O valor de) L existir um erro 8 > O para Xo tal que 8-aprox1ma?oes · - J(x ) de L . Amda. --1de x ' em X correspondam a E-aprox1maçoes X I Xo O j( ) ' 1 de outro modo, (6.6) ocorre quando pudermos tornar x tao proxmo de L quanto desejado, bastando para isso tomarmos x-# X\ {xo} suficientemente próximo de

Exemplos 6.3.

Escolhendo então ô > O tal que 25

X, e O< \x - xo\ < 8 =} \f(x) - L\ O para o candidato L a limite, temos de ser capazes de encontrar um erro 8 > O para x 0 ( o qual, em geral, dependerá tanto do E dado quanto do próprio x 0 ) de modo que a validade da condição O < lx - x 0 1 < 8 para um elemento x E X acarrete a validade da condição IJ(x) - LI < E. Vejamos, em alguns exemplos, como implementar a estratégia acima. Em tudo o que segue, por vezes omitiremos quaisquer referências explícitas ao domínio e/ou contradomínio das funções envolvidas, concentrando-nos em suas expressões em termos da variável independente (x, em geral). Sempre que tal ocorrer, convencionamos que o domínio da função é seu domínio maximal de definição (cf. seção 1.1), e o contradomínio é o conjunto dos números reais.

y

Xo

Antonio Caminha M. Neto

X

E IR,

::=::;

E, temos que

e o< lx - 21 O. Partindo de x E IR sujeito a um erro do tipo O < lx - 31 < ô, temos

IX2

-

91

1X - 3II X < 5(lx - 31

+ 3I < ÔIX - 3 + 6 I + 6) < 5(5 + 6),

Tópicos de Matemática Elementar 3

200

onde utilizamos a desigualdade triangular na penúltima passagem acima. Portanto, caso seja possível escolhermos O de tal forma que O tal que a função g não se anula no conjunto Y = (X\ {x 0 }) n (Xo - 60, Xo + 60). Portanto, ao considerarmos a função i, tal que i(x) = g(x)' sempre suporemos implicitamente que seu domínio é o conjunto Y.

203

(pela desigualdade triangular), a fim de que 1(f + g) (x) - (L + M) 1 < E para x E X próximo a (mas diferente de) x 0 , é suficiente que tenhamos IJ(x)-LI < ! e lg(x)-MI < f Mas, como! > O e limx--+xo f(x) = L e limx--+xo g( x) = M, a definição de limite garante a existência de reais positivos 61 e 62 tais que

x

E

X e O< lx - xol < 61 ~ lf(x) - LI
IMI para x E y 2 temos

Portanto, a fim de que \(f g)(x) - (LM)\ < E para x E X próximo a (mas diferente de) x 0 , é suficiente que tenhamos cada uma das parcelas \J(x)-L\\g(x)-M\, \L\\g(x)-M\ e \M\\J(x)-L\ menor que!· Para tanto, basta que tenhamos, por exemplo,

L\, \g(x) - M\
O, existe ô > O (dependendo de

E) tal que x E X e O< lx - xol O tal que f(xo) 2 f(x) (resp. f(x 0 ) ::::; f(x)), para todo x E (xo - E, x 0 + E) n I.

Genericamente, um ponto de máximo ou mínimo local para uma função f : I ---+ IR é denominado um extremo local de f. Se J for um intervalo aberto e f : I ---+ IR for derivável, mostraremos a seguir que os extremos locais de f .são zeros da derivada f' de f. Entretanto, uma vez que tais zeros desempenham papel preponderante na discussão subsequente, antes de apresentarmos a prova desse resultado introduzimos uma nomenclatura relevante. Definição 6.37. Se I e IR é um intervalo aberto e f : I---+ IR é uma função derivável, dizemos que x 0 E I é um ponto crítico de f se f'(xo) = O.

Podemos, agora, enunciar e provar o seguinte resultado fundamental, conhecido como o teste da primeira derivada para extremos locais. Proposição 6.38. Se I e IR é um intervalo aberto e f : I ---+ IR é uma função derivável, então todo extremo local de f também é ponto crítico. Prova. Analisemos o caso em que x 0 E I é um ponto de mínimo local para f, sendo a prova no outro caso totalmente análoga.

Tópicos de Matemática Elementar 3

232 Tome i5 > O tal que (x 0 O


(b) Se

f

O no interior de 1, então

é crescente em 1, mas a

recíproca não é válida. é não crescente em 1.

O no interior de 1, então

f

é decrescente em 1, mas a

Agora, vejamos como utilizar os resultados acima para estudar máximos e mínimos de funções.

Prova. Provemos somente os itens (a) e (b), sendo a prova dos itens

Exemplo 6.44. Se f : [O, +oo) -+ IR é a função dada por f (x) = x:.:l, mostre que f atinge um valor mínimo em [O, +oo) e calcule tal valor.

f'
O tal que (x-8,x+ô) C 1. Se O< h < ô, então x + h E 1, de sorte que f(x + h) - f(x) ;:::: O. Portanto, f(x+hz-f(x) ;:::: O e, fazendo h-+ O+, obtemos

. f(x f '( X ) = 1Im h--+0+

+ h)h - f(x) >_ O.

Reciprocamente, suponha que f' ;:::: O no interior de 1. Se a, b E 1 são tais que a < b, a continuidade de f em 1 garante sua continuidade também em [a, b]. Por outro lado, como (a, b) está contido no interior de 1, temos f derivável em (a, b), de sorte que o TVM garante a existência de e E ( a, b) tal que

f(b) - f(a) = f'(c) ;:::: O. b-a Logo, f(b) ;:::: f(a), e a arbitrariedade da escolha de a< bem 1 garante que

f

é não decrescente em 1.

i

1

i 1

de modo que f(b) > f(a). Logo, fé crescente em 1. Para ver que a recíproca não é verdadeira, considere f : IR -+ IR dada por f (x) = x 3 • Ela é crescente em toda a reta, mas f' (O) = O. •

f

(d) Se

I

237

e só se,

(c)

f' :S O no interior de 1 se,

Antonio Caminha M. Neto

Solução. Se f atinge um valor mínimo em [O, +oo), o corolário 6.39 garante que tal ocorre em O ou em um ponto crítico de f. Calculando a primeira derivada de f, obtemos

f'(x) = 2x(x + 1) - (x 2 + 3) · 1 = x 2 + 2x - 3 (x + 1) 2 (x + 1) 2 ' de sorte que f' (x) = O se, e só se, x = 1 (lembre-se de que devemos ter x ;:::: O). Por outro lado, como x 2 + 2x - 3 = (x + 3)(x - 1), concluímos que f' é negativa no intervalo (O, 1) e positiva no intervalo (1, +oo). Portanto, a proposição anterior garante que fé decrescente no intervalo [O, 1] e crescente no intervalo [1, +oo), de modo que segue que f realmente atinge seu valor, o fazendo em x = 1. Logo, o valor • mínimo de fé f(l) = 2.

O exemplo a seguir utiliza o estudo da primeira variação de uma função para dar uma outra prova da desigualdade entre as médias aritmética e geométrica. Exemplo 6.45. Dados n > 1 inteiro e reais positivos a 1 , a 2 , ... , an, use a proposição 6.43 para provar que

(b) Suponha que f' > O no interior de 1, e sejam a< b dois pontos quaiquer de 1. Novamente pelo TVM, existe e E (a, b) tal que

f(b) - f(a) = f'(c) > O, b-a

ocorrendo a igualdade se, e só se, a 1

= a 2 = · · · = ªn·

Tópicos de Matemática Elementar 3

238

Prova. Façamos indução sobre n > 1, sendo a prova do caso n = 2 aquela apresentada na seção 7.2 do volume 1. Seja dado k > 2 inteiro e suponha, por hipótese de indução, que já provamos a desigualdade do enunciado para quaisquer n = k - 1 reais positivos, com igualdade se, e só se, eles forem todos iguais. Dados k reais positivos a1, ... , ªk-I, ak, seja f : (O, +oo) ----+ lR. a função dada para x > O por

1

1

f(x) = a1 + · · · + an-1 + X - n{'./a1 ... ªn-1X.

'1 i '

Se mostrarmos que f(x) ~ O para todo x > O, com igualdade se, e só se, a 1 =. · · = ªn-I = x, concluiremos em particular que f(an) ~ O, com igualdade se, e só se, a 1 = · · · = ªn-1 = an, conforme desejado. Para tanto, observe que f é derivável, com

J'(x) = 1 - {/ai ... ªn-1Xl/n-l

de modo que f(x 0 )

~

(6.17)

1

1. Prove a proposição 1.25 utilizando os métodos desenvolvidos nesta seção.

2. Utilize o estudo da primeira variação para encontrar, se houver, o valor máximo da função f : [O, +oo) ----+ lR. dada para x > O _y'x ' _, por f (x) - x2+16. 3. Encontre, se houver, os valores máximo e mínimo da função

f : lR. ----+ lR. dada, para x E :IR., por reais positivos dados.

f (x) =

axJ+b, onde a e b são

4. Prove que ex> 1 + x, para x > O.

f : lR. ----+ lR. é a função dada por f(x) =

mostre que f para todo O :::::; mostrar que f para todo k ~

1/x 2

{

....L

e, se x, O O, se x = O

é duas vezes derivável em JR., com J(k)(o) = O k :::::; 2 (com um pouco mais de trabalho, pode-se é infinitamente derivável em JR., com J(k)(o) = O O).

O, pela hipótese de indução. Logo,

J(x) ~ J(xo) ~ O

1

239

Problemas - Seção 6.3

5. Se

para todo x > O. Portanto, sendo x 0 = n-..ya1 ... ªn-1, temos f' < O em (O, x 0 ), f'(x 0 ) = O e f' > O em (xo, +oo ), de maneira que f é decrescente em (O, x 0 ] e crescente em [xo, +oo ). Assim, atinge seu valor mínimo em x = x 0 , sendo f(x) = f(xo) se, e só se, x = Xo. Por fim, uma simples substituição fornece

f(xo) = a1 + ... + ªn-1 - (n - 1) n-..ya1 ... ªn-1,

Antonio Caminha M. Neto

para todo x > O, ocorrendo a igualdade se, e só se, x = xo e f(xo) = O. Basta agora observar que, graças a (6.17 ) e à hipótese de indução, f(xo) = O se, e só se, a 1 = · · · = ªn-I, de forma que há igualdade se, e só se, ª1 = · · · = ªn-1 = x.



6. Sejam f, g : lR.----+ lR. funções deriváveis, tais que f (O) = O, g(O) = 1, f'(x) = g(x) e g'(x) = - f(x), para todo x E R Prove que f(x) = senx e g(x) = cosx, para todo x E R

f : lR. ----+ lR. uma função duas vezes derivável e tal que f"(x) - - f(x), para todo x E R Mostre que

7. Seja

f(x) = f(O) cosx + J'(ü)senx, para todo x E JR..

Tópicos de Matemática Elementar 3

240 8. Mostre que

(::J f+l ~ (~f, para todos m, n EN.

Antonio Caminha M. Neto 15.

onde x e y são reais tais que

(x +\li+ x2 )(y +

(b) Se f'(a) O em I, então fé estritamente convexa em l. (c) fé côncava em I se, e só se,

f"

255

Analogamente, como u--+ y se, e somente se, t--+ 1, obtemos a partir da primeira desigualdade em (6.26) que

Começamos esta seção mostrando que a concavidade ou convexidade de uma função duas vezes derivável está intimamente relacionada ao sinal de sua segunda derivada. Em seguida, utilizamos tal relação para, em conjunção com a desigualdade de Jensen, obter mais algumas desigualdades interessantes. Teorema 6.57. Se I for um intervalo aberto e vezes derivável em I, então:

Antonio Caminha M. Neto

J'(r) = f(u) - f(x), J'(s) = f(y) - J(u). u-x y-u

:S: O em I. Mas, como f'(r) :S: f'(s) (resp. f'(r) < f'(s)), temos que

(d) Se f" < O em I, então fé estritamente côncava em l. Prova. Façamos a prova dos itens (a) e (b), sendo a prova dos demais itens totalmente análoga. Suponha, inicialmente, que f é convexa. Então, dados x < y em I e O < t < 1, temos

J(u) - f(x) :S: (resp. O escolhido tão pequeno que (x 0 - 8, xo + 8) C I (o outro caso pode ser tratado de forma análoga). Então, pelo teorema anterior, temos J"(x) ~ Oem (x 0 - 8, xo) e f"(x) :'.S: Oem (xo, Xo + 8). Se f"(x 0 ) < O, podemos escolher a E (x 0 - 8, x0 ) tal que J"(a) ~ O. Aplicando agora o teorema de Darboux (cf. problema 15, página 241), concluímos pela existência de b E (a, xo) (logo b E (xo - 8, xo)) tal que f"(b) < O, o que é uma contradição. Se f"(x 0 ) > O, chegamos a uma contradição de modo análogo. Logo, J"(xo) = O. • Os resultados discutidos ao longo deste capítulo permitem esboçar gráficos de funções duas vezes deriváveis com razoável precisão. Vejamos como fazer isto no exemplo a seguir. Exemplo 6.60. Esboce o gráfico da função

f : IR --+ IR dada por

1

J(x)

=

yl + 8ex

Solução. Primeiramente, veja que O < J(x) < 1, para todo x E IR. Ademais, como limx-t+oo ex = +oo e limx--+-oo ex = O, temos que lim f(x) x--++oo

= O e x--+-oo lim J(x) = 1.

Antonio Caminha M. Neto

257

Segue que o gráfico de f está inteiramente contido na faixa do plano Cartesiano delimitada pelas retas y = O e y = 1, e que tal gráfico se aproxima tanto quanto queiramos de tais retas à medida que x --+ +oo e x --+ -oo, respectivamente. Por outro lado, como f(O) = segue que o gráfico de f intersecta o eixo das ordenadas no ponto ( O, Para analisarmos a primeira variação de f, note que um cálculo imediato com o auxílio da regra da cadeia fornece

!,

! ).

4ex

J'(x) = - (1 + 8ex)3/2 < O, para todo x E IR, de sorte que f é decrescente em toda a reta. Por fim, quanto à segunda variação de f, calculando f" (novamente com o auxílio da regra da cadeia) obtemos

Então, o sinal de f" coincide com o sinal de 4ex -1, de sorte que (cf. teorema 6.57) J é estritamente convexa para x > -2 log 2, estritamente côncava para x < -2 log 2 e o ponto x 0 = - 2 log 2 é seu único ponto de inflexão. Por fim, -2 log 2 ~ -1, 38 e J(-2 log 2) = ~ ~ O, 58. Reunindo as informações acima, obtemos o esboço para o gráfico de f constante da figura 6.7

• Terminamos esta seção ilustrando o uso da desigualdade de Jensen em conjunção com o teorema 6.57. Exemplo 6.61 (IMO). Prove que, para todos os reais positivos a, b, e, temos a b e ----;::::==+ + >1. J a2 + bc Jb 2 + ac J c2 + ab -

Tópicos de Matemática Elementar 3

258

Antonio Caminha M. Neto

259

(iii) x ~ -2 log 2 < y, z: novamente pela desigualdade de Jensen, temos

y ---------- y - 1 0,58

f(x)

+ f(y) + f(z) 2 f(x) + 2f ( y; z)

=

f(x)

+ 2f (-~).

X

-1,38

Se g: (-oo, -2 log 2] -+ lR é a função dada por

g(x) = f(x) Figura 6.7: gráfico da função

+ 2f (-~),

X f-7 vl~8eX.

note inicialmente que

Prova. Note, primeiramente, que a expressão do primeiro membro na desigualdade do enunciado é igual a 1

Fazendo ~g provar que

=

1

J1 +

8bc a2

+

Jl

ex, ~g

=

eY e

~g

J1 +

g(-2log2) = -

8ªb. c2

ez, temos x+y+z

=

f(x)

X) f' (-2 = -

O e queremos

g'(x) = f'(x) -

JI7

>

1.

x

x

4e-x/ 2

+ (1 + 8e-x/2)3/2'

E ( -oo, -2 log 7]

=}

g(x) > 1

2

J3

E (-2log7, -2log2)

=}

g(x) > g(-2log2) > 1.



-2 log 2 < z: então 2f(-2log2) =

4ex

(1 + 8ex)3/2

e

+ f(y) + f(z) 2 3f ( x+y+z) = 3f(O) = 1. 3

+ f(y) + f(z) 2 f(x) + f(y) 2

2

+-

de sorte que g'(x) < O se, e só se, -2log7 < x < -2log2. Portanto,

(i) x, y, z > -2 log 2: pela desigualdade de Jensen, temos

~

+ 2 x~+oo lim f(x) = 1

Por outro lado, temos

onde f : lR -+ lR é a função cujo gráfico foi analisado no exemplo anterior. Há, pois, três possibilidades:

(ii) x, y

1

J3

f(x) + f(y) + f(z) 2 1,

f(x)

x~-oo

e

1

+ 8~g + =

lim g(x) = lim f(x)

x~-oo

>

1.

A discussão subsequente fornece uma ampla generalização da desigualdade entre as médias, conhecida como a desigualdade entre as médias de potências. Para tanto, precisamos, inicialmente, definir o que vêm a ser tais médias.

Tópicos de Matemática Elementar 3

260

Definição 6.62. Dados reais positivos a1, a 2, ... , an e a E :IR, definimos a média de potências de ordem a dos números a1, a2, ... , an como o número Ma = Ma(a1, ... , an) tal que M a -- { 1

Antonio Caminha M. Neto

261

ou, ainda,

Portanto, concluímos que é suficiente analisar os casos O < a < /3 e O= a< /3. Sem perda de generalidade, suponha que a 1 = max1::,:i::;n{ai}· Então, como nem todos os a/s são iguais, temos

1/a ,sear_j_ O ( af+a~+·+a\':) n y'a1a2 ... an, se a= O

1

Teorema 6.63. Sejam dados reais positivos a1, a2, ... , ªn· Se a < f3 são reais quaisquer, então

(6.27) ocorrendo a igualdade em uma qualquer das desigualdades acima se, e só se, todos os ai forem iguais. Prova. Podemos supor, sem perda de generalidade, que nem todos os a/s são iguais. Suponha, ainda, que (6.27) valha (com desigualdades estritas) para todos O < a < /3. Dados x < y < O, temos -x > -y > O; como os números...!..., ... ,...!... a1 an também são positivos e nem todos iguais, segue de nossas hipóteses que

Agora, um cálculo fácil fornece

< e, analogamente, min 1::,:i::;n{ ai} < Ma. Resta, pois, mostrarmos que

A primeira desigualdade acima equivale a

que, por sua vez, é uma decorrência imediata da desigualdade entre as médias aritmética e geométrica. Para provarmos a segunda desigualdade, fazendo M 13 = K, basta mostrarmos que -

1

K

para todo t E :IR, de sorte que as desigualdades acima fornecem

(

ª+ ª2ª+ ···+anª)1/a
1 :::::} x4 > x2 e lxl =

1 :::::}

x4 = x2 =

1,

justificando o fato de o gráfico de f 4 estar situado abaixo do gráfico de h no intervalo (-1, 1) e acima fora do intervalo [-1, 1] (veja a figura 7.2).

y x2

respectivamente.

o

X

Figura 7.2: gráficos de Í2 e

X

f4.

4. Resolva, para x E J, a equação f(x) = g(x).

As funções Í3 e f5 são positivas para x > O, negativas para x < O e se anulam em x = O. Também, é claro que se trata de funções ímpares e, daí, seus gráficos são simétricos em relação à origem do sistema Cartesiano. Agora, à medida que lxl aumenta é evidente que os valores de lfl e 191 se tornam cada vez maiores e, eventualmente, ultrapassam qualquer valor prefixado. Por último,

6. As funções h e f 4 são sempre não negativas e só se anulam em x = O. Também, é claro que são funções pares, de forma que seus gráficos são simétricos em relação ao eixo vertical do sistema Cartesiano. Por outro lado, à medida que lxl aumenta, é evidente que os valores de

justificando o fato de o gráfico de f5 estar, no intervalo (-1, 1), mais próximo do eixo horizontal que o gráfico de Í3 e mais distante de tal eixo fora do intervalo [-1, 1] (veja a figura 7.3).

Figura 7 .1: pontos fixos de uma função

f.

278

Tópicos de Matemática Elementar 3

Antonio Caminha M. Neto

279

demais itens podem ser analisados de forma similar.

x3

11. De posse do gráfico de f, mostre que obtemos o gráfico de g do seguinte modo: refletimos, ao longo do eixo das abscissas, a porção do gráfico de f situada abaixo de tal reta. 12. Em princípio, pode parecer que não podemos usar o resultado do problema anterior, haja vista que a função em questão não tem por domínio o conjunto dos reais. Contudo, como X

Figura 7.3: gráficos de Í3 e f5.

2x x+l

2x + 2 - 2 2 -----2--x+l x+l'

podemos raciocinar de modo análogo, esboçando o gráfico de f do seguinte modo: primeiro, traçamos o gráfico de x t--+ i-; em seguida, transladamos o gráfico anterior uma unidade para a esquerda, obtendo o gráfico de x t--+ x.!.i; agora, alongamos o gráfico anterior na direção vertical, pelo fator 2, obtendo o gráfico de x t--+ x!i; refletimos o resultado no eixo horizontal, obtendo x t--+ - x!i; por fim, transladamos o gráfico refletido duas unidades para cima, obtendo o gráfico de x t--+ O resultado final é a figura abaixo:

x!i.

13. Use o resultado do problema 8.2.4 do volume 2. 7. Observe que f é a inversa da função g : lR--+ lR tal que g(x) = x3, para todo x E R Em seguida, aplique o resultado da proposição 2.6. 9. A figura 4.1 esboça o gráfico da função parte fracionária. 10. Para o item (a), basta mostrarmos que (x, y) é um ponto do gráfico de f se, e só se, (x - a, y) é um ponto do gráfico de g. De fato, se (x, y) pertence ao gráfico de f, então f(x) = y e, daí, g(x - a) = J((x - a) + a) = y, quer dizer, (x - a, y) pertence ao gráfico de g; provamos a recíproca do mesmo modo. Para o item (b), basta mostrarmos que (x, y) é um ponto do gráfico de f se, e só se, (x, y+a) é um ponto do gráfico de g. De fato, se (x, y) pertence ao gráfico de f, então J(x) = y e, daí, g(x) = f(x)+a = y+a, quer dizer, (x, y+a) pertence ao gráfico de g; provamos a recíproca do mesmo modo. Os

Seção 2.2 1. Para os itens (a) e (b), referimos o leitor à discussão do exemplo 2.11, mais precisamente à equação (2.2); quanto ao item (c), sugerimos ao leitor rever os itens (a) e (e) do problema 10, página 68. 2. Use uma das fórmulas do arco duplo do problema 7.2.3 do volume 2 para cos2x. 3. Use o resultado do problema 4, página 67. 4. Opere a mudança de variável x =

v'5 cosa, onde O::; a

::; 1r.

Tópicos de Matemática Elementar 3

280

Antonio Caminha M. Neto

281

para k E Z. Ern seguida, se n é ímpar, por exemplo, mostre que a última igualdade implica a igualdade sen ( 5! + 1 cos = O, para todo x E lR. tal que nx # i + k1r para k E Z; conclua, então, que n divide 15.

y

f;)

(1f;)

7. Suponha que tal função seja periódica de período p > O; calcule f(x + p) com a ajuda das fórmulas de adição de arcos e, ern seguida, analise a igualdade f(x + p) = f(x).

(0,2)

--------------+--------------------1 1

1 1 1

Seção 3.1

(-1,0)1

X

1. Mostre que, se fosse a> b, teríamos, eventualmente, an >

Figura 7.4: gráfico de x

f--+

2. Suponha, sem perda de generalidade, que a1 :::; mostre que não podemos ter b1 > b2 > b3 > · · · .

2x/(x + 1).

a2


bn.

a3

< ··· e

3. Inicialmente, observe que 5. Suponhamos, por contradição, que f fosse periódica, de período O. Então, deveria ser f(T) = f(O) = 2. Mas aí 2

= f(T) = COST + cos(aT)

T

>

:::; 1 + 1 = 2,

urna vez que o cosseno de qualquer número real é no máximo 2. Assim, deve ser COST = cos(aT) = 1. Para tanto, devem existir inteiros (não nulos) k, l tais que T = 2k7r e aT = 2l1r. Dividindo membro a membro essas duas igualdades, chegamos a a= l/k, contradizendo o fato de ser a irracional.

n

Ír0 (y'n)k

ViaF = -1ª1---+ ~ < 1 quando n--+ +oo. Portanto, fixado urn real a tal que

. t e no E ex1s

ThT 1 '1

!

1 1

no :::::} V n Jr0 . d a, íaf nk íaf < a ou, am < an . k

Mas, corno an --+ O quando n --+ +oo, o rnesrno sucede com nn. a Alternativamente, faça ial = 1 + a, com a > O, e veja que, para n > k,

laln = (1 +ar=

t (~)aj > ( j=O

6. Inicialmente, mostre que a igualdade f (x + 31r) = f (x) implica a igualdade ( -1 rsen ( 5! + l~7r) = sen ( 5!), sempre que nx # i + k1r

1

J

_ n(n - 1) ... (n - k) k+l (k+l)! ·a ·

n )ak+l

k

+1

Tópicos de Matemática Elementar 3

282 Portanto, para n

> k, temos

nk

< laln

Antonio Caminha M. Neto 9. Use o fato de tn = -to - t1 - · · · - tn--l = O para escrever

(k + 1)! ak+l (k + 1)! ak+l

ak = to(Vk - ~ )

nk

+ tn--1 ( v'k + n

n(n - 1) ... (n - k)

1 n(l -

i) ... (1 -

-t

>

nk, a1, a2, · · ·, ªnk-1 ·

5. Escreva e = (1 - tk)c + tkc e use a desigualdade triangular para escrever lck - cl :S (1 - tk)lak - cl + tklbk - cl6. Note primeiro que lbn - li :S max{lan - li, lcn - li}, para todo n EN. Portanto, dado E > O e tomando no E N tal que n > no ==} lan li, lcn - li < E, teremos lbn - li < E para n > no, de modo que bn -t l.

2

+ n~~% 2

e, em seguida, faça k

-t

+n -t +oo. v'k

+oo.

4. Construa, indutivamente, uma subsequência (ank)k2'.1 tal que ank

2

v'k

+ n) + · · ·

~)'

Por fim, faça k

n:~%

-

+ t1(vk+l 1 - v'k + n)

to

com a última expressão acima tendendo a O quando n

7. Mostre que Iam - anl :S

283

- 1 + v'k

10. Faça a substituição trigonométrica Xn = 2 cos Yn, com Yn E [O,~], use um pouco de trigonometria e conclua que Yn+l ~ 2yn, para todo n ~ 1. A partir daí, mostre que Yn :S Yn2t 2 k :S 2k7'.;_ 1 , para todo k ~ 1, de sorte que Yn = O para todo n ~ 1. 11. Inicialmente, use a condição do enunciado para mostrar que l(an+l an) - (ak+l - ak)I < ~- Em seguida, fazendo n -t +oo, conclua que existe l = limn--,+oo(an+l - an), e mostre que ak+l - ªk = l, para todo k EN. 12. Conclua, a partir da definição de an, que ªk--l + 1 = inteiro k ~ 2. A partir daí, mostre que

+oo.

8. Se a= min{a1,a2} e f3 = max{a1,a2}, conclua que (an)n>l é limitada, considerando separadamente os casos O < a ::::; f3 ::::; 4, 4 ::::; a ::::; f3 e O < a ::::; 4 ::::; (3. Se a1 ::::; a2 ::::; a3 ou a1 ~ a2 ~ a3, escreva ªk+2 - ªk+l = yfaw - vªk--1 para concluir que, caso a sequência dada é monótona, ela será convergente; fazendo k -t +oo na recorrência do enunciado, conclua que ak -t 4. Se a1 ::::; a3 ::::; a2 ou a1 ~ a3 ~ a2, conclua, como acima, que as subsequências de índices pares e ímpares são convergentes, digamos a2k--l -t e e a 2 k -t d; fazendo k -t +oo na recorrência do enunciado, conclua que d= Jc+v'd e e = y'c + v'd, de sorte que e = d = 4.

+n·

ªk--l - ak


l é não crescente para n > 2. Por fim, faça k -t +oo na igualdade ªk--1 + 1 = k!l ak para concluir que an -t 1 quando n -t +oo. 13. Mostre, sucessivamente, que a;;= k+an--1 e a;;,+ 1 -a;;= an-an--1, e conclua, a partir daí, que a sequência (an)n2:1 é crescente. Use agora que a;;, < k + an para concluir que an < ~ (1 + v'4k + 1) para todo n ~ 1, de sorte que a sequência em questão é convergente. Agora, faça n -t +oo em a;;,= k + ªn--l para concluir que an -t ~(1 + v'4k + 1) quando n -t +oo. A partir daí, os itens (b) e (c) são relativamente imediatos.

Tópicos de Matemática Elementar 3

284

14. Primeiramente, use a desigualdade entre as médias aritmética e geométrica para concluir que ªk+I 2: Ja, para todo k E N. Agora, faça k ---+ +oo na recorrência acima para concluir que, se (an)n>l for convergente, então seu limite é igual a Ja. Para o que falta, use a desigualdade triangular para obter

!2 lak -

ªk-1

!2 lak -

ªk-11 · 11 -

+ .!!____ ªk

-ª-1

Antonio Caminha M. Neto Fixe k Po

>

285

no, np, de modo que ....E:k..... ªk+p

> p tal que

>

y; conclua pela existência de

-ªk - 2 : y > -ªk- ªk+po

Por fim, suponha que

ªk ªk+Po+l

ªk+po+l

:S x e chegue a uma contradição.

16. Seja co, c1, c2,, .. a sequência definida por co

=

1e

ªk-1

ª

ªkªk-1

ªn-1) Cn = ( l +an Cn-1,

1

'in 2: 1.

1

< 2lªk - ªk-11, para k > 2. Por fim, conclua a convergência de (an)n::=::1 a partir do exemplo 3.17. 15. Prove inicialmente que ªn+l 2: an + 1 e, daí, que an 2: n para todo n 2: 1. Agora, use produtos telescópicos para mostrar que, fixado p EN, temos

Reescrevendo esta relação como Cn soma telescópica (7.1) Por outro lado, a assertiva do problema equivale a termos ~ Cn-1


N, temos

e, daí, p-1

1 < ªn+p < an

p-1 ( 1 ) II ªn+j+l np ::::} --2:1.L > y. Utilizando que ak > k para todo an+p k, mostre que podemos escolher no natural tal que l+ 1 v1ªno+1 Em seguida, conclua que n > no e p natural fornecem

2(1+!+··+,\-) é uma constante positiva. Se 2k-l :S n
n 2, válida para todo

C·(m-r) 2 mostrando que a sorna dos Cn pode ser tornada arbitrariamente grande. Contudo, por (7.1) essa sorna nunca pode exceder ao. Essa contradição mostra que __fn_ < 2- 1/n se verifica para infinitos n EN, conforme

6. Inicialmente, use o teste da comparação para mostrar que a série Lj2:l converge. Agora, denotando por x o valor de sua sorna, observe que, para k ~ n,

tdj

Cn-1

desejado.

n



9



x- " '-J = """ L,; lüj L,; _J lüj < - """ L,; lüj j=l j2:n+l j2:n+l

Seção 3.2

1ª = 7. Use que -n+l

!,;

...!.... - a n1+ 1 para todo an

1 lün

n E N.

1. Faça Sn = L~=l 2 1 e, ern seguida, mostre que

1 1 1 ) 2n - 1 - + · · · +an-1 -- - (a - l)Sn =aSn -Sn =a+2 ( a + a2 an-

= a+ ~ a -1

a

2n - 1. an

an

a~l

+ (a~l) 2 quando n ---+ +oo.

2. Seja r > O a razão da PA. Para o item (a), ternos 1

1

ªk

a1 + (k - l)r

-=

1 2(k - l)r

>----

se k > ªj + 1. Para o item (b), ternos 1

\~,

(! _~) _

Agora, use os resultados do exemplo 3.3 e do problema 3, página 90, para concluir que Sn ---+

da

8. Use que a~+ ~ para todo k E N; ern seguida, aplique o resultado da proposição 3.22.

1 -= a2k a1 + (2k - l)r


trica do teorema de Ceva (cf. problema 7.3.37 do volume 2) para



= I: ai(ªi +

n

bit- 1

i=l

+ I: bi(ªi + bit- 1 . i=l

i

!

Em seguida, escolha q > O tal que = 1 e aplique a desigualdade de Hõlder a cada um dos somatórios acima.

Seção 6.1 1. Adapte, ao presente caso, a demonstração da unicidade do limite de uma sequência, dada na proposição 3.5. 6. Multiplique o numerador e o denominador da fração por 1 + cos x. Em seguida, use o limite trigonométrico fundamental. 7. Para a segunda parte, analise o comportamento de f ao longo das sequências (an)n?.I e (bn)n?.I, tais que an = 2~1r e bn = 2n1r.!.1r7 2 , para todo n 2:: 1. 10. Adapte, ao presente caso, a discussão do exemplo 4.17. 11. Comece fazendo x = 1 em (a) para obter f(J(y)) = yf(l), para todo y > O. Em seguida, use essa relação para mostrar que f é injetiva e, então, que f(l) = 1 e J(J(x)) = x, para todo x > O. Se a > O

Tópicos de Matemática Elementar 3

302

i

é ponto fixo de f, mostre que também o é, de sorte que podemos supor que a 2: 1. Conclua que ak é ponto fixo de f para todo k EN, e use (b) para mostrar que a = 1. Por fim, fazendo x = y em (a), para todo x > O. conclua que J(x) =

1,

Seção 6.2

Antonio Caminha M. Neto

10. Sendo T > O o período de f, mostre que j(k)(T) = j(k)(o), para todo inteiro k 2: O. Conclua que, para O :::; k :::; n - 1, tais equações fornecem um sistema de equações nas incógnitas cos(jT), 1 :::; j :::; n, cuja única solução é cos(jT) = 1, para todo 1 :::; j :::; n - para provar este último fato, você terá de utilizar o resultado da proposição 6.5 do volume 6. 11. Provemos que J'(vk) = O. Como vk (j. Q, segue que J(vk) Daí, sendo x um irracional diferente de vk temos

5. Adapte, ao presente caso, a discussão do exemplo 6.30.

Basta, pois, mostrarmos que

+ r~~:oo) = g'( XQ ) + s(x-xo)' x-xo f'(xo)

lim J(x) - f(vk) - O X - ,vk - .

x-+v'k xEIQ)

.r(x-xo) s(x-xo) _ O . . com 1Imx---+xo x-xo e 1Imx---+xo x-xo - . 7. Pela continuidade da função logaritmo natural, basta mostrar que lim x log x---t+oo

(1 + ~) = a X

. 1) , que 1·1my---+O+ ou, ainda (fazendo y = x use a regra de l'Hôspital.

log(l+ay) Y

= a.

p ara t ant o,

9. Para k 2: O, ponha j(k)(x) = ake-xsenx + bkcx cosx, de sorte que

+ bk)e-xsenx + (ak -

Para tanto, seja x = ~, com p, q E N, uma fração irredutível. Temos

f(x) - !( vk) x-vk

1

/p/q-vk/ =

8. Escolha um sistema Cartesiano de coordenadas tal que F(O, e) e d : {y = -e}. Em seguida, obtenha a equação de P em um tal sistema, escreva Pi(ai, bi), para i = 1, 2, e calcule a abscissa Xi do ponto de interseção da tangente a P traçada por Pi com a diretriz d. Por fim, use a colinearidade de Pi, P2 e F para concluir que X1 = x2.

lk+l\x) = -(ak

= O.

_f(.;._x'---)-_f'----'(c--vk--'-k) = 0 x-vk ·

6. Inicialmente, verifique que a discussão que precede o lema 6.27 nos permite escrever, para x E I \ { xo},

f(x) g(x)

303

bk)e-x cosx

e, portanto, ªk+l = -(ak +bk), bk+I = ak-bk. Conclua, a partir daí, que bk+ 2 + 2bk+1 + 2bk = O, para k 2: O, e use o material da seção 4.3 do volume 1 para obter j(k) (O) = bk.

p + qvk q2 /p2 - q2 k 1

q2/p-q,vk/

< p + q,vk q2

-

!

- q

(P.q + Fk)k .

Dado E> O, tome n EN tal que n > 2v;+ 1 e seja

A= {p/q; p, q EN, O< q :S n}. Veja que A é um conjunto finito. Tomando

J temos que /~ - vk/

= min{l,

{/x -

Vk/;

x

E A}},

< J =} q 2: n, ao passo que ~ < vf + 1 implica

f(x) - f(vk) x-vk

=

! q

(P. + yk) < 2,/f + 1 < q

n

E.

Tópicos de Matemática Elementar 3

304

Antonio Caminha M. Neto

(x+;t+

Seção 6.3 4. Aplique o resultado da proposição 6.43 à função dada por f(x) = ex - x - 1.

f

[O, +oo) --+ IR

5. Claramente, basta mostrarmos que j(k) (O) existe e é igual a O, ~ara todo k # O. Para tanto, observe inicialmente que limx---+O e-l/x = O = f(O), de sorte que f é contínua em R Examinemos, ~gora a primeira derivada de f. Para x # O, temos f'(x) = e-l/x ; para x = O, observe inicialmente que

1 , x > O, 8. Conclua que é suficiente mostrar que a função f(x) = atinge seu valor mínimo em x = n. Para tanto, estude a primeira variação de f.

9. Use o TVM em conjunção com o resultado do problema 7, página 150. 10. Mostre, inicialmente, que

;3 ·

X-Ü

Como et 2

i, temos t

t

;t2·

> 1 + t 2 pelo problema anterior, segue que l

f(x)-f(O) 1 = ~ < X -

Ü

et

_it_i --+ O

x) +

tcv1 +

x2

+ x)).

11. Use a desigualdade entre as médias para dois números para concluir que a expressão acima é maior ou igual que f(x + y), onde f(t) = t + + fi para t > O. Em seguida, estude a primeira variação da função f.

f

1 + t2

quando t--+ ±oo. Portanto, fé derivável em x = O, com f'(ü) = O. Para a segunda derivada, note que, para x # O, temos f'(x) = ;fr · f(x). Portanto, pela regra do produto, segue que

J"(x) = ( 4

~ (p( v1 + x 2 -

x

X

--+ ±oo quando x --+ O e f(xl=l(O)

y =

Em seguida, estude a variação da função f : IR--+ IR tal que f(x) = + y, onde y é dado em função de x como acima.

f(x) - f(O) _ e-l/x2 Fazendo t =

305

~~x2) f(x)

para x # O, e um argumento análogo ao feito acima para calcular J'(O) mostra que f"(ü) existe e também vale O. 6. Faça h(x) = f(x)senx + g(x) cosx e l(x) = f(x) cosx - g(x)senx, de sorte que h(O) = 1 e l(ü) = O. Em seguida, calcule h'(x) e l'(x) e conclua que h é constante e igual a 1, ao passo que l é constante e igual a O. 7. Se g, h: IR--+ IR são tais que g(x) = f(x) - f(O) cosx e h(x) = g'(x), para todo x E IR, mostre que h'(x) = -g(x). Em seguida, use o resultado do problema anterior para mostrar que g(x) = f'(ü)senx.

12. Fazendo a

= sen

4e /3 = sen J, mostre que basta analisar a igualdade

f(a) = f(/3), onde f(x) = (l~;~)24, x E (O, 1). Estude, então, a primeira variação de f. 13. Aplique o teorema de Rôlle à função h : [a, b] --+ IR dada, para x E [a, b], por h(x) = (f(b) - f(a))g(x) - (g(b) - g(a))J(x), x E [a, b]. 14. Tome, em I \ {e}, uma sequência (xn)n2:1, com Xn --+ e. Se Xn > e (resp. Xn < e), então, aplicando o TVM ao intervalo [e, xn] (resp. [xn, e]), concluímos pela existência de Yn E (e, Xn) (resp. Yn E (xn, e)) tal que

f'(Yn) = f(xn) - J(c). (7.3) Xn -e Em qualquer caso, temos Yn --+ e e Yn # e, de forma que, por nossas hipóteses, temos f' (Yn) --+ L. Portanto, segue de (7.3) que f(xn)- f(c) xn-c

--+

L Ü · , . . . rac10c1mo acima mostrou que

Xn--+ e::::} f(xn) - f(c) --+ L. Xn -e

Tópicos de Matemática Elementar 3

306

Mostre que isso é suficiente para garantir que limx--+c

f(xl=I(c)

existe

e é igual a L. 15. Para o item (b), use o resultado de (a). Para o item (a), considere dois casos separadamente: se f não for injetiva em [a, b] mostre que basta usar o TVM; se f for injetiva em (b), conclua que f será monótona e, daí, que

f' 2:

O em [a, b] ou

f' :S O em

[a, b].

16. Comece utilizando a relação do enunciado para mostrar que, se uma tal f existisse, então teríamos f' (x) :S -1, para todo x > O. Em seguida, use o TVM para mostrar que f deveria assumir valores negativos. 17. Aplique repetidas vezes o seguinte fato: se k E lR é tal que f (x) + kf'(x) 2: O para todo x E JR, então J(x) 2: O para todo x .E R Para provar tal fato, comece utilizando o resultado do problema 10, página 212, para mostrar que f tem grau par e coeficiente líder positivo. Em seguida, tome (pelo problema 5, página 149) xo E lR tal que f atinge seu valor mínimo em xo e aplique o corolário 6.38. 18. Use o TVM para garantir a existência de e E (a, b) tal que J'(c) = 1. Se f(c) = e, mostre que basta aplicar o TVM mais duas vezes. Se f(c) < e, use o TVM para garantir a existência de a E (O, e) e (3 E ( e, 1) tais que f' (a) < 1 e f' (,B) > 1; em seguida, aplique o teorema de Darboux (cf. problema 15). Se f(c) > e, argumente de maneira análoga. 19. Comece usando o TVM para mostrar que existe O < 8 < 1 tal que Jf(x)J < JxJ, para JxJ < 8. Itere esse argumento, mostrando que (com o mesmo 8) Jf(x)J :S JxJn, para lxl < 8 e todo n EN. Conclua, a partir daí, que f(x) = O, para lxl < 8. Em seguida, estenda o argumento acima, mostrando que, se f(xo) = O, então existe 8 > O tal que J(x) = O, para lx - xol < 8. Por fim, use o resultado do problema 12, página 140, para concluir que f(x) = O, para todo x E R

Antonio Caminha M. Neto

307

Seção 6.4 4. Estude a primeira variação da função J(x) = (1 + l)x (O )( . . x , para x E '+oo aqm, para x, g(x) > O, definimos g(x)x como exp(x logg(x))). 5. Derive g(x) = e-x f(x) e utilize a proposição 6.43. 6. Adapte, ao presente caso, a discussão do exemplo 6.56. 7. Por contradição, derive a igualdade p(x) -- log x e , em segm"d a, use o resultado do problema 10, página 212 . 8. Tomando logaritmos naturais, concluímos que basta comparar os nú· · varia,ção da função meros le e log7l' 7l' • p ara t anto, estude a primeira f: (0,+oo)--+ lR dada por f(x) = lo~x, para x E (0,+oo), mostrando que seu valor mínimo é atingido somente em x = e. 9. Adapte a sugestão dada ao problema anterior. 10. Aplique o TVM usual à integral indefinida de

f

baseada no ponto a.

11. Inicia~~ente, use os cálculos do exemplo 6.53 para mostrar que existe uma umca fo E F da forma fo(x) = Acosx+Bsenx, com A,B E R Agora, expanda a desigualdade fo'll' (f (x) - f o(x) )2 dx 2: O para chegar ao resultado desejado. 12. Para o item (a), estude a variação da função x f---+ log(x + 1) - x ~ 2: O. :ªra o item (b), observe inicialmente que, pela fórmula d~ mtegraçao por partes, temos

r1

rl xn nxn-l rl d n lo xn + a dx = n lo x. xn + a dx = lo x. dx log(xn + a)dx

= log(a + 1)

-1 11 1

log(xn + a)dx.

Por outro lado, pelo item (a), temos loga

< lal log(xn + a)dx = < loga + rl xn dx. lo a

(ioga+ log ( :n + 1)) dx

Tópicos de Matemática Elementar 3

308

Por fim, combine os cálculos acima para inferir o limite do enunciado.

Antonio Caminha M. Neto onde s como

= a1

+ a2 + · · · + an; a partir daí, reescreva tal desigualdade

tlog ( 1

Seção 6.5

i=l

2. É imediato que J(x) < 1 para todo x E lR e limlxl-++oo e-l/x2 = 1. Assim, o gráfico de f é simétrico em relação ao eixo das ordenadas (pois f é par), está inteiramente contido na faixa do plano Cartesiano situada entre as retas y = O e y = 1 e se aproxima mais e mais da reta y = 1 quando x --+ ±oo. Agora, mostramos no problema 5, página 239, que f é duas vezes derivável, com f'(O) = J"(ü) = O e, para

x#O, -l/x2 f '( x ) = ~. 3 e X

2 eJ"( x) = (4 - 66x ) • e -1/x2 ..

X

Portanto, f é crescente em [O, +oo), decrescente em ( -oo, O], estritamente convexa para lxl < ,J6 e estritamente côncava para lxl > ,J6' De posse das informações acima, esboçe o gráfico de f. 3. Use a desigualdade entre as médias de potências. 4. Use a concavidade estrita da função f (x) = log juntamente com a desigualdade de Jensen. 5. Use a convexidade estrita da função f (x) com a desigualdade de Jensen.

t;~,

O < x < 1,

= x log x, x > O, juntamente

6. Aplique a desigualdade de Jensen à função f(x) = log

se!x, x E (O, 1r).

7. Faça ai= exi, com Xi ~ O, mostre que a função J(x) = ex~ 1 , x ~ O, é estritamente convexa e aplique a desigualdade de Jensen. 8. Comece escrevendo a desigualdade do enunciado como n

1 s II 1-ai a· :S nn+l . 1- s' i=l

i

309

:i .) ai

:S -(n+ l)logn+log ( ~ ) . 1

s

Se ai < ! para 1 :Si :S n, aplique a desigualdade de Jensen à função

f(x) = log ( 1.:x), O< x